You are on page 1of 208

12/18/2020 Mock Analysis

LST Mock 20 2021 (CLAT)

Scorecard (ScoreCard.jsp?sid=aaa2ozgizeSUWcTOTxZzxFri Dec 18 14:02:38 IST


2020&qsetId=Lw5e7noFooA=&qsetName=LST Mock 20 2021 (CLAT))

Accuracy (AccSelectGraph.jsp?sid=aaa2ozgizeSUWcTOTxZzxFri Dec 18 14:02:38 IST


2020&qsetId=Lw5e7noFooA=&qsetName=LST Mock 20 2021 (CLAT))

Qs Analysis (QsAnalysis.jsp?sid=aaa2ozgizeSUWcTOTxZzxFri Dec 18 14:02:38 IST


2020&qsetId=Lw5e7noFooA=&qsetName=LST Mock 20 2021 (CLAT))

Video Attempt (VideoAnalysis.jsp?sid=aaa2ozgizeSUWcTOTxZzxFri Dec 18 14:02:38 IST


2020&qsetId=Lw5e7noFooA=&qsetName=LST Mock 20 2021 (CLAT))

Solutions (Solution.jsp?sid=aaa2ozgizeSUWcTOTxZzxFri Dec 18 14:02:38 IST


2020&qsetId=Lw5e7noFooA=&qsetName=LST Mock 20 2021 (CLAT))

Bookmarks (Bookmarks.jsp?sid=aaa2ozgizeSUWcTOTxZzxFri Dec 18 14:02:38 IST


2020&qsetId=Lw5e7noFooA=&qsetName=LST Mock 20 2021 (CLAT))

English Language

Current Affairs Including General Knowledge

Legal Reasoning

Logical Reasoning

Quantitative Techniques

Sec 1

https://www.aspiration.ai/LAW/sis/Solution.jsp?qsetId=Lw5e7noFooA=&qsetName=LST Mock 20 2021 (CLAT) 1/208


12/18/2020 Mock Analysis

Directions for questions 1 to 30: Each set of questions in this section is based on a single passage. Please
answer each question on the basis of what is stated or implied in the corresponding passage. In some
instances, more than one option may be the answer to the question; in such a case, please choose the option
that most accurately and comprehensively answers the question.

Passage – 1

The Chinese government will build a "super" dam on the Yarlung Zangbo river's lower reaches close to the Line
of Actual Control (LAC) in Tibet, according to a Chinese State media report. The trans-border river, which
originates in the Tibet Autonomous Region (TAR), ows into Arunachal Pradesh where it is called Siang, and
then to Assam as the Brahmaputra before owing into Bangladesh. The new dam's ability to generate
hydropower could be three times that of central China's Three Gorges Dam, which has the largest installed
hydropower capacity in the world.

As a lower riparian country, India has the right to be worried because this can have far-reaching impacts on the
lives and livelihoods of its people. According to the United Nations, more than half the global population will live
in water-stressed or water-scarce countries by 2025, and the vast majority of these people will be in China and
India. While China is home to almost 20% of the world population, it has only about 7% of water resources. India
is home to about 17% of the world's population but has less than 4% of water resources. It is dependent on
foreign originating rivers for about a third of its surface water. As both grow economically, water consumption
will increase and it will automatically place ever-greater pressure on water supplies. Macro challenges such as
the climate crisis and pollution will further strain freshwater resources.

According to Yan Zhiyong, chairman of the Power Construction Corp of China, the 60 million kWh hydropower
exploitation at the downstream of the Yarlung Zangbo River could provide 300 billion kWh of clean, renewable
and zero-carbon electricity annually. The project will play a signi cant role in realizing China's goal of reaching a
carbon emissions peak before 2030 and carbon neutrality in 2060.

"It is a project for national security, including water resources and domestic security," the Power China chairman
said, noting that the project will also smooth cooperation with South Asia.

India must explore all diplomatic and economic options to bring China to the discussion table. But whatever it
does, timing will be the key to make the right impact, says 'Water Wars: The Brahmaputra River and Sino-Indian
Relations', a paper by the US Naval War College. It has to be stressed that hydropower projects on cross-border
rivers cannot be developed without communication and cooperation between upstream and downstream
countries. The earlier India pushes back against Chinese dam building, the more options will be available to it in
the future.

Q.1 [30465398]
Out of the following options, which one is a synonym of the word 'riparian' as used in the passage?

a Anodyne

b Supine

c Saline

d Riverine

https://www.aspiration.ai/LAW/sis/Solution.jsp?qsetId=Lw5e7noFooA=&qsetName=LST Mock 20 2021 (CLAT) 2/208


12/18/2020 Mock Analysis

Solution:
 Answer key/Solution
Correct Answer : d
Your Answer : d
The word 'riparian' refers to anything related to a river or wetland.
FeedBack Bookmark

Directions for questions 1 to 30: Each set of questions in this section is based on a single passage. Please
answer each question on the basis of what is stated or implied in the corresponding passage. In some
instances, more than one option may be the answer to the question; in such a case, please choose the option
that most accurately and comprehensively answers the question.

Passage – 1

The Chinese government will build a "super" dam on the Yarlung Zangbo river's lower reaches close to the Line
of Actual Control (LAC) in Tibet, according to a Chinese State media report. The trans-border river, which
originates in the Tibet Autonomous Region (TAR), ows into Arunachal Pradesh where it is called Siang, and
then to Assam as the Brahmaputra before owing into Bangladesh. The new dam's ability to generate
hydropower could be three times that of central China's Three Gorges Dam, which has the largest installed
hydropower capacity in the world.

As a lower riparian country, India has the right to be worried because this can have far-reaching impacts on the
lives and livelihoods of its people. According to the United Nations, more than half the global population will live
in water-stressed or water-scarce countries by 2025, and the vast majority of these people will be in China and
India. While China is home to almost 20% of the world population, it has only about 7% of water resources. India
is home to about 17% of the world's population but has less than 4% of water resources. It is dependent on
foreign originating rivers for about a third of its surface water. As both grow economically, water consumption
will increase and it will automatically place ever-greater pressure on water supplies. Macro challenges such as
the climate crisis and pollution will further strain freshwater resources.

According to Yan Zhiyong, chairman of the Power Construction Corp of China, the 60 million kWh hydropower
exploitation at the downstream of the Yarlung Zangbo River could provide 300 billion kWh of clean, renewable
and zero-carbon electricity annually. The project will play a signi cant role in realizing China's goal of reaching a
carbon emissions peak before 2030 and carbon neutrality in 2060.

"It is a project for national security, including water resources and domestic security," the Power China chairman
said, noting that the project will also smooth cooperation with South Asia.

India must explore all diplomatic and economic options to bring China to the discussion table. But whatever it
does, timing will be the key to make the right impact, says 'Water Wars: The Brahmaputra River and Sino-Indian
Relations', a paper by the US Naval War College. It has to be stressed that hydropower projects on cross-border
rivers cannot be developed without communication and cooperation between upstream and downstream
countries. The earlier India pushes back against Chinese dam building, the more options will be available to it in
the future.

https://www.aspiration.ai/LAW/sis/Solution.jsp?qsetId=Lw5e7noFooA=&qsetName=LST Mock 20 2021 (CLAT) 3/208


12/18/2020 Mock Analysis

Q.2 [30465398]
What can be said about the tone of the author in the passage?

a Rhetoric

b Sardonic

c Didactic

d Sadistic

Solution:
 Answer key/Solution
Correct Answer : c
Your Answer : c
Refer to the last paragraph of the passage where the author gives his opinion on
what India should do to address the issue described in the passage. Hence, option (c) is the correct answer.
There is nothing mocking in the passage. Hence, option (b) is negated. Furthermore, options (a) and (d) are
inappropriate in the context of the passage as well.
FeedBack Bookmark

https://www.aspiration.ai/LAW/sis/Solution.jsp?qsetId=Lw5e7noFooA=&qsetName=LST Mock 20 2021 (CLAT) 4/208


12/18/2020 Mock Analysis

Directions for questions 1 to 30: Each set of questions in this section is based on a single passage. Please
answer each question on the basis of what is stated or implied in the corresponding passage. In some
instances, more than one option may be the answer to the question; in such a case, please choose the option
that most accurately and comprehensively answers the question.

Passage – 1

The Chinese government will build a "super" dam on the Yarlung Zangbo river's lower reaches close to the Line
of Actual Control (LAC) in Tibet, according to a Chinese State media report. The trans-border river, which
originates in the Tibet Autonomous Region (TAR), ows into Arunachal Pradesh where it is called Siang, and
then to Assam as the Brahmaputra before owing into Bangladesh. The new dam's ability to generate
hydropower could be three times that of central China's Three Gorges Dam, which has the largest installed
hydropower capacity in the world.

As a lower riparian country, India has the right to be worried because this can have far-reaching impacts on the
lives and livelihoods of its people. According to the United Nations, more than half the global population will live
in water-stressed or water-scarce countries by 2025, and the vast majority of these people will be in China and
India. While China is home to almost 20% of the world population, it has only about 7% of water resources. India
is home to about 17% of the world's population but has less than 4% of water resources. It is dependent on
foreign originating rivers for about a third of its surface water. As both grow economically, water consumption
will increase and it will automatically place ever-greater pressure on water supplies. Macro challenges such as
the climate crisis and pollution will further strain freshwater resources.

According to Yan Zhiyong, chairman of the Power Construction Corp of China, the 60 million kWh hydropower
exploitation at the downstream of the Yarlung Zangbo River could provide 300 billion kWh of clean, renewable
and zero-carbon electricity annually. The project will play a signi cant role in realizing China's goal of reaching a
carbon emissions peak before 2030 and carbon neutrality in 2060.

"It is a project for national security, including water resources and domestic security," the Power China chairman
said, noting that the project will also smooth cooperation with South Asia.

India must explore all diplomatic and economic options to bring China to the discussion table. But whatever it
does, timing will be the key to make the right impact, says 'Water Wars: The Brahmaputra River and Sino-Indian
Relations', a paper by the US Naval War College. It has to be stressed that hydropower projects on cross-border
rivers cannot be developed without communication and cooperation between upstream and downstream
countries. The earlier India pushes back against Chinese dam building, the more options will be available to it in
the future.

Q.3 [30465398]
The author of the passage is most likely to agree with which of the following statements:

a China's foreign policy is imperialistic to say the least.

b Accord among concerned nations is required before dams on transnational rivers are built.

c The UN has to be consulted before dams on transnational rivers can be built.

d Unanimity among concerned nations is required if a thermal power station is built.

https://www.aspiration.ai/LAW/sis/Solution.jsp?qsetId=Lw5e7noFooA=&qsetName=LST Mock 20 2021 (CLAT) 5/208


12/18/2020 Mock Analysis

Solution:
 Answer key/Solution
Correct Answer : b
Your Answer : d
Refer to the second last sentence of the passage for the answer where the author
mentions about cooperation among nations when hydropower projects are built. Although international
cooperation is required, it is not stated in the passage that the UN has to be consulted. Hence, option (c) is
negated. Option (d) is wrong because in the passage, hydropower station is mentioned and not thermal power
station. Option (a) is out of scope.

FeedBack Bookmark

https://www.aspiration.ai/LAW/sis/Solution.jsp?qsetId=Lw5e7noFooA=&qsetName=LST Mock 20 2021 (CLAT) 6/208


12/18/2020 Mock Analysis

Directions for questions 1 to 30: Each set of questions in this section is based on a single passage. Please
answer each question on the basis of what is stated or implied in the corresponding passage. In some
instances, more than one option may be the answer to the question; in such a case, please choose the option
that most accurately and comprehensively answers the question.

Passage – 1

The Chinese government will build a "super" dam on the Yarlung Zangbo river's lower reaches close to the Line
of Actual Control (LAC) in Tibet, according to a Chinese State media report. The trans-border river, which
originates in the Tibet Autonomous Region (TAR), ows into Arunachal Pradesh where it is called Siang, and
then to Assam as the Brahmaputra before owing into Bangladesh. The new dam's ability to generate
hydropower could be three times that of central China's Three Gorges Dam, which has the largest installed
hydropower capacity in the world.

As a lower riparian country, India has the right to be worried because this can have far-reaching impacts on the
lives and livelihoods of its people. According to the United Nations, more than half the global population will live
in water-stressed or water-scarce countries by 2025, and the vast majority of these people will be in China and
India. While China is home to almost 20% of the world population, it has only about 7% of water resources. India
is home to about 17% of the world's population but has less than 4% of water resources. It is dependent on
foreign originating rivers for about a third of its surface water. As both grow economically, water consumption
will increase and it will automatically place ever-greater pressure on water supplies. Macro challenges such as
the climate crisis and pollution will further strain freshwater resources.

According to Yan Zhiyong, chairman of the Power Construction Corp of China, the 60 million kWh hydropower
exploitation at the downstream of the Yarlung Zangbo River could provide 300 billion kWh of clean, renewable
and zero-carbon electricity annually. The project will play a signi cant role in realizing China's goal of reaching a
carbon emissions peak before 2030 and carbon neutrality in 2060.

"It is a project for national security, including water resources and domestic security," the Power China chairman
said, noting that the project will also smooth cooperation with South Asia.

India must explore all diplomatic and economic options to bring China to the discussion table. But whatever it
does, timing will be the key to make the right impact, says 'Water Wars: The Brahmaputra River and Sino-Indian
Relations', a paper by the US Naval War College. It has to be stressed that hydropower projects on cross-border
rivers cannot be developed without communication and cooperation between upstream and downstream
countries. The earlier India pushes back against Chinese dam building, the more options will be available to it in
the future.

Q.4 [30465398]
Out of the following options, which one is an appropriate title of the given passage?

a Concerns on China's dam building

b An overview of the river Brahmaputra

c The Chinese government's discrimination against India

d Riparian policies of the modern day nations

https://www.aspiration.ai/LAW/sis/Solution.jsp?qsetId=Lw5e7noFooA=&qsetName=LST Mock 20 2021 (CLAT) 7/208


12/18/2020 Mock Analysis

Solution:
 Answer key/Solution
Correct Answer : a
Your Answer : a
Only option (a) is speci c and captures the essence of the passage. The
remaining options are generic in scope and so, cannot be appropriate titles of the passage.
FeedBack Bookmark

https://www.aspiration.ai/LAW/sis/Solution.jsp?qsetId=Lw5e7noFooA=&qsetName=LST Mock 20 2021 (CLAT) 8/208


12/18/2020 Mock Analysis

Directions for questions 1 to 30: Each set of questions in this section is based on a single passage. Please
answer each question on the basis of what is stated or implied in the corresponding passage. In some
instances, more than one option may be the answer to the question; in such a case, please choose the option
that most accurately and comprehensively answers the question.

Passage – 1

The Chinese government will build a "super" dam on the Yarlung Zangbo river's lower reaches close to the Line
of Actual Control (LAC) in Tibet, according to a Chinese State media report. The trans-border river, which
originates in the Tibet Autonomous Region (TAR), ows into Arunachal Pradesh where it is called Siang, and
then to Assam as the Brahmaputra before owing into Bangladesh. The new dam's ability to generate
hydropower could be three times that of central China's Three Gorges Dam, which has the largest installed
hydropower capacity in the world.

As a lower riparian country, India has the right to be worried because this can have far-reaching impacts on the
lives and livelihoods of its people. According to the United Nations, more than half the global population will live
in water-stressed or water-scarce countries by 2025, and the vast majority of these people will be in China and
India. While China is home to almost 20% of the world population, it has only about 7% of water resources. India
is home to about 17% of the world's population but has less than 4% of water resources. It is dependent on
foreign originating rivers for about a third of its surface water. As both grow economically, water consumption
will increase and it will automatically place ever-greater pressure on water supplies. Macro challenges such as
the climate crisis and pollution will further strain freshwater resources.

According to Yan Zhiyong, chairman of the Power Construction Corp of China, the 60 million kWh hydropower
exploitation at the downstream of the Yarlung Zangbo River could provide 300 billion kWh of clean, renewable
and zero-carbon electricity annually. The project will play a signi cant role in realizing China's goal of reaching a
carbon emissions peak before 2030 and carbon neutrality in 2060.

"It is a project for national security, including water resources and domestic security," the Power China chairman
said, noting that the project will also smooth cooperation with South Asia.

India must explore all diplomatic and economic options to bring China to the discussion table. But whatever it
does, timing will be the key to make the right impact, says 'Water Wars: The Brahmaputra River and Sino-Indian
Relations', a paper by the US Naval War College. It has to be stressed that hydropower projects on cross-border
rivers cannot be developed without communication and cooperation between upstream and downstream
countries. The earlier India pushes back against Chinese dam building, the more options will be available to it in
the future.

Q.5 [30465398]
Out of the following options, which one is factually correct in the light of the passage?

a Pollution of rivers can affect their navigability.

b Pollution can affect rivers.

c Pollution can affect the quality of sea water.

d The Brahmaputra river is the lifeline of Northeast India.

https://www.aspiration.ai/LAW/sis/Solution.jsp?qsetId=Lw5e7noFooA=&qsetName=LST Mock 20 2021 (CLAT) 9/208


12/18/2020 Mock Analysis

Solution:
 Answer key/Solution
Correct Answer : b
Your Answer : c
Refer to the last sentence of the second paragraph for the answer. The author
mentions the challenges of climate change and pollution as factors affecting freshwater resources. So, option
(b) is correct. Option (a) is wrong because navigability is not mentioned in the passage. Option (c) is wrong
because freshwater resources are discussed and not sea water. Option (d) is out of scope.

FeedBack Bookmark

https://www.aspiration.ai/LAW/sis/Solution.jsp?qsetId=Lw5e7noFooA=&qsetName=LST Mock 20 2021 (CLAT) 10/208


12/18/2020 Mock Analysis

Directions for questions 1 to 30: Each set of questions in this section is based on a single passage. Please
answer each question on the basis of what is stated or implied in the corresponding passage. In some
instances, more than one option may be the answer to the question; in such a case, please choose the option
that most accurately and comprehensively answers the question.

Passage – 2

Racism has raised its ugly head in full public view once again. It was revolting to see an adult gasping for breath,
writhing in pain as the knee of the white policeman crushed his neck, and, within minutes, dying - the umpteenth
time that a black life has been barbarically taken away by police brutality in America. Despite the civil war over
slavery, and the civil rights movement for dignity and equality, systemic discrimination and violence against
blacks persists. Racism continues unabated.

My sole focus here is coming to grips with what racism is. In a nutshell, and with slight, only slight
oversimpli cation, it is this: one can tell everything important about a person, his group, its past and future, by
noting the colour of his skin.

Of course, noticing the physical characteristics of a person, say the colour of her skin, is not itself racist. Good
writers are expected to provide a vivid description of a character's physical features, including skin-colour. This
need not imply the idea of race, leave alone racism. For instance, Indian epics describe Krishna as having shyam
varna, being the dark-skinned one. This description has no evaluative connotation. Being conscious of the
colour of a person, your own or that of the other may be pretty innocent.

However, when speci c bodily features (colour, shape of nose, eye, lips) are permanently clumped together and
human beings are classi ed in terms of these distinct biological clusters, and if, further, it is believed that these
shared features are inter-generationally transmitted, then we possess the idea of race, i.e. a group with a
common biological descent. Every single human being is not only seen then to be assigned to separate
biologically-determined groups but also as born with traits directly inherited from biological ancestors. Each
race is then believed to be fundamentally, permanently different from others - differences that are innate and
indelible, for one can neither cease to have what one has inherited nor acquire characteristics which one does
not already have.

The idea of race is deeply problematic. Despite many attempts, particularly in the 1930s to demonstrate its
scienti c basis, race or racial classi cations have virtually no scienti c foundation. If anything, the only
conclusion from available evidence is that the whole of humanity has the same lineage, that there are no races
within humans but only one single human 'race'. Yet, while scienti cally speaking, race is a ction, a large
number of people believe in the existence of races. Race is very much a cultural and social reality.

Q.6 [30465398]
The author's main purpose of writing the given passage is:

a To explain racism.

b To show that racism existed only in the past.

c To measure the degree of racism in America.

d To show the difference between racism and race.

https://www.aspiration.ai/LAW/sis/Solution.jsp?qsetId=Lw5e7noFooA=&qsetName=LST Mock 20 2021 (CLAT) 11/208


12/18/2020 Mock Analysis

Solution:
 Answer key/Solution
Correct Answer : d
Your Answer : a
The passage talks about racism and race in a simpli ed manner. Hence, option
(d) is the answer. Option (a) is only partially correct. Option (b) talks only about the existence of racism in the
past. This is only partially correct. Option (c) focuses on racism only and leaves out race.
FeedBack Bookmark

https://www.aspiration.ai/LAW/sis/Solution.jsp?qsetId=Lw5e7noFooA=&qsetName=LST Mock 20 2021 (CLAT) 12/208


12/18/2020 Mock Analysis

Directions for questions 1 to 30: Each set of questions in this section is based on a single passage. Please
answer each question on the basis of what is stated or implied in the corresponding passage. In some
instances, more than one option may be the answer to the question; in such a case, please choose the option
that most accurately and comprehensively answers the question.

Passage – 2

Racism has raised its ugly head in full public view once again. It was revolting to see an adult gasping for breath,
writhing in pain as the knee of the white policeman crushed his neck, and, within minutes, dying - the umpteenth
time that a black life has been barbarically taken away by police brutality in America. Despite the civil war over
slavery, and the civil rights movement for dignity and equality, systemic discrimination and violence against
blacks persists. Racism continues unabated.

My sole focus here is coming to grips with what racism is. In a nutshell, and with slight, only slight
oversimpli cation, it is this: one can tell everything important about a person, his group, its past and future, by
noting the colour of his skin.

Of course, noticing the physical characteristics of a person, say the colour of her skin, is not itself racist. Good
writers are expected to provide a vivid description of a character's physical features, including skin-colour. This
need not imply the idea of race, leave alone racism. For instance, Indian epics describe Krishna as having shyam
varna, being the dark-skinned one. This description has no evaluative connotation. Being conscious of the
colour of a person, your own or that of the other may be pretty innocent.

However, when speci c bodily features (colour, shape of nose, eye, lips) are permanently clumped together and
human beings are classi ed in terms of these distinct biological clusters, and if, further, it is believed that these
shared features are inter-generationally transmitted, then we possess the idea of race, i.e. a group with a
common biological descent. Every single human being is not only seen then to be assigned to separate
biologically-determined groups but also as born with traits directly inherited from biological ancestors. Each
race is then believed to be fundamentally, permanently different from others - differences that are innate and
indelible, for one can neither cease to have what one has inherited nor acquire characteristics which one does
not already have.

The idea of race is deeply problematic. Despite many attempts, particularly in the 1930s to demonstrate its
scienti c basis, race or racial classi cations have virtually no scienti c foundation. If anything, the only
conclusion from available evidence is that the whole of humanity has the same lineage, that there are no races
within humans but only one single human 'race'. Yet, while scienti cally speaking, race is a ction, a large
number of people believe in the existence of races. Race is very much a cultural and social reality.

Q.7 [30465398]
According to the given passage all of the following are true EXCEPT:

a There is a systemic discrimination against the blacks in America.

b Very often people are discriminated against because of their skin colour.

c It has been proved scienti cally that race exists.

d Race and racial classi cations have virtually no scienti c foundation.

https://www.aspiration.ai/LAW/sis/Solution.jsp?qsetId=Lw5e7noFooA=&qsetName=LST Mock 20 2021 (CLAT) 13/208


12/18/2020 Mock Analysis

Solution:
 Answer key/Solution
Correct Answer : c
Your Answer : c
Options (a), (b) and (d) are true according to the given passage, however, option
(c) contradicts what is mentioned in the passage. In the passage, it is said that scienti cally speaking, race
does not exist. Refer to "…scienti cally speaking, race is a ction."

FeedBack Bookmark

https://www.aspiration.ai/LAW/sis/Solution.jsp?qsetId=Lw5e7noFooA=&qsetName=LST Mock 20 2021 (CLAT) 14/208


12/18/2020 Mock Analysis

Directions for questions 1 to 30: Each set of questions in this section is based on a single passage. Please
answer each question on the basis of what is stated or implied in the corresponding passage. In some
instances, more than one option may be the answer to the question; in such a case, please choose the option
that most accurately and comprehensively answers the question.

Passage – 2

Racism has raised its ugly head in full public view once again. It was revolting to see an adult gasping for breath,
writhing in pain as the knee of the white policeman crushed his neck, and, within minutes, dying - the umpteenth
time that a black life has been barbarically taken away by police brutality in America. Despite the civil war over
slavery, and the civil rights movement for dignity and equality, systemic discrimination and violence against
blacks persists. Racism continues unabated.

My sole focus here is coming to grips with what racism is. In a nutshell, and with slight, only slight
oversimpli cation, it is this: one can tell everything important about a person, his group, its past and future, by
noting the colour of his skin.

Of course, noticing the physical characteristics of a person, say the colour of her skin, is not itself racist. Good
writers are expected to provide a vivid description of a character's physical features, including skin-colour. This
need not imply the idea of race, leave alone racism. For instance, Indian epics describe Krishna as having shyam
varna, being the dark-skinned one. This description has no evaluative connotation. Being conscious of the
colour of a person, your own or that of the other may be pretty innocent.

However, when speci c bodily features (colour, shape of nose, eye, lips) are permanently clumped together and
human beings are classi ed in terms of these distinct biological clusters, and if, further, it is believed that these
shared features are inter-generationally transmitted, then we possess the idea of race, i.e. a group with a
common biological descent. Every single human being is not only seen then to be assigned to separate
biologically-determined groups but also as born with traits directly inherited from biological ancestors. Each
race is then believed to be fundamentally, permanently different from others - differences that are innate and
indelible, for one can neither cease to have what one has inherited nor acquire characteristics which one does
not already have.

The idea of race is deeply problematic. Despite many attempts, particularly in the 1930s to demonstrate its
scienti c basis, race or racial classi cations have virtually no scienti c foundation. If anything, the only
conclusion from available evidence is that the whole of humanity has the same lineage, that there are no races
within humans but only one single human 'race'. Yet, while scienti cally speaking, race is a ction, a large
number of people believe in the existence of races. Race is very much a cultural and social reality.

Q.8 [30465398]
A writer describing a person's skin colour:

a is an honest necessity.

b is a case of racism.

c is an unnecessary endeavour.

d can be a result of his evil intention.

https://www.aspiration.ai/LAW/sis/Solution.jsp?qsetId=Lw5e7noFooA=&qsetName=LST Mock 20 2021 (CLAT) 15/208


12/18/2020 Mock Analysis

Solution:
 Answer key/Solution
Correct Answer : a
Your Answer : a
Option (b) is not true according to the passage. Option (c) is out of scope and
option (d) is extreme. Option (a) is the answer. Refer to "Of course, noticing the physical characteristics of a
person, say the colour of her skin, is not itself racist. Good writers are expected to provide a vivid description
of a character's physical features, including skin-colour."

FeedBack Bookmark

https://www.aspiration.ai/LAW/sis/Solution.jsp?qsetId=Lw5e7noFooA=&qsetName=LST Mock 20 2021 (CLAT) 16/208


12/18/2020 Mock Analysis

Directions for questions 1 to 30: Each set of questions in this section is based on a single passage. Please
answer each question on the basis of what is stated or implied in the corresponding passage. In some
instances, more than one option may be the answer to the question; in such a case, please choose the option
that most accurately and comprehensively answers the question.

Passage – 2

Racism has raised its ugly head in full public view once again. It was revolting to see an adult gasping for breath,
writhing in pain as the knee of the white policeman crushed his neck, and, within minutes, dying - the umpteenth
time that a black life has been barbarically taken away by police brutality in America. Despite the civil war over
slavery, and the civil rights movement for dignity and equality, systemic discrimination and violence against
blacks persists. Racism continues unabated.

My sole focus here is coming to grips with what racism is. In a nutshell, and with slight, only slight
oversimpli cation, it is this: one can tell everything important about a person, his group, its past and future, by
noting the colour of his skin.

Of course, noticing the physical characteristics of a person, say the colour of her skin, is not itself racist. Good
writers are expected to provide a vivid description of a character's physical features, including skin-colour. This
need not imply the idea of race, leave alone racism. For instance, Indian epics describe Krishna as having shyam
varna, being the dark-skinned one. This description has no evaluative connotation. Being conscious of the
colour of a person, your own or that of the other may be pretty innocent.

However, when speci c bodily features (colour, shape of nose, eye, lips) are permanently clumped together and
human beings are classi ed in terms of these distinct biological clusters, and if, further, it is believed that these
shared features are inter-generationally transmitted, then we possess the idea of race, i.e. a group with a
common biological descent. Every single human being is not only seen then to be assigned to separate
biologically-determined groups but also as born with traits directly inherited from biological ancestors. Each
race is then believed to be fundamentally, permanently different from others - differences that are innate and
indelible, for one can neither cease to have what one has inherited nor acquire characteristics which one does
not already have.

The idea of race is deeply problematic. Despite many attempts, particularly in the 1930s to demonstrate its
scienti c basis, race or racial classi cations have virtually no scienti c foundation. If anything, the only
conclusion from available evidence is that the whole of humanity has the same lineage, that there are no races
within humans but only one single human 'race'. Yet, while scienti cally speaking, race is a ction, a large
number of people believe in the existence of races. Race is very much a cultural and social reality.

Q.9 [30465398]
What is the antonym of indelible?

a Enduring

b Erasable

c Persistent

d Perennial

https://www.aspiration.ai/LAW/sis/Solution.jsp?qsetId=Lw5e7noFooA=&qsetName=LST Mock 20 2021 (CLAT) 17/208


12/18/2020 Mock Analysis

Solution:
 Answer key/Solution
Correct Answer : b
Your Answer : b
'Indelible' means permanent or something that cannot be forgotten or erased. So,
option (b), 'erasable' is the antonym of 'indelible'. Enduring means lasting. Persistent means determined and
perennial means persistent or perpetual. So, options (a), (c) and (d) are synonymous.

FeedBack Bookmark

https://www.aspiration.ai/LAW/sis/Solution.jsp?qsetId=Lw5e7noFooA=&qsetName=LST Mock 20 2021 (CLAT) 18/208


12/18/2020 Mock Analysis

Directions for questions 1 to 30: Each set of questions in this section is based on a single passage. Please
answer each question on the basis of what is stated or implied in the corresponding passage. In some
instances, more than one option may be the answer to the question; in such a case, please choose the option
that most accurately and comprehensively answers the question.

Passage – 2

Racism has raised its ugly head in full public view once again. It was revolting to see an adult gasping for breath,
writhing in pain as the knee of the white policeman crushed his neck, and, within minutes, dying - the umpteenth
time that a black life has been barbarically taken away by police brutality in America. Despite the civil war over
slavery, and the civil rights movement for dignity and equality, systemic discrimination and violence against
blacks persists. Racism continues unabated.

My sole focus here is coming to grips with what racism is. In a nutshell, and with slight, only slight
oversimpli cation, it is this: one can tell everything important about a person, his group, its past and future, by
noting the colour of his skin.

Of course, noticing the physical characteristics of a person, say the colour of her skin, is not itself racist. Good
writers are expected to provide a vivid description of a character's physical features, including skin-colour. This
need not imply the idea of race, leave alone racism. For instance, Indian epics describe Krishna as having shyam
varna, being the dark-skinned one. This description has no evaluative connotation. Being conscious of the
colour of a person, your own or that of the other may be pretty innocent.

However, when speci c bodily features (colour, shape of nose, eye, lips) are permanently clumped together and
human beings are classi ed in terms of these distinct biological clusters, and if, further, it is believed that these
shared features are inter-generationally transmitted, then we possess the idea of race, i.e. a group with a
common biological descent. Every single human being is not only seen then to be assigned to separate
biologically-determined groups but also as born with traits directly inherited from biological ancestors. Each
race is then believed to be fundamentally, permanently different from others - differences that are innate and
indelible, for one can neither cease to have what one has inherited nor acquire characteristics which one does
not already have.

The idea of race is deeply problematic. Despite many attempts, particularly in the 1930s to demonstrate its
scienti c basis, race or racial classi cations have virtually no scienti c foundation. If anything, the only
conclusion from available evidence is that the whole of humanity has the same lineage, that there are no races
within humans but only one single human 'race'. Yet, while scienti cally speaking, race is a ction, a large
number of people believe in the existence of races. Race is very much a cultural and social reality.

Q.10 [30465398]
The fact that the whole of humanity has the same lineage can be concluded from:

a the available evidence.

b the result of human interactions.

c ctional books.

d what humans have inherited from their biological ancestors.

https://www.aspiration.ai/LAW/sis/Solution.jsp?qsetId=Lw5e7noFooA=&qsetName=LST Mock 20 2021 (CLAT) 19/208


12/18/2020 Mock Analysis

Solution:
 Answer key/Solution
Correct Answer : a
Your Answer : a
The answer to this question can be found in the sentence, "If anything, the only
conclusion from available evidence is that the whole of humanity has the same lineage, that there are no races
within humans but only one single human 'race'." Option (b) cannot be ascertained. Option (c) is not true.
Option (d) is not true either.
FeedBack Bookmark

https://www.aspiration.ai/LAW/sis/Solution.jsp?qsetId=Lw5e7noFooA=&qsetName=LST Mock 20 2021 (CLAT) 20/208


12/18/2020 Mock Analysis

Directions for questions 1 to 30: Each set of questions in this section is based on a single passage. Please
answer each question on the basis of what is stated or implied in the corresponding passage. In some
instances, more than one option may be the answer to the question; in such a case, please choose the option
that most accurately and comprehensively answers the question.

Passage – 3

India is in an economic recession for the rst time in its independent history. Since April, large swathes of the
nation were completely shut for three months and only partially open there after. A recession and a pall of
gloom [1] the economy were inevitable. The only surprise would have been if India's economy had not suffered a
deleterious impact. But, for a tiny sliver of India's elite, it is not gloom but boom times.

In the seven-month period between April and October, when businesses were shut and jobs were lost, 274
million rural Indians earned their livelihoods primarily by labouring an entire day to earn a paltry Rs. 200 under
the Mahatma Gandhi National Rural Employment Guarantee Scheme (MGNREGS). Hundred million more people
asked for work under MGNREGS this year vis-à-vis last year, re ecting the economic impact of the lockdown.

In exactly this sombre period, the top 50 companies in India's stock market increased their value by $200 billion
combined. This is seven times more than the increase in the value of these same 50 companies in the same
period last year, when the economy was functioning normally. It de es conventional wisdom that when
businesses are shut, and sales are faltering, that the market value of companies should increase so sharply.

The market value of most of these companies has risen irrationally because of exuberant stock market activity
with utter disregard to the state of the underlying economy or the performance of the companies. Foreign
investors ush with money from central banks in their home countries have poured their excess cash into India's
stock markets.

Just when millions of rural Indians were toiling in the sun for Rs. 200 rupees, these investors spent $250bn
every day to speculate on the prices of shares, currency, derivatives and other exotic nancial instruments in
India's stock markets.

These 50 companies employ nearly two million workers. A vast majority of these workers did not gain a single
rupee from the new wealth added to their companies. Nearly all the gains accrued only to the owners of these
companies, their shareholders and people employed in the nancial markets sector. Worse, such increase in
market wealth did not help to create new jobs in these companies. Instead, some reduced their workforce while
many reduced the wages of their employees. There was no tangible bene t to the economy or society from this
excessive speculative stock market activity except enriching a select few and widening the economic and social
inequality.

Excesses of nancialisation and nancial markets are a real threat to social stability and harmony, not just in
India but in many other countries such as the United States and United Kingdom. In the garb of market
e ciency and price discovery, nancial markets have turned into a den for gambling and a channel for easy
wealth for a select few. But their risks are borne by all and protected by governments using taxes and savings of
ordinary citizens. The social costs of excessive nancialisation of economies now far exceed their perceived
trickle down economic bene ts.

Q.11 [30465398]
Out of the following options, which preposition can replace [1] in the passage?

https://www.aspiration.ai/LAW/sis/Solution.jsp?qsetId=Lw5e7noFooA=&qsetName=LST Mock 20 2021 (CLAT) 21/208


12/18/2020 Mock Analysis

a beside

b over

c behind

d below

Solution:
 Answer key/Solution
Correct Answer : b
Your Answer : b
The correct preposition is 'over' that can replace [1] in the sentence.

FeedBack Bookmark

https://www.aspiration.ai/LAW/sis/Solution.jsp?qsetId=Lw5e7noFooA=&qsetName=LST Mock 20 2021 (CLAT) 22/208


12/18/2020 Mock Analysis

Directions for questions 1 to 30: Each set of questions in this section is based on a single passage. Please
answer each question on the basis of what is stated or implied in the corresponding passage. In some
instances, more than one option may be the answer to the question; in such a case, please choose the option
that most accurately and comprehensively answers the question.

Passage – 3

India is in an economic recession for the rst time in its independent history. Since April, large swathes of the
nation were completely shut for three months and only partially open there after. A recession and a pall of
gloom [1] the economy were inevitable. The only surprise would have been if India's economy had not suffered a
deleterious impact. But, for a tiny sliver of India's elite, it is not gloom but boom times.

In the seven-month period between April and October, when businesses were shut and jobs were lost, 274
million rural Indians earned their livelihoods primarily by labouring an entire day to earn a paltry Rs. 200 under
the Mahatma Gandhi National Rural Employment Guarantee Scheme (MGNREGS). Hundred million more people
asked for work under MGNREGS this year vis-à-vis last year, re ecting the economic impact of the lockdown.

In exactly this sombre period, the top 50 companies in India's stock market increased their value by $200 billion
combined. This is seven times more than the increase in the value of these same 50 companies in the same
period last year, when the economy was functioning normally. It de es conventional wisdom that when
businesses are shut, and sales are faltering, that the market value of companies should increase so sharply.

The market value of most of these companies has risen irrationally because of exuberant stock market activity
with utter disregard to the state of the underlying economy or the performance of the companies. Foreign
investors ush with money from central banks in their home countries have poured their excess cash into India's
stock markets.

Just when millions of rural Indians were toiling in the sun for Rs. 200 rupees, these investors spent $250bn
every day to speculate on the prices of shares, currency, derivatives and other exotic nancial instruments in
India's stock markets.

These 50 companies employ nearly two million workers. A vast majority of these workers did not gain a single
rupee from the new wealth added to their companies. Nearly all the gains accrued only to the owners of these
companies, their shareholders and people employed in the nancial markets sector. Worse, such increase in
market wealth did not help to create new jobs in these companies. Instead, some reduced their workforce while
many reduced the wages of their employees. There was no tangible bene t to the economy or society from this
excessive speculative stock market activity except enriching a select few and widening the economic and social
inequality.

Excesses of nancialisation and nancial markets are a real threat to social stability and harmony, not just in
India but in many other countries such as the United States and United Kingdom. In the garb of market
e ciency and price discovery, nancial markets have turned into a den for gambling and a channel for easy
wealth for a select few. But their risks are borne by all and protected by governments using taxes and savings of
ordinary citizens. The social costs of excessive nancialisation of economies now far exceed their perceived
trickle down economic bene ts.

Q.12 [30465398]
Out of the following options, which one is factually correct in the light of the passage?

https://www.aspiration.ai/LAW/sis/Solution.jsp?qsetId=Lw5e7noFooA=&qsetName=LST Mock 20 2021 (CLAT) 23/208


12/18/2020 Mock Analysis

a Every Indian company resorted to layoffs during the recession.

b The transport sector was the worst affected during the lockdown.

c Many Indians died of starvation during the economic recession.

d Some companies may not pass on part of their pro ts to the employees.

Solution:
 Answer key/Solution
Correct Answer : d
Your Answer : a
Refer to the second sentence of the second last paragraph where the author
makes such an observation. Option (a) is wrong because in the passage, it is mentioned that some and not
every Indian company reduced their workforce. The author has not mentioned anything about the transport
sector and so, option (b) is wrong. Option (c) is out of scope.

FeedBack Bookmark

https://www.aspiration.ai/LAW/sis/Solution.jsp?qsetId=Lw5e7noFooA=&qsetName=LST Mock 20 2021 (CLAT) 24/208


12/18/2020 Mock Analysis

Directions for questions 1 to 30: Each set of questions in this section is based on a single passage. Please
answer each question on the basis of what is stated or implied in the corresponding passage. In some
instances, more than one option may be the answer to the question; in such a case, please choose the option
that most accurately and comprehensively answers the question.

Passage – 3

India is in an economic recession for the rst time in its independent history. Since April, large swathes of the
nation were completely shut for three months and only partially open there after. A recession and a pall of
gloom [1] the economy were inevitable. The only surprise would have been if India's economy had not suffered a
deleterious impact. But, for a tiny sliver of India's elite, it is not gloom but boom times.

In the seven-month period between April and October, when businesses were shut and jobs were lost, 274
million rural Indians earned their livelihoods primarily by labouring an entire day to earn a paltry Rs. 200 under
the Mahatma Gandhi National Rural Employment Guarantee Scheme (MGNREGS). Hundred million more people
asked for work under MGNREGS this year vis-à-vis last year, re ecting the economic impact of the lockdown.

In exactly this sombre period, the top 50 companies in India's stock market increased their value by $200 billion
combined. This is seven times more than the increase in the value of these same 50 companies in the same
period last year, when the economy was functioning normally. It de es conventional wisdom that when
businesses are shut, and sales are faltering, that the market value of companies should increase so sharply.

The market value of most of these companies has risen irrationally because of exuberant stock market activity
with utter disregard to the state of the underlying economy or the performance of the companies. Foreign
investors ush with money from central banks in their home countries have poured their excess cash into India's
stock markets.

Just when millions of rural Indians were toiling in the sun for Rs. 200 rupees, these investors spent $250bn
every day to speculate on the prices of shares, currency, derivatives and other exotic nancial instruments in
India's stock markets.

These 50 companies employ nearly two million workers. A vast majority of these workers did not gain a single
rupee from the new wealth added to their companies. Nearly all the gains accrued only to the owners of these
companies, their shareholders and people employed in the nancial markets sector. Worse, such increase in
market wealth did not help to create new jobs in these companies. Instead, some reduced their workforce while
many reduced the wages of their employees. There was no tangible bene t to the economy or society from this
excessive speculative stock market activity except enriching a select few and widening the economic and social
inequality.

Excesses of nancialisation and nancial markets are a real threat to social stability and harmony, not just in
India but in many other countries such as the United States and United Kingdom. In the garb of market
e ciency and price discovery, nancial markets have turned into a den for gambling and a channel for easy
wealth for a select few. But their risks are borne by all and protected by governments using taxes and savings of
ordinary citizens. The social costs of excessive nancialisation of economies now far exceed their perceived
trickle down economic bene ts.

Q.13 [30465398]
The author of the passage is most likely to agree with which of the following statements:

https://www.aspiration.ai/LAW/sis/Solution.jsp?qsetId=Lw5e7noFooA=&qsetName=LST Mock 20 2021 (CLAT) 25/208


12/18/2020 Mock Analysis

a When there is an economic recession, the market value of companies should decrease.

b When there is an economic recession, the market value of companies should increase.

c When there is an economic recession, the market value of companies should remain stable.

d When there is an economic recession, the market value of companies should be unpredictable.

Solution:
 Answer key/Solution
Correct Answer : a
Your Answer : a
Refer to the sentence, "It de es conventional wisdom that when businesses are
shut, and sales are faltering, that the market value of companies should increase so sharply." Therefore, option
(a) is correct. The remaining options can be nulli ed in the light of the sentence quoted above.

FeedBack Bookmark

https://www.aspiration.ai/LAW/sis/Solution.jsp?qsetId=Lw5e7noFooA=&qsetName=LST Mock 20 2021 (CLAT) 26/208


12/18/2020 Mock Analysis

Directions for questions 1 to 30: Each set of questions in this section is based on a single passage. Please
answer each question on the basis of what is stated or implied in the corresponding passage. In some
instances, more than one option may be the answer to the question; in such a case, please choose the option
that most accurately and comprehensively answers the question.

Passage – 3

India is in an economic recession for the rst time in its independent history. Since April, large swathes of the
nation were completely shut for three months and only partially open there after. A recession and a pall of
gloom [1] the economy were inevitable. The only surprise would have been if India's economy had not suffered a
deleterious impact. But, for a tiny sliver of India's elite, it is not gloom but boom times.

In the seven-month period between April and October, when businesses were shut and jobs were lost, 274
million rural Indians earned their livelihoods primarily by labouring an entire day to earn a paltry Rs. 200 under
the Mahatma Gandhi National Rural Employment Guarantee Scheme (MGNREGS). Hundred million more people
asked for work under MGNREGS this year vis-à-vis last year, re ecting the economic impact of the lockdown.

In exactly this sombre period, the top 50 companies in India's stock market increased their value by $200 billion
combined. This is seven times more than the increase in the value of these same 50 companies in the same
period last year, when the economy was functioning normally. It de es conventional wisdom that when
businesses are shut, and sales are faltering, that the market value of companies should increase so sharply.

The market value of most of these companies has risen irrationally because of exuberant stock market activity
with utter disregard to the state of the underlying economy or the performance of the companies. Foreign
investors ush with money from central banks in their home countries have poured their excess cash into India's
stock markets.

Just when millions of rural Indians were toiling in the sun for Rs. 200 rupees, these investors spent $250bn
every day to speculate on the prices of shares, currency, derivatives and other exotic nancial instruments in
India's stock markets.

These 50 companies employ nearly two million workers. A vast majority of these workers did not gain a single
rupee from the new wealth added to their companies. Nearly all the gains accrued only to the owners of these
companies, their shareholders and people employed in the nancial markets sector. Worse, such increase in
market wealth did not help to create new jobs in these companies. Instead, some reduced their workforce while
many reduced the wages of their employees. There was no tangible bene t to the economy or society from this
excessive speculative stock market activity except enriching a select few and widening the economic and social
inequality.

Excesses of nancialisation and nancial markets are a real threat to social stability and harmony, not just in
India but in many other countries such as the United States and United Kingdom. In the garb of market
e ciency and price discovery, nancial markets have turned into a den for gambling and a channel for easy
wealth for a select few. But their risks are borne by all and protected by governments using taxes and savings of
ordinary citizens. The social costs of excessive nancialisation of economies now far exceed their perceived
trickle down economic bene ts.

Q.14 [30465398]
According to the passage, what was the source of funds for Indian companies during the lockdown?

https://www.aspiration.ai/LAW/sis/Solution.jsp?qsetId=Lw5e7noFooA=&qsetName=LST Mock 20 2021 (CLAT) 27/208


12/18/2020 Mock Analysis

a The International Monetary Fund

b Overseas lenders

c American bankers

d Crowd funding

Solution:
 Answer key/Solution
Correct Answer : b
Your Answer : b
Refer to the fourth paragraph where the author mentions about foreign investors
who have invested money. The other options can be negated in the light of the passage.

FeedBack Bookmark

https://www.aspiration.ai/LAW/sis/Solution.jsp?qsetId=Lw5e7noFooA=&qsetName=LST Mock 20 2021 (CLAT) 28/208


12/18/2020 Mock Analysis

Directions for questions 1 to 30: Each set of questions in this section is based on a single passage. Please
answer each question on the basis of what is stated or implied in the corresponding passage. In some
instances, more than one option may be the answer to the question; in such a case, please choose the option
that most accurately and comprehensively answers the question.

Passage – 3

India is in an economic recession for the rst time in its independent history. Since April, large swathes of the
nation were completely shut for three months and only partially open there after. A recession and a pall of
gloom [1] the economy were inevitable. The only surprise would have been if India's economy had not suffered a
deleterious impact. But, for a tiny sliver of India's elite, it is not gloom but boom times.

In the seven-month period between April and October, when businesses were shut and jobs were lost, 274
million rural Indians earned their livelihoods primarily by labouring an entire day to earn a paltry Rs. 200 under
the Mahatma Gandhi National Rural Employment Guarantee Scheme (MGNREGS). Hundred million more people
asked for work under MGNREGS this year vis-à-vis last year, re ecting the economic impact of the lockdown.

In exactly this sombre period, the top 50 companies in India's stock market increased their value by $200 billion
combined. This is seven times more than the increase in the value of these same 50 companies in the same
period last year, when the economy was functioning normally. It de es conventional wisdom that when
businesses are shut, and sales are faltering, that the market value of companies should increase so sharply.

The market value of most of these companies has risen irrationally because of exuberant stock market activity
with utter disregard to the state of the underlying economy or the performance of the companies. Foreign
investors ush with money from central banks in their home countries have poured their excess cash into India's
stock markets.

Just when millions of rural Indians were toiling in the sun for Rs. 200 rupees, these investors spent $250bn
every day to speculate on the prices of shares, currency, derivatives and other exotic nancial instruments in
India's stock markets.

These 50 companies employ nearly two million workers. A vast majority of these workers did not gain a single
rupee from the new wealth added to their companies. Nearly all the gains accrued only to the owners of these
companies, their shareholders and people employed in the nancial markets sector. Worse, such increase in
market wealth did not help to create new jobs in these companies. Instead, some reduced their workforce while
many reduced the wages of their employees. There was no tangible bene t to the economy or society from this
excessive speculative stock market activity except enriching a select few and widening the economic and social
inequality.

Excesses of nancialisation and nancial markets are a real threat to social stability and harmony, not just in
India but in many other countries such as the United States and United Kingdom. In the garb of market
e ciency and price discovery, nancial markets have turned into a den for gambling and a channel for easy
wealth for a select few. But their risks are borne by all and protected by governments using taxes and savings of
ordinary citizens. The social costs of excessive nancialisation of economies now far exceed their perceived
trickle down economic bene ts.

Q.15 [30465398]
Out of the following options, which one is an appropriate title of the given passage?

https://www.aspiration.ai/LAW/sis/Solution.jsp?qsetId=Lw5e7noFooA=&qsetName=LST Mock 20 2021 (CLAT) 29/208


12/18/2020 Mock Analysis

a Foreign investors and their role in the Indian stock market

b The MGNREGS: A social welfare scheme

c The perils of nancialisation and nancial markets

d The bene ts of nancial markets

Solution:
 Answer key/Solution
Correct Answer : c
Your Answer : c
Option (d) is wrong because the author, in the passage, mentions the
disadvantages of a nancial market. Options (a) and (b) are narrow in scope. The passage is a commentary on
nancialisation and the perils of nancial markets. The author sums up his views in the last paragraph. Hence,
option (c) is the appropriate title of the passage.

FeedBack Bookmark

https://www.aspiration.ai/LAW/sis/Solution.jsp?qsetId=Lw5e7noFooA=&qsetName=LST Mock 20 2021 (CLAT) 30/208


12/18/2020 Mock Analysis

Directions for questions 1 to 30: Each set of questions in this section is based on a single passage. Please
answer each question on the basis of what is stated or implied in the corresponding passage. In some
instances, more than one option may be the answer to the question; in such a case, please choose the option
that most accurately and comprehensively answers the question.

Passage – 4

Six years ago, on November 12, 2014, President Barack Obama and President Xi Jinping struck a deal on
climate change. It made sense then for the largest historical carbon polluter and the largest current polluter to
agree to act together. But the deal also disrupted developing country unity. Caught in a "climate labyrinth", India
was encircled by larger economies demanding more climate action from it - a far poorer country but, in
aggregate terms, a major emitter. The ascent of US President-elect Joe Biden raises the prospect of renewed
efforts against the climate crisis. It also offers a strategic opportunity for India - an historic Biden-Modi deal in
2021.

The biggest damage President Donald Trump did to the ght against climate change was to remove trust
between major economies. The Paris Agreement painstakingly created the conditions for building trust
incrementally after more than two decades of fractious climate negotiations. By withdrawing, Trump threatened
to fatally weaken those foundations. He also elevated denial of climate science into high art. And his actions
gave other major polluters an excuse to slow down their efforts.

A Biden administration promises to change this. At home, it intends to spend more than $2 trillion on energy and
infrastructure to achieve net-zero emissions by 2050. This includes the aim to have zero-emission power
generation by 2035, stronger regulation of polluters, and focus on climate risks for companies and their supply
chains.

Abroad, Biden plans to bring the US back into the Paris Agreement and wishes to host a world summit to ask
other countries to ramp up their climate pledges. He will certainly put pressure on India to announce its mid-
century plans. More ominously, with the EU already promising to introduce a carbon border adjustment
mechanism by 2021, the US could also bring in border tariffs (something on which the Democrats and Trump
Republicans might converge), potentially triggering a slew of climate-related trade disputes. Against such
compulsions, India could change the game and shift focus on climate action rather than political rhetoric. It
should reach out to the Biden administration early on and propose a bilateral deal, drawing out four contours -
changing narratives, facilitating investment, developing technology and increasing resilience.

The narrative [1] climate ambition is increasingly crowded with terms such as net-zero, carbon-neutral and
climate positive. Recent announcements by China (2060), the EU (2050) and Japan (2050) for net-zero
emissions are too far into the future, highly unlikely to have any serious accountability and reduce growth
opportunities for developing countries. Lesser the action in the past or near future means tighter the carbon
budget in the future for them.

Q.16 [30465398]
Which of the following would be the most appropriate title of the given passage?

a The major contributors of carbon

b Trump's withdrawal from Paris Agreement

https://www.aspiration.ai/LAW/sis/Solution.jsp?qsetId=Lw5e7noFooA=&qsetName=LST Mock 20 2021 (CLAT) 31/208


12/18/2020 Mock Analysis

c India and US climate change action-plan

d The shift in the climate change plans

Solution:
 Answer key/Solution
Correct Answer : c
Your Answer : d
The passage is primarily about India and US climate change actionplan as
president-elect Biden plans to bring the US back into the Paris Agreement. Refer to "Biden plans to bring the
US back into the Paris Agreement…" and "He will certainly put pressure on India to announce its mid-century
plans." Hence, option (c) is the answer. None of the other options can be the title of the given passage.

FeedBack Bookmark

https://www.aspiration.ai/LAW/sis/Solution.jsp?qsetId=Lw5e7noFooA=&qsetName=LST Mock 20 2021 (CLAT) 32/208


12/18/2020 Mock Analysis

Directions for questions 1 to 30: Each set of questions in this section is based on a single passage. Please
answer each question on the basis of what is stated or implied in the corresponding passage. In some
instances, more than one option may be the answer to the question; in such a case, please choose the option
that most accurately and comprehensively answers the question.

Passage – 4

Six years ago, on November 12, 2014, President Barack Obama and President Xi Jinping struck a deal on
climate change. It made sense then for the largest historical carbon polluter and the largest current polluter to
agree to act together. But the deal also disrupted developing country unity. Caught in a "climate labyrinth", India
was encircled by larger economies demanding more climate action from it - a far poorer country but, in
aggregate terms, a major emitter. The ascent of US President-elect Joe Biden raises the prospect of renewed
efforts against the climate crisis. It also offers a strategic opportunity for India - an historic Biden-Modi deal in
2021.

The biggest damage President Donald Trump did to the ght against climate change was to remove trust
between major economies. The Paris Agreement painstakingly created the conditions for building trust
incrementally after more than two decades of fractious climate negotiations. By withdrawing, Trump threatened
to fatally weaken those foundations. He also elevated denial of climate science into high art. And his actions
gave other major polluters an excuse to slow down their efforts.

A Biden administration promises to change this. At home, it intends to spend more than $2 trillion on energy and
infrastructure to achieve net-zero emissions by 2050. This includes the aim to have zero-emission power
generation by 2035, stronger regulation of polluters, and focus on climate risks for companies and their supply
chains.

Abroad, Biden plans to bring the US back into the Paris Agreement and wishes to host a world summit to ask
other countries to ramp up their climate pledges. He will certainly put pressure on India to announce its mid-
century plans. More ominously, with the EU already promising to introduce a carbon border adjustment
mechanism by 2021, the US could also bring in border tariffs (something on which the Democrats and Trump
Republicans might converge), potentially triggering a slew of climate-related trade disputes. Against such
compulsions, India could change the game and shift focus on climate action rather than political rhetoric. It
should reach out to the Biden administration early on and propose a bilateral deal, drawing out four contours -
changing narratives, facilitating investment, developing technology and increasing resilience.

The narrative [1] climate ambition is increasingly crowded with terms such as net-zero, carbon-neutral and
climate positive. Recent announcements by China (2060), the EU (2050) and Japan (2050) for net-zero
emissions are too far into the future, highly unlikely to have any serious accountability and reduce growth
opportunities for developing countries. Lesser the action in the past or near future means tighter the carbon
budget in the future for them.

Q.17 [30465398]
According to the given passage, all of the following are true EXCEPT:

a Biden administration aims to achieve net-zero emissions by 2035.

b Biden administration aims to have zero-emission power generation by 2035.

https://www.aspiration.ai/LAW/sis/Solution.jsp?qsetId=Lw5e7noFooA=&qsetName=LST Mock 20 2021 (CLAT) 33/208


12/18/2020 Mock Analysis

c Biden administration's aim is to achieve net-zero emissions by 2050.

d Biden administration's wish is to ask other countries to increase their climate pledges.

Solution:
 Answer key/Solution
Correct Answer : a
Your Answer : a
Option (a) is the correct answer. Refer to "A Biden administration promises to
change this. At home, it intends to spend more than $2 trillion on energy and infrastructure to achieve net-
zero emissions by 2050." Other options are mentioned in the given passage.

FeedBack Bookmark

https://www.aspiration.ai/LAW/sis/Solution.jsp?qsetId=Lw5e7noFooA=&qsetName=LST Mock 20 2021 (CLAT) 34/208


12/18/2020 Mock Analysis

Directions for questions 1 to 30: Each set of questions in this section is based on a single passage. Please
answer each question on the basis of what is stated or implied in the corresponding passage. In some
instances, more than one option may be the answer to the question; in such a case, please choose the option
that most accurately and comprehensively answers the question.

Passage – 4

Six years ago, on November 12, 2014, President Barack Obama and President Xi Jinping struck a deal on
climate change. It made sense then for the largest historical carbon polluter and the largest current polluter to
agree to act together. But the deal also disrupted developing country unity. Caught in a "climate labyrinth", India
was encircled by larger economies demanding more climate action from it - a far poorer country but, in
aggregate terms, a major emitter. The ascent of US President-elect Joe Biden raises the prospect of renewed
efforts against the climate crisis. It also offers a strategic opportunity for India - an historic Biden-Modi deal in
2021.

The biggest damage President Donald Trump did to the ght against climate change was to remove trust
between major economies. The Paris Agreement painstakingly created the conditions for building trust
incrementally after more than two decades of fractious climate negotiations. By withdrawing, Trump threatened
to fatally weaken those foundations. He also elevated denial of climate science into high art. And his actions
gave other major polluters an excuse to slow down their efforts.

A Biden administration promises to change this. At home, it intends to spend more than $2 trillion on energy and
infrastructure to achieve net-zero emissions by 2050. This includes the aim to have zero-emission power
generation by 2035, stronger regulation of polluters, and focus on climate risks for companies and their supply
chains.

Abroad, Biden plans to bring the US back into the Paris Agreement and wishes to host a world summit to ask
other countries to ramp up their climate pledges. He will certainly put pressure on India to announce its mid-
century plans. More ominously, with the EU already promising to introduce a carbon border adjustment
mechanism by 2021, the US could also bring in border tariffs (something on which the Democrats and Trump
Republicans might converge), potentially triggering a slew of climate-related trade disputes. Against such
compulsions, India could change the game and shift focus on climate action rather than political rhetoric. It
should reach out to the Biden administration early on and propose a bilateral deal, drawing out four contours -
changing narratives, facilitating investment, developing technology and increasing resilience.

The narrative [1] climate ambition is increasingly crowded with terms such as net-zero, carbon-neutral and
climate positive. Recent announcements by China (2060), the EU (2050) and Japan (2050) for net-zero
emissions are too far into the future, highly unlikely to have any serious accountability and reduce growth
opportunities for developing countries. Lesser the action in the past or near future means tighter the carbon
budget in the future for them.

Q.18 [30465398]
Choose the correct preposition in place of '[1]' from the following.

a with

b around

https://www.aspiration.ai/LAW/sis/Solution.jsp?qsetId=Lw5e7noFooA=&qsetName=LST Mock 20 2021 (CLAT) 35/208


12/18/2020 Mock Analysis

c beside

d in

Solution:
 Answer key/Solution
Correct Answer : b
Your Answer : a
The blank needs a preposition that would mean encircling or about. Hence, option
(b) is the answer. Other options do not t in.

FeedBack Bookmark

https://www.aspiration.ai/LAW/sis/Solution.jsp?qsetId=Lw5e7noFooA=&qsetName=LST Mock 20 2021 (CLAT) 36/208


12/18/2020 Mock Analysis

Directions for questions 1 to 30: Each set of questions in this section is based on a single passage. Please
answer each question on the basis of what is stated or implied in the corresponding passage. In some
instances, more than one option may be the answer to the question; in such a case, please choose the option
that most accurately and comprehensively answers the question.

Passage – 4

Six years ago, on November 12, 2014, President Barack Obama and President Xi Jinping struck a deal on
climate change. It made sense then for the largest historical carbon polluter and the largest current polluter to
agree to act together. But the deal also disrupted developing country unity. Caught in a "climate labyrinth", India
was encircled by larger economies demanding more climate action from it - a far poorer country but, in
aggregate terms, a major emitter. The ascent of US President-elect Joe Biden raises the prospect of renewed
efforts against the climate crisis. It also offers a strategic opportunity for India - an historic Biden-Modi deal in
2021.

The biggest damage President Donald Trump did to the ght against climate change was to remove trust
between major economies. The Paris Agreement painstakingly created the conditions for building trust
incrementally after more than two decades of fractious climate negotiations. By withdrawing, Trump threatened
to fatally weaken those foundations. He also elevated denial of climate science into high art. And his actions
gave other major polluters an excuse to slow down their efforts.

A Biden administration promises to change this. At home, it intends to spend more than $2 trillion on energy and
infrastructure to achieve net-zero emissions by 2050. This includes the aim to have zero-emission power
generation by 2035, stronger regulation of polluters, and focus on climate risks for companies and their supply
chains.

Abroad, Biden plans to bring the US back into the Paris Agreement and wishes to host a world summit to ask
other countries to ramp up their climate pledges. He will certainly put pressure on India to announce its mid-
century plans. More ominously, with the EU already promising to introduce a carbon border adjustment
mechanism by 2021, the US could also bring in border tariffs (something on which the Democrats and Trump
Republicans might converge), potentially triggering a slew of climate-related trade disputes. Against such
compulsions, India could change the game and shift focus on climate action rather than political rhetoric. It
should reach out to the Biden administration early on and propose a bilateral deal, drawing out four contours -
changing narratives, facilitating investment, developing technology and increasing resilience.

The narrative [1] climate ambition is increasingly crowded with terms such as net-zero, carbon-neutral and
climate positive. Recent announcements by China (2060), the EU (2050) and Japan (2050) for net-zero
emissions are too far into the future, highly unlikely to have any serious accountability and reduce growth
opportunities for developing countries. Lesser the action in the past or near future means tighter the carbon
budget in the future for them.

Q.19 [30465398]
It can be inferred from the given passage that:

a India's carbon emission level is very low.

b India totally lacks the capability to tackle the climate change issue.

https://www.aspiration.ai/LAW/sis/Solution.jsp?qsetId=Lw5e7noFooA=&qsetName=LST Mock 20 2021 (CLAT) 37/208


12/18/2020 Mock Analysis

c India does not have an action plan for tackling climate change.

d India's climate change concern is primarily political and it lacks climate action.

Solution:
 Answer key/Solution
Correct Answer : d
Your Answer : a
Option (d) can be inferred from "…India could change the game and shift focus on
climate action rather than political rhetoric." Option (a) cannot be inferred. Option (b) cannot be inferred as it is
not known from the passage whether India totally lacks the capability to to tackle the climate change issue.
Option (c) cannot be inferred as well.

FeedBack Bookmark

https://www.aspiration.ai/LAW/sis/Solution.jsp?qsetId=Lw5e7noFooA=&qsetName=LST Mock 20 2021 (CLAT) 38/208


12/18/2020 Mock Analysis

Directions for questions 1 to 30: Each set of questions in this section is based on a single passage. Please
answer each question on the basis of what is stated or implied in the corresponding passage. In some
instances, more than one option may be the answer to the question; in such a case, please choose the option
that most accurately and comprehensively answers the question.

Passage – 4

Six years ago, on November 12, 2014, President Barack Obama and President Xi Jinping struck a deal on
climate change. It made sense then for the largest historical carbon polluter and the largest current polluter to
agree to act together. But the deal also disrupted developing country unity. Caught in a "climate labyrinth", India
was encircled by larger economies demanding more climate action from it - a far poorer country but, in
aggregate terms, a major emitter. The ascent of US President-elect Joe Biden raises the prospect of renewed
efforts against the climate crisis. It also offers a strategic opportunity for India - an historic Biden-Modi deal in
2021.

The biggest damage President Donald Trump did to the ght against climate change was to remove trust
between major economies. The Paris Agreement painstakingly created the conditions for building trust
incrementally after more than two decades of fractious climate negotiations. By withdrawing, Trump threatened
to fatally weaken those foundations. He also elevated denial of climate science into high art. And his actions
gave other major polluters an excuse to slow down their efforts.

A Biden administration promises to change this. At home, it intends to spend more than $2 trillion on energy and
infrastructure to achieve net-zero emissions by 2050. This includes the aim to have zero-emission power
generation by 2035, stronger regulation of polluters, and focus on climate risks for companies and their supply
chains.

Abroad, Biden plans to bring the US back into the Paris Agreement and wishes to host a world summit to ask
other countries to ramp up their climate pledges. He will certainly put pressure on India to announce its mid-
century plans. More ominously, with the EU already promising to introduce a carbon border adjustment
mechanism by 2021, the US could also bring in border tariffs (something on which the Democrats and Trump
Republicans might converge), potentially triggering a slew of climate-related trade disputes. Against such
compulsions, India could change the game and shift focus on climate action rather than political rhetoric. It
should reach out to the Biden administration early on and propose a bilateral deal, drawing out four contours -
changing narratives, facilitating investment, developing technology and increasing resilience.

The narrative [1] climate ambition is increasingly crowded with terms such as net-zero, carbon-neutral and
climate positive. Recent announcements by China (2060), the EU (2050) and Japan (2050) for net-zero
emissions are too far into the future, highly unlikely to have any serious accountability and reduce growth
opportunities for developing countries. Lesser the action in the past or near future means tighter the carbon
budget in the future for them.

Q.20 [30465398]
The 'bilateral deal' as used in the given passage refers to:

a the deal between China and the US.

b the deal between India and China.

https://www.aspiration.ai/LAW/sis/Solution.jsp?qsetId=Lw5e7noFooA=&qsetName=LST Mock 20 2021 (CLAT) 39/208


12/18/2020 Mock Analysis

c the deal between India and the US

d the deals between US and Japan.

Solution:
 Answer key/Solution
Correct Answer : c
Your Answer : a
The phrase 'bilateral deal' in the given context means deal between India and the
US. Refer to "Against such compulsions, India could change the game and shift focus on climate action rather
than political rhetoric. It should reach out to the Biden administration early on and propose a bilateral deal…"
Hence, option (c) is the answer. Other options are incorrect.
FeedBack Bookmark

https://www.aspiration.ai/LAW/sis/Solution.jsp?qsetId=Lw5e7noFooA=&qsetName=LST Mock 20 2021 (CLAT) 40/208


12/18/2020 Mock Analysis

Directions for questions 1 to 30: Each set of questions in this section is based on a single passage. Please
answer each question on the basis of what is stated or implied in the corresponding passage. In some
instances, more than one option may be the answer to the question; in such a case, please choose the option
that most accurately and comprehensively answers the question.

Passage – 5

It was late in the afternoon, and the light was waning. There was a difference in the look of the tree shadows out
in the yard. Somewhere in the distance cows were lowing and a little bell was tinkling; now and then a farm-
wagon tilted by, and the dust ew; some blue-shirted laborers with shovels over their shoulders plodded past;
little swarms of ies were dancing up and down before the peoples' faces in the soft air. There seemed to be a
gentle stir arising over everything for the mere sake of subsidence-a very premonition of rest and hush and
night.

This soft diurnal commotion was over Louisa Ellis also. She had been peacefully sewing at her sitting-room
window all the afternoon. Now she quilted her needle carefully into her work, which she folded precisely, and laid
in a basket with her thimble and thread and scissors. Louisa Ellis could not remember that ever in her life she
had mislaid one of these little feminine appurtenances, which had become, from long use and constant
association, a very part of her personality.

Louisa tied a green apron round her waist, and got out a at straw hat with a green ribbon. Then she went into
the garden with a little blue crockery bowl, to pick some currants for her tea. After the currants were picked she
sat on the back door-step and stemmed them, collecting the stems carefully in her apron, and afterwards
throwing them into the hen-coop. She looked sharply at the grass beside the step to see if any had fallen there.

Louisa was slow and still in her movements; it took her a long time to prepare her tea; but when ready it was set
forth with as much grace as if she had been a veritable guest to her own self. The little square table stood
exactly in the centre of the kitchen, and was covered with a starched linen cloth whose border pattern of owers
glistened. Louisa had a damask napkin on her tea-tray, where were arranged a cut-glass tumbler full of
teaspoons, a silver cream-pitcher, a china sugar-bowl, and one pink china cup and saucer. Louisa used china
every day-something which none of her neighbors did. They whispered about it among themselves. Their daily
tables were laid with common crockery, their sets of best china stayed in the parlor closet, and Louisa Ellis was
no richer nor better bred than they. Still she would use the china. She had for her supper a glass dish full of
sugared currants, a plate of little cakes, and one of light white biscuits. Also a leaf or two of lettuce, which she
cut up daintily. Louisa was very fond of lettuce, which she raised to perfection in her little garden. She ate quite
heartily, though in a delicate, pecking way; it seemed almost surprising that any considerable bulk of the food
should vanish.

Q.21 [30465398]
Which of the following suggests a synonymous meaning to the word 'subsidence'?

a respite

b settling

c evasion

d lull

https://www.aspiration.ai/LAW/sis/Solution.jsp?qsetId=Lw5e7noFooA=&qsetName=LST Mock 20 2021 (CLAT) 41/208


12/18/2020 Mock Analysis

Solution:
 Answer key/Solution
Correct Answer : b
Your Answer : b
Subsidence in the given context means settling. It is seen in the rst paragraph
that people and everything around them are preparing for settling for the night as the evening approaches.
Hence, option (b) is the answer. Options (a), (c) and (d) fail to give that meaning. Respite means relief or lull
and evasion means dodging or avoidance.
FeedBack Bookmark

https://www.aspiration.ai/LAW/sis/Solution.jsp?qsetId=Lw5e7noFooA=&qsetName=LST Mock 20 2021 (CLAT) 42/208


12/18/2020 Mock Analysis

Directions for questions 1 to 30: Each set of questions in this section is based on a single passage. Please
answer each question on the basis of what is stated or implied in the corresponding passage. In some
instances, more than one option may be the answer to the question; in such a case, please choose the option
that most accurately and comprehensively answers the question.

Passage – 5

It was late in the afternoon, and the light was waning. There was a difference in the look of the tree shadows out
in the yard. Somewhere in the distance cows were lowing and a little bell was tinkling; now and then a farm-
wagon tilted by, and the dust ew; some blue-shirted laborers with shovels over their shoulders plodded past;
little swarms of ies were dancing up and down before the peoples' faces in the soft air. There seemed to be a
gentle stir arising over everything for the mere sake of subsidence-a very premonition of rest and hush and
night.

This soft diurnal commotion was over Louisa Ellis also. She had been peacefully sewing at her sitting-room
window all the afternoon. Now she quilted her needle carefully into her work, which she folded precisely, and laid
in a basket with her thimble and thread and scissors. Louisa Ellis could not remember that ever in her life she
had mislaid one of these little feminine appurtenances, which had become, from long use and constant
association, a very part of her personality.

Louisa tied a green apron round her waist, and got out a at straw hat with a green ribbon. Then she went into
the garden with a little blue crockery bowl, to pick some currants for her tea. After the currants were picked she
sat on the back door-step and stemmed them, collecting the stems carefully in her apron, and afterwards
throwing them into the hen-coop. She looked sharply at the grass beside the step to see if any had fallen there.

Louisa was slow and still in her movements; it took her a long time to prepare her tea; but when ready it was set
forth with as much grace as if she had been a veritable guest to her own self. The little square table stood
exactly in the centre of the kitchen, and was covered with a starched linen cloth whose border pattern of owers
glistened. Louisa had a damask napkin on her tea-tray, where were arranged a cut-glass tumbler full of
teaspoons, a silver cream-pitcher, a china sugar-bowl, and one pink china cup and saucer. Louisa used china
every day-something which none of her neighbors did. They whispered about it among themselves. Their daily
tables were laid with common crockery, their sets of best china stayed in the parlor closet, and Louisa Ellis was
no richer nor better bred than they. Still she would use the china. She had for her supper a glass dish full of
sugared currants, a plate of little cakes, and one of light white biscuits. Also a leaf or two of lettuce, which she
cut up daintily. Louisa was very fond of lettuce, which she raised to perfection in her little garden. She ate quite
heartily, though in a delicate, pecking way; it seemed almost surprising that any considerable bulk of the food
should vanish.

Q.22 [30465398]
Which of the following qualities can be inferred about Louisa Ellis from the passage?

a She is quick and ambitious.

b She is versatile but carefree.

c She is sensitive and motivated.

d She is graceful and composed.

https://www.aspiration.ai/LAW/sis/Solution.jsp?qsetId=Lw5e7noFooA=&qsetName=LST Mock 20 2021 (CLAT) 43/208


12/18/2020 Mock Analysis

Solution:
 Answer key/Solution
Correct Answer : d
Your Answer : d
Louisa Ellis is mentioned as slow in the given story. So option (a) contradicts that.
Louisa Ellis is mentioned as a person who takes care of many things. So, 'carefree' in option (b) contradicts
that quality in her. Whether Louisa Ellis is motivated or not cannot be inferred from the story. So, option (c) is
incorrect. Option (d) can be inferred from "it took her a long time to prepare her tea; but when ready it was set
forth with as much grace as if she had been a veritable guest to her own self."

FeedBack Bookmark

https://www.aspiration.ai/LAW/sis/Solution.jsp?qsetId=Lw5e7noFooA=&qsetName=LST Mock 20 2021 (CLAT) 44/208


12/18/2020 Mock Analysis

Directions for questions 1 to 30: Each set of questions in this section is based on a single passage. Please
answer each question on the basis of what is stated or implied in the corresponding passage. In some
instances, more than one option may be the answer to the question; in such a case, please choose the option
that most accurately and comprehensively answers the question.

Passage – 5

It was late in the afternoon, and the light was waning. There was a difference in the look of the tree shadows out
in the yard. Somewhere in the distance cows were lowing and a little bell was tinkling; now and then a farm-
wagon tilted by, and the dust ew; some blue-shirted laborers with shovels over their shoulders plodded past;
little swarms of ies were dancing up and down before the peoples' faces in the soft air. There seemed to be a
gentle stir arising over everything for the mere sake of subsidence-a very premonition of rest and hush and
night.

This soft diurnal commotion was over Louisa Ellis also. She had been peacefully sewing at her sitting-room
window all the afternoon. Now she quilted her needle carefully into her work, which she folded precisely, and laid
in a basket with her thimble and thread and scissors. Louisa Ellis could not remember that ever in her life she
had mislaid one of these little feminine appurtenances, which had become, from long use and constant
association, a very part of her personality.

Louisa tied a green apron round her waist, and got out a at straw hat with a green ribbon. Then she went into
the garden with a little blue crockery bowl, to pick some currants for her tea. After the currants were picked she
sat on the back door-step and stemmed them, collecting the stems carefully in her apron, and afterwards
throwing them into the hen-coop. She looked sharply at the grass beside the step to see if any had fallen there.

Louisa was slow and still in her movements; it took her a long time to prepare her tea; but when ready it was set
forth with as much grace as if she had been a veritable guest to her own self. The little square table stood
exactly in the centre of the kitchen, and was covered with a starched linen cloth whose border pattern of owers
glistened. Louisa had a damask napkin on her tea-tray, where were arranged a cut-glass tumbler full of
teaspoons, a silver cream-pitcher, a china sugar-bowl, and one pink china cup and saucer. Louisa used china
every day-something which none of her neighbors did. They whispered about it among themselves. Their daily
tables were laid with common crockery, their sets of best china stayed in the parlor closet, and Louisa Ellis was
no richer nor better bred than they. Still she would use the china. She had for her supper a glass dish full of
sugared currants, a plate of little cakes, and one of light white biscuits. Also a leaf or two of lettuce, which she
cut up daintily. Louisa was very fond of lettuce, which she raised to perfection in her little garden. She ate quite
heartily, though in a delicate, pecking way; it seemed almost surprising that any considerable bulk of the food
should vanish.

Q.23 [30465398]
Which of the following style has the author used in the given story?

a expository

b descriptive

c analytical

d didactic

https://www.aspiration.ai/LAW/sis/Solution.jsp?qsetId=Lw5e7noFooA=&qsetName=LST Mock 20 2021 (CLAT) 45/208


12/18/2020 Mock Analysis

Solution:
 Answer key/Solution
Correct Answer : b
Your Answer : b
Details of activities in the story are described in a lucid manner. So, option (b) is
the answer. Other options are incorrect.

FeedBack Bookmark

https://www.aspiration.ai/LAW/sis/Solution.jsp?qsetId=Lw5e7noFooA=&qsetName=LST Mock 20 2021 (CLAT) 46/208


12/18/2020 Mock Analysis

Directions for questions 1 to 30: Each set of questions in this section is based on a single passage. Please
answer each question on the basis of what is stated or implied in the corresponding passage. In some
instances, more than one option may be the answer to the question; in such a case, please choose the option
that most accurately and comprehensively answers the question.

Passage – 5

It was late in the afternoon, and the light was waning. There was a difference in the look of the tree shadows out
in the yard. Somewhere in the distance cows were lowing and a little bell was tinkling; now and then a farm-
wagon tilted by, and the dust ew; some blue-shirted laborers with shovels over their shoulders plodded past;
little swarms of ies were dancing up and down before the peoples' faces in the soft air. There seemed to be a
gentle stir arising over everything for the mere sake of subsidence-a very premonition of rest and hush and
night.

This soft diurnal commotion was over Louisa Ellis also. She had been peacefully sewing at her sitting-room
window all the afternoon. Now she quilted her needle carefully into her work, which she folded precisely, and laid
in a basket with her thimble and thread and scissors. Louisa Ellis could not remember that ever in her life she
had mislaid one of these little feminine appurtenances, which had become, from long use and constant
association, a very part of her personality.

Louisa tied a green apron round her waist, and got out a at straw hat with a green ribbon. Then she went into
the garden with a little blue crockery bowl, to pick some currants for her tea. After the currants were picked she
sat on the back door-step and stemmed them, collecting the stems carefully in her apron, and afterwards
throwing them into the hen-coop. She looked sharply at the grass beside the step to see if any had fallen there.

Louisa was slow and still in her movements; it took her a long time to prepare her tea; but when ready it was set
forth with as much grace as if she had been a veritable guest to her own self. The little square table stood
exactly in the centre of the kitchen, and was covered with a starched linen cloth whose border pattern of owers
glistened. Louisa had a damask napkin on her tea-tray, where were arranged a cut-glass tumbler full of
teaspoons, a silver cream-pitcher, a china sugar-bowl, and one pink china cup and saucer. Louisa used china
every day-something which none of her neighbors did. They whispered about it among themselves. Their daily
tables were laid with common crockery, their sets of best china stayed in the parlor closet, and Louisa Ellis was
no richer nor better bred than they. Still she would use the china. She had for her supper a glass dish full of
sugared currants, a plate of little cakes, and one of light white biscuits. Also a leaf or two of lettuce, which she
cut up daintily. Louisa was very fond of lettuce, which she raised to perfection in her little garden. She ate quite
heartily, though in a delicate, pecking way; it seemed almost surprising that any considerable bulk of the food
should vanish.

Q.24 [30465398]
The lines, "This soft diurnal commotion was over Louisa Ellis also. She had been peacefully sewing at her
sitting-room window all the afternoon…" suggest that Louisa Ellis is:

a a person who goes about her daily chores like others before they get ready to relax in the evenings.

b a person who loves sewing all through the afternoons.

c a person who is able to keep her nerves calm by sewing and looking out of the window of her house.

https://www.aspiration.ai/LAW/sis/Solution.jsp?qsetId=Lw5e7noFooA=&qsetName=LST Mock 20 2021 (CLAT) 47/208


12/18/2020 Mock Analysis

d a seamstress whose work is highly appreciated by other people.

Solution:
 Answer key/Solution
Correct Answer : a
Your Answer : c
Option (a) is the correct answer because the given lines are continuation of the
rst paragraph which depicts the evening as a time for preparing for the night after a day's work. Option (b)
provides only half of the information. It cannot be inferred from the story whether Louisa Ellis is able to calm
her nerves by sewing and looking out of the window of her house. There is no doubt that Louisa Ellis is found
sewing but it cannot be inferred whether her work is appreciated by others or not.

FeedBack Bookmark

https://www.aspiration.ai/LAW/sis/Solution.jsp?qsetId=Lw5e7noFooA=&qsetName=LST Mock 20 2021 (CLAT) 48/208


12/18/2020 Mock Analysis

Directions for questions 1 to 30: Each set of questions in this section is based on a single passage. Please
answer each question on the basis of what is stated or implied in the corresponding passage. In some
instances, more than one option may be the answer to the question; in such a case, please choose the option
that most accurately and comprehensively answers the question.

Passage – 5

It was late in the afternoon, and the light was waning. There was a difference in the look of the tree shadows out
in the yard. Somewhere in the distance cows were lowing and a little bell was tinkling; now and then a farm-
wagon tilted by, and the dust ew; some blue-shirted laborers with shovels over their shoulders plodded past;
little swarms of ies were dancing up and down before the peoples' faces in the soft air. There seemed to be a
gentle stir arising over everything for the mere sake of subsidence-a very premonition of rest and hush and
night.

This soft diurnal commotion was over Louisa Ellis also. She had been peacefully sewing at her sitting-room
window all the afternoon. Now she quilted her needle carefully into her work, which she folded precisely, and laid
in a basket with her thimble and thread and scissors. Louisa Ellis could not remember that ever in her life she
had mislaid one of these little feminine appurtenances, which had become, from long use and constant
association, a very part of her personality.

Louisa tied a green apron round her waist, and got out a at straw hat with a green ribbon. Then she went into
the garden with a little blue crockery bowl, to pick some currants for her tea. After the currants were picked she
sat on the back door-step and stemmed them, collecting the stems carefully in her apron, and afterwards
throwing them into the hen-coop. She looked sharply at the grass beside the step to see if any had fallen there.

Louisa was slow and still in her movements; it took her a long time to prepare her tea; but when ready it was set
forth with as much grace as if she had been a veritable guest to her own self. The little square table stood
exactly in the centre of the kitchen, and was covered with a starched linen cloth whose border pattern of owers
glistened. Louisa had a damask napkin on her tea-tray, where were arranged a cut-glass tumbler full of
teaspoons, a silver cream-pitcher, a china sugar-bowl, and one pink china cup and saucer. Louisa used china
every day-something which none of her neighbors did. They whispered about it among themselves. Their daily
tables were laid with common crockery, their sets of best china stayed in the parlor closet, and Louisa Ellis was
no richer nor better bred than they. Still she would use the china. She had for her supper a glass dish full of
sugared currants, a plate of little cakes, and one of light white biscuits. Also a leaf or two of lettuce, which she
cut up daintily. Louisa was very fond of lettuce, which she raised to perfection in her little garden. She ate quite
heartily, though in a delicate, pecking way; it seemed almost surprising that any considerable bulk of the food
should vanish.

Q.25 [30465398]
What can be inferred from the line, "Louisa used china every day-something which none of her neighbors did."?

a She is amboyant.

b She is richer than others in the neighborhood.

c She loves to be seen as a woman of substance.

d She loves to treat herself well.

https://www.aspiration.ai/LAW/sis/Solution.jsp?qsetId=Lw5e7noFooA=&qsetName=LST Mock 20 2021 (CLAT) 49/208


12/18/2020 Mock Analysis

Solution:
 Answer key/Solution
Correct Answer : d
Your Answer : d
Options (a) and (c) are too shallow to be expected from a lady who is graceful and
hardworking like Louisa Ellis. Besides, none of these options can be inferred from the story. Option (b)
contradicts the information provided about Louisa Ellis in the given story. Only option (d) can be inferred
because it is said in the given story that her neighbours' daily tables were laid with common crockery, their
sets of best china stayed in the parlor closet, and Louisa Ellis was no richer nor better bred than they.
FeedBack Bookmark

https://www.aspiration.ai/LAW/sis/Solution.jsp?qsetId=Lw5e7noFooA=&qsetName=LST Mock 20 2021 (CLAT) 50/208


12/18/2020 Mock Analysis

Directions for questions 1 to 30: Each set of questions in this section is based on a single passage. Please
answer each question on the basis of what is stated or implied in the corresponding passage. In some
instances, more than one option may be the answer to the question; in such a case, please choose the option
that most accurately and comprehensively answers the question.

Passage – 6

As President-elect Joe Biden prepares to take charge of America on January 20, Pakistan hopes to reset
bilateral relations with the US and draw Washington into the Kashmir dispute with India. Mobilising America on
Kashmir has always been a major preoccupation for Pakistan. It has become an obsession after India altered
the constitutional status of Jammu and Kashmir last year.

Pakistan's Kashmir strategy over the last three decades and more is a familiar one. It is about triggering
violence in Kashmir and intensifying the military confrontation at the Line of Control. It then appeals to
Washington to defuse the crisis that could escalate to the nuclear level and compel India to talk Kashmir with
Pakistan. This script has played out frequently since the late 1980s and the US has inevitably stepped in. The
degree of American intervention has, however, varied from crisis to crisis.

Meanwhile, India has learnt to adapt. In the past, India was hesitant to escalate hostilities because of the
nuclear factor and the fear of third-party mediation in Kashmir. Prime Minister Narendra Modi, however, is
unafraid to escalate. Delhi has recognised that the threat of escalation cuts both ways and it is possible to turn
the nuclear dimension and external intervention to India's advantage. Consider, for example, the recent claim of
a former speaker of Pakistan's national assembly that army chief General Qamar Jawed Bajwa had quivering
knees and a sweaty brow, when Modi threatened to rain destruction on Pakistan if Islamabad did not
immediately release Wing Commander Abhinandan Varthaman, who had bailed out in Pakistan after a dog ght
with the Pakistani Air Force in February 2019.

Given the region's temptation for the theatrical, this claim is easily discounted. But the Indian threat to escalate
inevitably brought Washington into the picture. There were reports that President Donald Trump persuaded
Pakistan to release the Indian pilot and avoid escalation.

If Delhi is more willing [1] before to raise the stakes in confronting Pakistan-supported terror, it has also
bene ted from the shift in the relative balance of power between India and Pakistan. India's economy today at
$2.7 trillion is nearly 10 times that of Pakistan. As the US-India partnership becomes more comprehensive and
global, Pakistan's ability to get Washington to act against India has declined. This does not mean Pakistan has
no leverage at all in Washington; or that it might simply give up on the old Kashmir strategy. Pakistan sits at the
critical con uence of the subcontinent, Central Asia and the Gulf. It has nuclear weapons and a strong army that
can shape regional geopolitics.

Q.26 [30465398]
Out of the following options, which one is factually correct in the light of the passage?

a There is an existing defence tie up between India and the US.

b The Pakistani army is the strongest in the South Asian region.

c The Pakistani economy is half the size of India's.

https://www.aspiration.ai/LAW/sis/Solution.jsp?qsetId=Lw5e7noFooA=&qsetName=LST Mock 20 2021 (CLAT) 51/208


12/18/2020 Mock Analysis

d The American government has not been unwilling to intervene in the Kashmir issue.

Solution:
 Answer key/Solution
Correct Answer : d
Your Answer : d
Refer to the rst 2 sentences of the passage where the author mentions that
Pakistan has always wanted the US to step in and the latter has obliged many a time. Hence, option (d) is
factually correct in the light of the passage. Option (a) is not mentioned anywhere in the passage. Option (b) is
wrong because the last sentence of the passage says that Pakistan has a strong army and not the strongest.
Option (c) is incorrect because the passage says that the Indian economy is 10 times the size of Pakistan's.

FeedBack Bookmark

https://www.aspiration.ai/LAW/sis/Solution.jsp?qsetId=Lw5e7noFooA=&qsetName=LST Mock 20 2021 (CLAT) 52/208


12/18/2020 Mock Analysis

Directions for questions 1 to 30: Each set of questions in this section is based on a single passage. Please
answer each question on the basis of what is stated or implied in the corresponding passage. In some
instances, more than one option may be the answer to the question; in such a case, please choose the option
that most accurately and comprehensively answers the question.

Passage – 6

As President-elect Joe Biden prepares to take charge of America on January 20, Pakistan hopes to reset
bilateral relations with the US and draw Washington into the Kashmir dispute with India. Mobilising America on
Kashmir has always been a major preoccupation for Pakistan. It has become an obsession after India altered
the constitutional status of Jammu and Kashmir last year.

Pakistan's Kashmir strategy over the last three decades and more is a familiar one. It is about triggering
violence in Kashmir and intensifying the military confrontation at the Line of Control. It then appeals to
Washington to defuse the crisis that could escalate to the nuclear level and compel India to talk Kashmir with
Pakistan. This script has played out frequently since the late 1980s and the US has inevitably stepped in. The
degree of American intervention has, however, varied from crisis to crisis.

Meanwhile, India has learnt to adapt. In the past, India was hesitant to escalate hostilities because of the
nuclear factor and the fear of third-party mediation in Kashmir. Prime Minister Narendra Modi, however, is
unafraid to escalate. Delhi has recognised that the threat of escalation cuts both ways and it is possible to turn
the nuclear dimension and external intervention to India's advantage. Consider, for example, the recent claim of
a former speaker of Pakistan's national assembly that army chief General Qamar Jawed Bajwa had quivering
knees and a sweaty brow, when Modi threatened to rain destruction on Pakistan if Islamabad did not
immediately release Wing Commander Abhinandan Varthaman, who had bailed out in Pakistan after a dog ght
with the Pakistani Air Force in February 2019.

Given the region's temptation for the theatrical, this claim is easily discounted. But the Indian threat to escalate
inevitably brought Washington into the picture. There were reports that President Donald Trump persuaded
Pakistan to release the Indian pilot and avoid escalation.

If Delhi is more willing [1] before to raise the stakes in confronting Pakistan-supported terror, it has also
bene ted from the shift in the relative balance of power between India and Pakistan. India's economy today at
$2.7 trillion is nearly 10 times that of Pakistan. As the US-India partnership becomes more comprehensive and
global, Pakistan's ability to get Washington to act against India has declined. This does not mean Pakistan has
no leverage at all in Washington; or that it might simply give up on the old Kashmir strategy. Pakistan sits at the
critical con uence of the subcontinent, Central Asia and the Gulf. It has nuclear weapons and a strong army that
can shape regional geopolitics.

Q.27 [30465398]
Out of the following options, which one captures the central idea of the passage?

a The abrogation of Article 370 in Kashmir and the Pakistani response to it

b Pakistan's Kashmir strategy vis-a-vis the US

c Pakistan's clout in South Asian politics

https://www.aspiration.ai/LAW/sis/Solution.jsp?qsetId=Lw5e7noFooA=&qsetName=LST Mock 20 2021 (CLAT) 53/208


12/18/2020 Mock Analysis

d The American government's eagerness to defuse the Kashmir problem

Solution:
 Answer key/Solution
Correct Answer : b
Your Answer : b
Option (b) is the most appropriate answer because the passage talks about
Pakistan's strategy on Kashmir and how it involves the US in it. The remaining options are narrow in scope and
so, cannot be considered to be correct.

FeedBack Bookmark

https://www.aspiration.ai/LAW/sis/Solution.jsp?qsetId=Lw5e7noFooA=&qsetName=LST Mock 20 2021 (CLAT) 54/208


12/18/2020 Mock Analysis

Directions for questions 1 to 30: Each set of questions in this section is based on a single passage. Please
answer each question on the basis of what is stated or implied in the corresponding passage. In some
instances, more than one option may be the answer to the question; in such a case, please choose the option
that most accurately and comprehensively answers the question.

Passage – 6

As President-elect Joe Biden prepares to take charge of America on January 20, Pakistan hopes to reset
bilateral relations with the US and draw Washington into the Kashmir dispute with India. Mobilising America on
Kashmir has always been a major preoccupation for Pakistan. It has become an obsession after India altered
the constitutional status of Jammu and Kashmir last year.

Pakistan's Kashmir strategy over the last three decades and more is a familiar one. It is about triggering
violence in Kashmir and intensifying the military confrontation at the Line of Control. It then appeals to
Washington to defuse the crisis that could escalate to the nuclear level and compel India to talk Kashmir with
Pakistan. This script has played out frequently since the late 1980s and the US has inevitably stepped in. The
degree of American intervention has, however, varied from crisis to crisis.

Meanwhile, India has learnt to adapt. In the past, India was hesitant to escalate hostilities because of the
nuclear factor and the fear of third-party mediation in Kashmir. Prime Minister Narendra Modi, however, is
unafraid to escalate. Delhi has recognised that the threat of escalation cuts both ways and it is possible to turn
the nuclear dimension and external intervention to India's advantage. Consider, for example, the recent claim of
a former speaker of Pakistan's national assembly that army chief General Qamar Jawed Bajwa had quivering
knees and a sweaty brow, when Modi threatened to rain destruction on Pakistan if Islamabad did not
immediately release Wing Commander Abhinandan Varthaman, who had bailed out in Pakistan after a dog ght
with the Pakistani Air Force in February 2019.

Given the region's temptation for the theatrical, this claim is easily discounted. But the Indian threat to escalate
inevitably brought Washington into the picture. There were reports that President Donald Trump persuaded
Pakistan to release the Indian pilot and avoid escalation.

If Delhi is more willing [1] before to raise the stakes in confronting Pakistan-supported terror, it has also
bene ted from the shift in the relative balance of power between India and Pakistan. India's economy today at
$2.7 trillion is nearly 10 times that of Pakistan. As the US-India partnership becomes more comprehensive and
global, Pakistan's ability to get Washington to act against India has declined. This does not mean Pakistan has
no leverage at all in Washington; or that it might simply give up on the old Kashmir strategy. Pakistan sits at the
critical con uence of the subcontinent, Central Asia and the Gulf. It has nuclear weapons and a strong army that
can shape regional geopolitics.

Q.28 [30465398]
Out of the following options, which one is a possible inference in the light of the last paragraph of the passage?

a Pakistan's nuclear arsenal is higher than India's.

b Pakistani nuclear bombs can hit any Indian city.

c Pakistan can destabilise certain countries in South Asia.

https://www.aspiration.ai/LAW/sis/Solution.jsp?qsetId=Lw5e7noFooA=&qsetName=LST Mock 20 2021 (CLAT) 55/208


12/18/2020 Mock Analysis

d Pakistan does not want to get involved in the Kashmir issue after the Abhinandan Varthaman incident.

Solution:
 Answer key/Solution
Correct Answer : c
Your Answer : c
Refer to the last sentence of the passage where the author mentions that
Pakistan has a strong army that can shape regional geopolitics. Hence, option (c) is a possible scenario.
Option (d) is wrong because it is clearly mentioned in the passage that Pakistan won't give up on the Kashmir
issue. Options (a) and (b) are wrong because although we know that Pakistan has nuclear bombs, we don't
know about the capabilities. Hence, both (a) and (b) are out of scope.

FeedBack Bookmark

https://www.aspiration.ai/LAW/sis/Solution.jsp?qsetId=Lw5e7noFooA=&qsetName=LST Mock 20 2021 (CLAT) 56/208


12/18/2020 Mock Analysis

Directions for questions 1 to 30: Each set of questions in this section is based on a single passage. Please
answer each question on the basis of what is stated or implied in the corresponding passage. In some
instances, more than one option may be the answer to the question; in such a case, please choose the option
that most accurately and comprehensively answers the question.

Passage – 6

As President-elect Joe Biden prepares to take charge of America on January 20, Pakistan hopes to reset
bilateral relations with the US and draw Washington into the Kashmir dispute with India. Mobilising America on
Kashmir has always been a major preoccupation for Pakistan. It has become an obsession after India altered
the constitutional status of Jammu and Kashmir last year.

Pakistan's Kashmir strategy over the last three decades and more is a familiar one. It is about triggering
violence in Kashmir and intensifying the military confrontation at the Line of Control. It then appeals to
Washington to defuse the crisis that could escalate to the nuclear level and compel India to talk Kashmir with
Pakistan. This script has played out frequently since the late 1980s and the US has inevitably stepped in. The
degree of American intervention has, however, varied from crisis to crisis.

Meanwhile, India has learnt to adapt. In the past, India was hesitant to escalate hostilities because of the
nuclear factor and the fear of third-party mediation in Kashmir. Prime Minister Narendra Modi, however, is
unafraid to escalate. Delhi has recognised that the threat of escalation cuts both ways and it is possible to turn
the nuclear dimension and external intervention to India's advantage. Consider, for example, the recent claim of
a former speaker of Pakistan's national assembly that army chief General Qamar Jawed Bajwa had quivering
knees and a sweaty brow, when Modi threatened to rain destruction on Pakistan if Islamabad did not
immediately release Wing Commander Abhinandan Varthaman, who had bailed out in Pakistan after a dog ght
with the Pakistani Air Force in February 2019.

Given the region's temptation for the theatrical, this claim is easily discounted. But the Indian threat to escalate
inevitably brought Washington into the picture. There were reports that President Donald Trump persuaded
Pakistan to release the Indian pilot and avoid escalation.

If Delhi is more willing [1] before to raise the stakes in confronting Pakistan-supported terror, it has also
bene ted from the shift in the relative balance of power between India and Pakistan. India's economy today at
$2.7 trillion is nearly 10 times that of Pakistan. As the US-India partnership becomes more comprehensive and
global, Pakistan's ability to get Washington to act against India has declined. This does not mean Pakistan has
no leverage at all in Washington; or that it might simply give up on the old Kashmir strategy. Pakistan sits at the
critical con uence of the subcontinent, Central Asia and the Gulf. It has nuclear weapons and a strong army that
can shape regional geopolitics.

Q.29 [30465398]
Out of the following options, which one can replace [1] in the last paragraph?

a Than

b Then

c For

https://www.aspiration.ai/LAW/sis/Solution.jsp?qsetId=Lw5e7noFooA=&qsetName=LST Mock 20 2021 (CLAT) 57/208


12/18/2020 Mock Analysis

d To

Solution:
 Answer key/Solution
Correct Answer : a
Your Answer : a
The grammatically correct option is 'than'. Hence, the correct answer is option (a).

FeedBack Bookmark

https://www.aspiration.ai/LAW/sis/Solution.jsp?qsetId=Lw5e7noFooA=&qsetName=LST Mock 20 2021 (CLAT) 58/208


12/18/2020 Mock Analysis

Directions for questions 1 to 30: Each set of questions in this section is based on a single passage. Please
answer each question on the basis of what is stated or implied in the corresponding passage. In some
instances, more than one option may be the answer to the question; in such a case, please choose the option
that most accurately and comprehensively answers the question.

Passage – 6

As President-elect Joe Biden prepares to take charge of America on January 20, Pakistan hopes to reset
bilateral relations with the US and draw Washington into the Kashmir dispute with India. Mobilising America on
Kashmir has always been a major preoccupation for Pakistan. It has become an obsession after India altered
the constitutional status of Jammu and Kashmir last year.

Pakistan's Kashmir strategy over the last three decades and more is a familiar one. It is about triggering
violence in Kashmir and intensifying the military confrontation at the Line of Control. It then appeals to
Washington to defuse the crisis that could escalate to the nuclear level and compel India to talk Kashmir with
Pakistan. This script has played out frequently since the late 1980s and the US has inevitably stepped in. The
degree of American intervention has, however, varied from crisis to crisis.

Meanwhile, India has learnt to adapt. In the past, India was hesitant to escalate hostilities because of the
nuclear factor and the fear of third-party mediation in Kashmir. Prime Minister Narendra Modi, however, is
unafraid to escalate. Delhi has recognised that the threat of escalation cuts both ways and it is possible to turn
the nuclear dimension and external intervention to India's advantage. Consider, for example, the recent claim of
a former speaker of Pakistan's national assembly that army chief General Qamar Jawed Bajwa had quivering
knees and a sweaty brow, when Modi threatened to rain destruction on Pakistan if Islamabad did not
immediately release Wing Commander Abhinandan Varthaman, who had bailed out in Pakistan after a dog ght
with the Pakistani Air Force in February 2019.

Given the region's temptation for the theatrical, this claim is easily discounted. But the Indian threat to escalate
inevitably brought Washington into the picture. There were reports that President Donald Trump persuaded
Pakistan to release the Indian pilot and avoid escalation.

If Delhi is more willing [1] before to raise the stakes in confronting Pakistan-supported terror, it has also
bene ted from the shift in the relative balance of power between India and Pakistan. India's economy today at
$2.7 trillion is nearly 10 times that of Pakistan. As the US-India partnership becomes more comprehensive and
global, Pakistan's ability to get Washington to act against India has declined. This does not mean Pakistan has
no leverage at all in Washington; or that it might simply give up on the old Kashmir strategy. Pakistan sits at the
critical con uence of the subcontinent, Central Asia and the Gulf. It has nuclear weapons and a strong army that
can shape regional geopolitics.

Q.30 [30465398]
Out of the following options, which one is factually correct in the light of the passage?

a Pakistani forces often resort to unprovoked shelling on the Line of Control.

b Pakistani forces often target civilians living near the Line of Control.

c When Pakistani forces re across the LOC, Indian forces retaliate.

https://www.aspiration.ai/LAW/sis/Solution.jsp?qsetId=Lw5e7noFooA=&qsetName=LST Mock 20 2021 (CLAT) 59/208


12/18/2020 Mock Analysis

d Pakistani forces often create trouble on the Line of Control.

Solution:
 Answer key/Solution
Correct Answer : d
Your Answer : c
Refer to the second sentence of the second paragraph where the author mentions
"military confrontations" on the LOC. Options (a), (b) and (c) are very speci c and nothing of that sort is
mentioned in the passage. We don't know whether military confrontations refer to shelling. It might mean just
small arms re. There is also no mention of retaliation by India forces. Also, nothing has been said about
civilian areas being hit by Pakistani forces. Therefore, other than option (d), the remaining options are out of
scope.

FeedBack Bookmark

Sec 2

https://www.aspiration.ai/LAW/sis/Solution.jsp?qsetId=Lw5e7noFooA=&qsetName=LST Mock 20 2021 (CLAT) 60/208


12/18/2020 Mock Analysis

Passage-01

Face-to-face negotiations between Britain and the European Union over a trade deal restarted earlier as the two
sides make a last ditch attempt to reach an agreement with just ve weeks to go before their current
relationship ends.

There is currently no call scheduled between UK Prime Minister [1] and EU Commission President [2], a UK
source told Reuters, after the Times newspaper reported the pair would speak in the next 48 hours.

The rst sign of movement -- either towards a deal or that talks are crumbling -- is likely to be a call between
[1]and [2].

The Times also reported that the European Commission has started to “lean on” EU negotiator Michel Barnier to
reach a deal with Britain, raising hopes that an agreement could come.

Barnier and Britain’s negotiator David Frost are working to secure a deal before the UK’s transition period with
the EU ends on December 31, 2020.

Both sides said that there were still big differences to overcome, as they both called for the other to
compromise on the three main issues of contention - shing, state aid and how to resolve any future disputes.

[1] talked of “substantial and important differences”, while Barnier referred to “signi cant divergences”.

Britain left the bloc on January 31 this year and a “no-deal” nal exit would snarl borders, spook nancial
markets and disrupt delicate supply chains that stretch across Europe and beyond -- just as the world grapples
with the vast economic cost of the COVID-19 outbreak.

On the major sticking point of shing, some media reports suggested that Britain had rejected an EU proposal
on the value of sh quota that European eets catch in British waters that are due to be restored to the UK.

Q.31 [30465398]
What are the names of the current UK Prime Minister and European Union Commission President whose names
have been redacted with [1] and [2] in the passage above?

a Boris Johnson and Ursula von der Leyen

b Jeremy Hunt and Ursula von der Leyen

c Dominic Rennie Raab and Jean-Claude Juncker

d Damian Green and Jean-Claude Juncker

https://www.aspiration.ai/LAW/sis/Solution.jsp?qsetId=Lw5e7noFooA=&qsetName=LST Mock 20 2021 (CLAT) 61/208


12/18/2020 Mock Analysis

Solution:
 Answer key/Solution
Correct Answer : a
Your Answer : a
Face-to-face negotiations between Britain and the European Union over a trade
deal restarted earlier as the two sides make a last ditch attempt to reach an agreement with just ve weeks to
go before their current relationship ends. There is currently no call scheduled between UK Prime Minister
Boris Johnson and EU Commission President Ursula von der Leyen, a UK source told Reuters, after the Times
newspaper reported the pair would speak in the next 48 hours.

FeedBack Bookmark

Passage-01

Face-to-face negotiations between Britain and the European Union over a trade deal restarted earlier as the two
sides make a last ditch attempt to reach an agreement with just ve weeks to go before their current
relationship ends.

There is currently no call scheduled between UK Prime Minister [1] and EU Commission President [2], a UK
source told Reuters, after the Times newspaper reported the pair would speak in the next 48 hours.

The rst sign of movement -- either towards a deal or that talks are crumbling -- is likely to be a call between
[1]and [2].

The Times also reported that the European Commission has started to “lean on” EU negotiator Michel Barnier to
reach a deal with Britain, raising hopes that an agreement could come.

Barnier and Britain’s negotiator David Frost are working to secure a deal before the UK’s transition period with
the EU ends on December 31, 2020.

Both sides said that there were still big differences to overcome, as they both called for the other to
compromise on the three main issues of contention - shing, state aid and how to resolve any future disputes.

[1] talked of “substantial and important differences”, while Barnier referred to “signi cant divergences”.

Britain left the bloc on January 31 this year and a “no-deal” nal exit would snarl borders, spook nancial
markets and disrupt delicate supply chains that stretch across Europe and beyond -- just as the world grapples
with the vast economic cost of the COVID-19 outbreak.

On the major sticking point of shing, some media reports suggested that Britain had rejected an EU proposal
on the value of sh quota that European eets catch in British waters that are due to be restored to the UK.

Q.32 [30465398]
Who among the following UK Prime Ministers called for the Brexit referendum and when the Brexit referendum
took place?

https://www.aspiration.ai/LAW/sis/Solution.jsp?qsetId=Lw5e7noFooA=&qsetName=LST Mock 20 2021 (CLAT) 62/208


12/18/2020 Mock Analysis

a David Cameron, 23 June 2016

b Theresa May, 23 June 2017

c Gordon Brown, 23 June 2016

d Tony Blair, 23 June 2014

Solution:
 Answer key/Solution
Correct Answer : a
Your Answer : a
David Cameron was the UK PM who called for the Brexit referendum. He resigned
the PM position post the verdict of the referendum. Brexit referendum took place on 23 June 2016 in the
United Kingdom (UK) and Gibraltar to ask the electorate whether the country should remain a member of, or
leave, the European Union (EU). The United Kingdom of Great Britain and Northern Ireland, commonly known
as the United Kingdom (UK or U.K.) or Britain. Northern Ireland is the only part of the United Kingdom that
shares a land border with another sovereign state, the Republic of Ireland, usually referred to as Ireland.

FeedBack Bookmark

https://www.aspiration.ai/LAW/sis/Solution.jsp?qsetId=Lw5e7noFooA=&qsetName=LST Mock 20 2021 (CLAT) 63/208


12/18/2020 Mock Analysis

Passage-01

Face-to-face negotiations between Britain and the European Union over a trade deal restarted earlier as the two
sides make a last ditch attempt to reach an agreement with just ve weeks to go before their current
relationship ends.

There is currently no call scheduled between UK Prime Minister [1] and EU Commission President [2], a UK
source told Reuters, after the Times newspaper reported the pair would speak in the next 48 hours.

The rst sign of movement -- either towards a deal or that talks are crumbling -- is likely to be a call between
[1]and [2].

The Times also reported that the European Commission has started to “lean on” EU negotiator Michel Barnier to
reach a deal with Britain, raising hopes that an agreement could come.

Barnier and Britain’s negotiator David Frost are working to secure a deal before the UK’s transition period with
the EU ends on December 31, 2020.

Both sides said that there were still big differences to overcome, as they both called for the other to
compromise on the three main issues of contention - shing, state aid and how to resolve any future disputes.

[1] talked of “substantial and important differences”, while Barnier referred to “signi cant divergences”.

Britain left the bloc on January 31 this year and a “no-deal” nal exit would snarl borders, spook nancial
markets and disrupt delicate supply chains that stretch across Europe and beyond -- just as the world grapples
with the vast economic cost of the COVID-19 outbreak.

On the major sticking point of shing, some media reports suggested that Britain had rejected an EU proposal
on the value of sh quota that European eets catch in British waters that are due to be restored to the UK.

Q.33 [30465398]
The “backstop” was initially proposed so as not to compromise on which one of the following peace
agreements reached between the British and Irish government in 1998?

a Common Travel Area Agreement

b Smart Border Agreement

c Customs Union Agreement

d Good Friday Agreement

https://www.aspiration.ai/LAW/sis/Solution.jsp?qsetId=Lw5e7noFooA=&qsetName=LST Mock 20 2021 (CLAT) 64/208


12/18/2020 Mock Analysis

Solution:
 Answer key/Solution
Correct Answer : d
Your Answer : d
The Belfast Agreement is also known as the Good Friday Agreement, because it
was reached on Good Friday, 10 April 1998. It was a peace agreement between the British and Irish
governments, and most of the political parties in Northern Ireland, on how Northern Ireland should be
governed. The backstop is an arrangement for the Irish border that will come into effect if no other solutions
to maintain the current open border can be agreed once the UK leaves the EU. It is intended to protect the
Good Friday Agreement/Belfast Agreement and keep an open border between Northern Ireland and Republic
of Ireland after Brexit.

FeedBack Bookmark

Passage-01

Face-to-face negotiations between Britain and the European Union over a trade deal restarted earlier as the two
sides make a last ditch attempt to reach an agreement with just ve weeks to go before their current
relationship ends.

There is currently no call scheduled between UK Prime Minister [1] and EU Commission President [2], a UK
source told Reuters, after the Times newspaper reported the pair would speak in the next 48 hours.

The rst sign of movement -- either towards a deal or that talks are crumbling -- is likely to be a call between
[1]and [2].

The Times also reported that the European Commission has started to “lean on” EU negotiator Michel Barnier to
reach a deal with Britain, raising hopes that an agreement could come.

Barnier and Britain’s negotiator David Frost are working to secure a deal before the UK’s transition period with
the EU ends on December 31, 2020.

Both sides said that there were still big differences to overcome, as they both called for the other to
compromise on the three main issues of contention - shing, state aid and how to resolve any future disputes.

[1] talked of “substantial and important differences”, while Barnier referred to “signi cant divergences”.

Britain left the bloc on January 31 this year and a “no-deal” nal exit would snarl borders, spook nancial
markets and disrupt delicate supply chains that stretch across Europe and beyond -- just as the world grapples
with the vast economic cost of the COVID-19 outbreak.

On the major sticking point of shing, some media reports suggested that Britain had rejected an EU proposal
on the value of sh quota that European eets catch in British waters that are due to be restored to the UK.

https://www.aspiration.ai/LAW/sis/Solution.jsp?qsetId=Lw5e7noFooA=&qsetName=LST Mock 20 2021 (CLAT) 65/208


12/18/2020 Mock Analysis

Q.34 [30465398]
A recent event happened in the month of January 2020 with reference to UK is dubbed as Megxit. The event
involves:

a The Prime Minister of Britain

b The British Royal Family

c One of the Britain’s largest company

d People of a certain ethnicity

Solution:
 Answer key/Solution
Correct Answer : b
Your Answer : b
On 8 January 2020, Prince Harry, Duke of Sussex and Meghan, Duchess of Sussex
announced their decision on Instagram to "step back as 'senior' members of the Royal Family", splitting their
time between the United Kingdom and North America, and to become nancially independent; the event was
dubbed Megxit (a portmanteau of the words "Meghan" and "exit" and a play on the word "Brexit"),

FeedBack Bookmark

https://www.aspiration.ai/LAW/sis/Solution.jsp?qsetId=Lw5e7noFooA=&qsetName=LST Mock 20 2021 (CLAT) 66/208


12/18/2020 Mock Analysis

Passage-01

Face-to-face negotiations between Britain and the European Union over a trade deal restarted earlier as the two
sides make a last ditch attempt to reach an agreement with just ve weeks to go before their current
relationship ends.

There is currently no call scheduled between UK Prime Minister [1] and EU Commission President [2], a UK
source told Reuters, after the Times newspaper reported the pair would speak in the next 48 hours.

The rst sign of movement -- either towards a deal or that talks are crumbling -- is likely to be a call between
[1]and [2].

The Times also reported that the European Commission has started to “lean on” EU negotiator Michel Barnier to
reach a deal with Britain, raising hopes that an agreement could come.

Barnier and Britain’s negotiator David Frost are working to secure a deal before the UK’s transition period with
the EU ends on December 31, 2020.

Both sides said that there were still big differences to overcome, as they both called for the other to
compromise on the three main issues of contention - shing, state aid and how to resolve any future disputes.

[1] talked of “substantial and important differences”, while Barnier referred to “signi cant divergences”.

Britain left the bloc on January 31 this year and a “no-deal” nal exit would snarl borders, spook nancial
markets and disrupt delicate supply chains that stretch across Europe and beyond -- just as the world grapples
with the vast economic cost of the COVID-19 outbreak.

On the major sticking point of shing, some media reports suggested that Britain had rejected an EU proposal
on the value of sh quota that European eets catch in British waters that are due to be restored to the UK.

Q.35 [30465398]
Which of the following is not a reason for the UK leaving European Union?

a The UK has to adopt and follow the Euro currency for the all the transactions.

b The UK has to pay millions of pounds each week as a contribution to the European budget.

c The extremely bureaucratic nature of the European parliament is hurting British exporters.

d Migration from the European Union into Britain is creating an imbalance in the welfare schemes of the UK
government.

https://www.aspiration.ai/LAW/sis/Solution.jsp?qsetId=Lw5e7noFooA=&qsetName=LST Mock 20 2021 (CLAT) 67/208


12/18/2020 Mock Analysis

Solution:
 Answer key/Solution
Correct Answer : a
Your Answer : a
The Leave Campaign argues that Britain is losing out a big deal by staying in the
EU. It has to pay millions of pounds each week as a contribution to the European budget. The extremely
bureaucratic nature of the European parliament is hurting British exporters Migration from the European Union
into Britain (mainly PIGS economies) is creating an imbalance in the welfare schemes of the UK government.
The UK does not use the Euro and has retained its own currency, the Pound Sterling even when it was a
member of EU. Hence option D is not a reason for the UK leaving European Union.

FeedBack Bookmark

Passage-01

Face-to-face negotiations between Britain and the European Union over a trade deal restarted earlier as the two
sides make a last ditch attempt to reach an agreement with just ve weeks to go before their current
relationship ends.

There is currently no call scheduled between UK Prime Minister [1] and EU Commission President [2], a UK
source told Reuters, after the Times newspaper reported the pair would speak in the next 48 hours.

The rst sign of movement -- either towards a deal or that talks are crumbling -- is likely to be a call between
[1]and [2].

The Times also reported that the European Commission has started to “lean on” EU negotiator Michel Barnier to
reach a deal with Britain, raising hopes that an agreement could come.

Barnier and Britain’s negotiator David Frost are working to secure a deal before the UK’s transition period with
the EU ends on December 31, 2020.

Both sides said that there were still big differences to overcome, as they both called for the other to
compromise on the three main issues of contention - shing, state aid and how to resolve any future disputes.

[1] talked of “substantial and important differences”, while Barnier referred to “signi cant divergences”.

Britain left the bloc on January 31 this year and a “no-deal” nal exit would snarl borders, spook nancial
markets and disrupt delicate supply chains that stretch across Europe and beyond -- just as the world grapples
with the vast economic cost of the COVID-19 outbreak.

On the major sticking point of shing, some media reports suggested that Britain had rejected an EU proposal
on the value of sh quota that European eets catch in British waters that are due to be restored to the UK.

Q.36 [30465398]
Which of the following is not a negative impact of the Brexit referendum on India?

https://www.aspiration.ai/LAW/sis/Solution.jsp?qsetId=Lw5e7noFooA=&qsetName=LST Mock 20 2021 (CLAT) 68/208


12/18/2020 Mock Analysis

a Prices of gold, electronic goods, among others may increase.

b Due to raise in the value of Pound sterling, Indian exports to the UK will be boosted.

c Cheaper rupee will make Indian exports, including IT and ITeS, competitive.

d Sensex and Nifty may tumble in the short-run.

Solution:
 Answer key/Solution
Correct Answer : b
Negative Impacts of the Brexit referendum on India are 1. India will have to adjust
to changing world order. 2. There may be foreign fund out ow and dollar rise.3.
Rupee may depreciate because of the double effect of foreign fund out ow and dollar rise. 4. This may
increase petrol and diesel prices to an extent. 5. The government then may want to reduce additional excise
duty imposed on fuel when it was on a downward trajectory. 6.This may increase scal de cit unless revenue
increased. 7. Prices of gold, electronic goods, among others may also increase. 8. Sensex and Nifty may
tumble in the short-run. 9. The falling value of the pound could render several existing contracts loss-making
10. Many Indian companies are listed on the London Stock Exchange and many have European headquarters
in London. Brexit will take away this advantage. 11. Due to fall in the value of Pound sterling, Indian exports to
the UK will suffer. Cheaper rupee will make Indian exports, including IT and ITeS, competitive.

FeedBack Bookmark

Passage-02

Diego Armando Maradona, who is regarded as one of football's greatest players ever, died of heart failure at the
age of 60 as the world mourned him.

Maradona was born on Oct. 30, 1960, in the [1] capital Buenos Aires. Known for his pace, high technique, and
dribbling skills, Maradona was a multifaceted talent on the green eld as he could play as an attacking
mid elder and forward.

During his football career, Maradona played for [1]s Juniors, Boca Juniors, Newell's Old Boys, Spanish clubs
Barcelona and Sevilla, and Italian team Napoli.

Separately he was a regular player for the [1] national team, where he won the [2] FIFA World Cup in Mexico.

He also scored 34 goals in 91 international caps for [1]. Maradona played in four FIFA World Cups, including the
[2] World Cup in Mexico, where he captained [1] and led them to victory over West Germany in the nal, and won
the Golden Ball as the tournament's best player.

In the [2] World Cup quarter nal, he scored both goals in a 2–1 victory over England that entered football
history for two different reasons. The rst goal was an unpenalized handling foul known as the "[3]", while the
second goal followed a 60 m (66 yd) dribble past ve England players, voted "Goal of the Century" by FIFA.com
voters in 2002. He made his full international debut at age 16, against Hungary, on 27 February 1977.

https://www.aspiration.ai/LAW/sis/Solution.jsp?qsetId=Lw5e7noFooA=&qsetName=LST Mock 20 2021 (CLAT) 69/208


12/18/2020 Mock Analysis

Q.37 [30465398]
Diego Armando Maradona, who is regarded as one of football's greatest players ever passed away. He belongs
which of the following countries which has been redacted with [1] in the passage above?

a Brazil

b Spain

c Argentina

d Cuba

Solution:
 Answer key/Solution
Correct Answer : c
Your Answer : c
Diego Armando Maradona, who is regarded as one of football's greatest players
ever, died of heart failure at the age of 60 as the world mourned him. Maradona was born on Oct. 30, 1960, in
the Argentine capital Buenos Aires.
FeedBack Bookmark

Passage-02

Diego Armando Maradona, who is regarded as one of football's greatest players ever, died of heart failure at the
age of 60 as the world mourned him.

Maradona was born on Oct. 30, 1960, in the [1] capital Buenos Aires. Known for his pace, high technique, and
dribbling skills, Maradona was a multifaceted talent on the green eld as he could play as an attacking
mid elder and forward.

During his football career, Maradona played for [1]s Juniors, Boca Juniors, Newell's Old Boys, Spanish clubs
Barcelona and Sevilla, and Italian team Napoli.

Separately he was a regular player for the [1] national team, where he won the [2] FIFA World Cup in Mexico.

He also scored 34 goals in 91 international caps for [1]. Maradona played in four FIFA World Cups, including the
[2] World Cup in Mexico, where he captained [1] and led them to victory over West Germany in the nal, and won
the Golden Ball as the tournament's best player.

In the [2] World Cup quarter nal, he scored both goals in a 2–1 victory over England that entered football
history for two different reasons. The rst goal was an unpenalized handling foul known as the "[3]", while the
second goal followed a 60 m (66 yd) dribble past ve England players, voted "Goal of the Century" by FIFA.com
voters in 2002. He made his full international debut at age 16, against Hungary, on 27 February 1977.

https://www.aspiration.ai/LAW/sis/Solution.jsp?qsetId=Lw5e7noFooA=&qsetName=LST Mock 20 2021 (CLAT) 70/208


12/18/2020 Mock Analysis

Q.38 [30465398]
Diego Armando Maradona captained [1] and led them to victory over West Germany in the [2] FIFA World Cup
nal held in Mexico. Which of the following years has been redacted with [2] in the passage above?

a 1982

b 1986

c 1990

d 1994

Solution:
 Answer key/Solution
Correct Answer : b
Your Answer : b
Diego Armando Maradona scored 34 goals in 91 international caps for Argentina.
Maradona played in four FIFA World Cups, including the 1986 World Cup in Mexico, where he captained
Argentina and led them to victory over West Germany in the nal, and won the Golden Ball as the tournament's
best player.

FeedBack Bookmark

https://www.aspiration.ai/LAW/sis/Solution.jsp?qsetId=Lw5e7noFooA=&qsetName=LST Mock 20 2021 (CLAT) 71/208


12/18/2020 Mock Analysis

Passage-02

Diego Armando Maradona, who is regarded as one of football's greatest players ever, died of heart failure at the
age of 60 as the world mourned him.

Maradona was born on Oct. 30, 1960, in the [1] capital Buenos Aires. Known for his pace, high technique, and
dribbling skills, Maradona was a multifaceted talent on the green eld as he could play as an attacking
mid elder and forward.

During his football career, Maradona played for [1]s Juniors, Boca Juniors, Newell's Old Boys, Spanish clubs
Barcelona and Sevilla, and Italian team Napoli.

Separately he was a regular player for the [1] national team, where he won the [2] FIFA World Cup in Mexico.

He also scored 34 goals in 91 international caps for [1]. Maradona played in four FIFA World Cups, including the
[2] World Cup in Mexico, where he captained [1] and led them to victory over West Germany in the nal, and won
the Golden Ball as the tournament's best player.

In the [2] World Cup quarter nal, he scored both goals in a 2–1 victory over England that entered football
history for two different reasons. The rst goal was an unpenalized handling foul known as the "[3]", while the
second goal followed a 60 m (66 yd) dribble past ve England players, voted "Goal of the Century" by FIFA.com
voters in 2002. He made his full international debut at age 16, against Hungary, on 27 February 1977.

Q.39 [30465398]
In the [2] World Cup quarter nal, Diego Armando Maradona scored both goals in a 2–1 victory over England
that entered football history for two different reasons. The rst goal was an unpenalized handling foul known as
the "[3]". Which of the following has been redacted with [3] in the passage above?

a Goal of God

b Miracle Hand of God

c Hand of God

d Golden Hand and Boot of God

FeedBack Bookmark  Answer key/Solution

https://www.aspiration.ai/LAW/sis/Solution.jsp?qsetId=Lw5e7noFooA=&qsetName=LST Mock 20 2021 (CLAT) 72/208


12/18/2020 Mock Analysis

Passage-02

Diego Armando Maradona, who is regarded as one of football's greatest players ever, died of heart failure at the
age of 60 as the world mourned him.

Maradona was born on Oct. 30, 1960, in the [1] capital Buenos Aires. Known for his pace, high technique, and
dribbling skills, Maradona was a multifaceted talent on the green eld as he could play as an attacking
mid elder and forward.

During his football career, Maradona played for [1]s Juniors, Boca Juniors, Newell's Old Boys, Spanish clubs
Barcelona and Sevilla, and Italian team Napoli.

Separately he was a regular player for the [1] national team, where he won the [2] FIFA World Cup in Mexico.

He also scored 34 goals in 91 international caps for [1]. Maradona played in four FIFA World Cups, including the
[2] World Cup in Mexico, where he captained [1] and led them to victory over West Germany in the nal, and won
the Golden Ball as the tournament's best player.

In the [2] World Cup quarter nal, he scored both goals in a 2–1 victory over England that entered football
history for two different reasons. The rst goal was an unpenalized handling foul known as the "[3]", while the
second goal followed a 60 m (66 yd) dribble past ve England players, voted "Goal of the Century" by FIFA.com
voters in 2002. He made his full international debut at age 16, against Hungary, on 27 February 1977.

Q.40 [30465398]
The 2022 FIFA World Cup is scheduled to take place in ___________from 21 November to 18 December 2022.

a Qatar

b Japan

c France

d Saudi Arabia

Solution:
 Answer key/Solution
Correct Answer : a
Your Answer : a
The 2022 FIFA World Cup is scheduled to be the 22nd edition of the FIFA World
Cup, the quadrennial international men's association football championship contested by the national teams of
the member associations of FIFA. It is scheduled to take place in Qatar from 21 November to 18 December
2022. This will be the rst World Cup ever to be held in the Arab world and the rst in a Muslim-majority
country. This will be the second World Cup held entirely in Asia after the 2002 tournament in South Korea and
Japan.

FeedBack Bookmark

https://www.aspiration.ai/LAW/sis/Solution.jsp?qsetId=Lw5e7noFooA=&qsetName=LST Mock 20 2021 (CLAT) 73/208


12/18/2020 Mock Analysis

Passage-02

Diego Armando Maradona, who is regarded as one of football's greatest players ever, died of heart failure at the
age of 60 as the world mourned him.

Maradona was born on Oct. 30, 1960, in the [1] capital Buenos Aires. Known for his pace, high technique, and
dribbling skills, Maradona was a multifaceted talent on the green eld as he could play as an attacking
mid elder and forward.

During his football career, Maradona played for [1]s Juniors, Boca Juniors, Newell's Old Boys, Spanish clubs
Barcelona and Sevilla, and Italian team Napoli.

Separately he was a regular player for the [1] national team, where he won the [2] FIFA World Cup in Mexico.

He also scored 34 goals in 91 international caps for [1]. Maradona played in four FIFA World Cups, including the
[2] World Cup in Mexico, where he captained [1] and led them to victory over West Germany in the nal, and won
the Golden Ball as the tournament's best player.

In the [2] World Cup quarter nal, he scored both goals in a 2–1 victory over England that entered football
history for two different reasons. The rst goal was an unpenalized handling foul known as the "[3]", while the
second goal followed a 60 m (66 yd) dribble past ve England players, voted "Goal of the Century" by FIFA.com
voters in 2002. He made his full international debut at age 16, against Hungary, on 27 February 1977.

Q.41 [30465398]
France won the 2018 FIFA World Cup by defeating which of the following countries?

a Croatia

b Argentina

c Russia

d Spain

https://www.aspiration.ai/LAW/sis/Solution.jsp?qsetId=Lw5e7noFooA=&qsetName=LST Mock 20 2021 (CLAT) 74/208


12/18/2020 Mock Analysis

Solution:
 Answer key/Solution
Correct Answer : a
Your Answer : a
The 2018 FIFA World Cup was an international football tournament contested by
men's national teams and took place between 14 June and 15 July 2018 in Russia. It was the 21st FIFA World
Cup, a worldwide football tournament held once every four years. In the nal, France played Croatia on 15 July
at the Luzhniki Stadium in Moscow. France won the match 4–2 to claim their second World Cup. Zabivaka was
the o cial mascot for the event —an anthropomorphic wolf dressed in the colours of the Russian national
team.
FeedBack Bookmark

Passage-03

The US formally exited the Paris Climate Agreement amid election uncertainty, three years after President
Donald Trump announced his intent to remove the country from participating in the landmark global pact to
reduce greenhouse gas emissions.

President Trump originally announced his intention to withdraw from the agreement in 2017 and formally
noti ed the United Nations last year. The US exited the pact after a mandatory year-long waiting period ended.

The historic accord seeks to limit global warming to less than [1] Celsius; the value that climate scientists have
determined will have disastrous consequences if exceeded.

Mr Trump has repeatedly criticised the agreement as economically detrimental and claimed it could cost the
country 2.5 million jobs by 2025. He also said it gave other major emitters, like China and India, a free pass.

The US is the only country to withdraw from the global pact. It can still attend negotiations and give opinions,
but is relegated to observer status.

Mr Trump stated that he intended to renegotiate the details of the United States’ membership within the Paris
Agreement that can better protect US workers in industries like coal, paper, and steel. The US is the second
leading producer of all carbon dioxide emissions globally, behind [2].

A possible second Trump term would make clear that an international effort to slow the Earth’s warming will not
include the US government. Democratic presidential candidate Joe Biden, meanwhile, has vowed to rejoin the
Paris accord as soon as he is inaugurated and to make the US a global leader on climate action.

Republicans have criticised Mr Biden’s climate plan as being too expensive, with Vice President Mike Pence
describing it as “a USD 2 trillion version of the Green New Deal” during his debate with Democratic rival Senator
Kamala Harris last month.

Q.42 [30465398]
The landmark Paris deal struck in 2015 aims to limit global warming to well below [1] degree Celsius above pre-
industrial temperatures. Which of the following has been redacted with [1] in the passage above?

https://www.aspiration.ai/LAW/sis/Solution.jsp?qsetId=Lw5e7noFooA=&qsetName=LST Mock 20 2021 (CLAT) 75/208


12/18/2020 Mock Analysis

a 1.5 degrees Celsius

b 2 degrees Celsius

c 2.5 degrees Celsius

d 3 degrees Celsius

Solution:
 Answer key/Solution
Correct Answer : b
Your Answer : b
The landmark Paris deal, struck in 2015, aims to limit global warming to “well
below” 2 °C above pre-industrial temperatures. The agreement came into force on November 4, 2016 and 189
countries have adopted it within the United Nations Framework Convention on Climate Change (UNFCCC),
dealing with greenhouse-gas-emissions mitigation, adaptation, and nance.

FeedBack Bookmark

https://www.aspiration.ai/LAW/sis/Solution.jsp?qsetId=Lw5e7noFooA=&qsetName=LST Mock 20 2021 (CLAT) 76/208


12/18/2020 Mock Analysis

Passage-03

The US formally exited the Paris Climate Agreement amid election uncertainty, three years after President
Donald Trump announced his intent to remove the country from participating in the landmark global pact to
reduce greenhouse gas emissions.

President Trump originally announced his intention to withdraw from the agreement in 2017 and formally
noti ed the United Nations last year. The US exited the pact after a mandatory year-long waiting period ended.

The historic accord seeks to limit global warming to less than [1] Celsius; the value that climate scientists have
determined will have disastrous consequences if exceeded.

Mr Trump has repeatedly criticised the agreement as economically detrimental and claimed it could cost the
country 2.5 million jobs by 2025. He also said it gave other major emitters, like China and India, a free pass.

The US is the only country to withdraw from the global pact. It can still attend negotiations and give opinions,
but is relegated to observer status.

Mr Trump stated that he intended to renegotiate the details of the United States’ membership within the Paris
Agreement that can better protect US workers in industries like coal, paper, and steel. The US is the second
leading producer of all carbon dioxide emissions globally, behind [2].

A possible second Trump term would make clear that an international effort to slow the Earth’s warming will not
include the US government. Democratic presidential candidate Joe Biden, meanwhile, has vowed to rejoin the
Paris accord as soon as he is inaugurated and to make the US a global leader on climate action.

Republicans have criticised Mr Biden’s climate plan as being too expensive, with Vice President Mike Pence
describing it as “a USD 2 trillion version of the Green New Deal” during his debate with Democratic rival Senator
Kamala Harris last month.

Q.43 [30465398]
The US is the second leading producer of all carbon dioxide emissions globally, behind [2]. Which of the
following countries has been redacted with [2] in the passage above?

a Russia

b China

c India

d The United Kingdom

https://www.aspiration.ai/LAW/sis/Solution.jsp?qsetId=Lw5e7noFooA=&qsetName=LST Mock 20 2021 (CLAT) 77/208


12/18/2020 Mock Analysis

Solution:
 Answer key/Solution
Correct Answer : b
Your Answer : b
The US is the second leading producer of all carbon dioxide emissions globally,
behind China.

FeedBack Bookmark

Passage-03

The US formally exited the Paris Climate Agreement amid election uncertainty, three years after President
Donald Trump announced his intent to remove the country from participating in the landmark global pact to
reduce greenhouse gas emissions.

President Trump originally announced his intention to withdraw from the agreement in 2017 and formally
noti ed the United Nations last year. The US exited the pact after a mandatory year-long waiting period ended.

The historic accord seeks to limit global warming to less than [1] Celsius; the value that climate scientists have
determined will have disastrous consequences if exceeded.

Mr Trump has repeatedly criticised the agreement as economically detrimental and claimed it could cost the
country 2.5 million jobs by 2025. He also said it gave other major emitters, like China and India, a free pass.

The US is the only country to withdraw from the global pact. It can still attend negotiations and give opinions,
but is relegated to observer status.

Mr Trump stated that he intended to renegotiate the details of the United States’ membership within the Paris
Agreement that can better protect US workers in industries like coal, paper, and steel. The US is the second
leading producer of all carbon dioxide emissions globally, behind [2].

A possible second Trump term would make clear that an international effort to slow the Earth’s warming will not
include the US government. Democratic presidential candidate Joe Biden, meanwhile, has vowed to rejoin the
Paris accord as soon as he is inaugurated and to make the US a global leader on climate action.

Republicans have criticised Mr Biden’s climate plan as being too expensive, with Vice President Mike Pence
describing it as “a USD 2 trillion version of the Green New Deal” during his debate with Democratic rival Senator
Kamala Harris last month.

Q.44 [30465398]
Article ___________ of the Paris Agreement allows countries to leave the Paris Agreement and lays down the
process for leaving.

a Article 2

b Article 20

https://www.aspiration.ai/LAW/sis/Solution.jsp?qsetId=Lw5e7noFooA=&qsetName=LST Mock 20 2021 (CLAT) 78/208


12/18/2020 Mock Analysis

c Article 25

d Article 28

Solution:
 Answer key/Solution
Correct Answer : d
Your Answer : d
Article 28 of the Paris Agreement allows countries to leave the Paris Agreement
and lays down the process for leaving. A country can only give a notice for leaving at least three years after
the Paris Agreement came into force.

FeedBack Bookmark

Passage-03

The US formally exited the Paris Climate Agreement amid election uncertainty, three years after President
Donald Trump announced his intent to remove the country from participating in the landmark global pact to
reduce greenhouse gas emissions.

President Trump originally announced his intention to withdraw from the agreement in 2017 and formally
noti ed the United Nations last year. The US exited the pact after a mandatory year-long waiting period ended.

The historic accord seeks to limit global warming to less than [1] Celsius; the value that climate scientists have
determined will have disastrous consequences if exceeded.

Mr Trump has repeatedly criticised the agreement as economically detrimental and claimed it could cost the
country 2.5 million jobs by 2025. He also said it gave other major emitters, like China and India, a free pass.

The US is the only country to withdraw from the global pact. It can still attend negotiations and give opinions,
but is relegated to observer status.

Mr Trump stated that he intended to renegotiate the details of the United States’ membership within the Paris
Agreement that can better protect US workers in industries like coal, paper, and steel. The US is the second
leading producer of all carbon dioxide emissions globally, behind [2].

A possible second Trump term would make clear that an international effort to slow the Earth’s warming will not
include the US government. Democratic presidential candidate Joe Biden, meanwhile, has vowed to rejoin the
Paris accord as soon as he is inaugurated and to make the US a global leader on climate action.

Republicans have criticised Mr Biden’s climate plan as being too expensive, with Vice President Mike Pence
describing it as “a USD 2 trillion version of the Green New Deal” during his debate with Democratic rival Senator
Kamala Harris last month.

https://www.aspiration.ai/LAW/sis/Solution.jsp?qsetId=Lw5e7noFooA=&qsetName=LST Mock 20 2021 (CLAT) 79/208


12/18/2020 Mock Analysis

Q.45 [30465398]
Which of the following statements is not true regarding the Paris agreement?

a All remaining parties to the Paris agreement must submit their new 2030 targets before the next major
United Nations climate meeting, set to take place in Glasgow, UK, in November 2021.

b India announced that by 2030, it would increase the share of non-fossil fuels in the installed energy
capacity to 40 per cent.

c As part of its commitment to the Paris Agreement, the United States had promised to reduce its emissions
by 26 per cent to 28 per cent by the year 2025 from 2005 levels is the one of the reason for leaving agreement.

d The Paris agreement came into force on November 4, 2017.

Solution:
 Answer key/Solution
Correct Answer : d
Your Answer : d
All remaining parties to the agreement must submit their new 2030 targets before
the next major United Nations climate meeting, set to take place in Glasgow, UK, in November 2021. In
October 2015, ahead of the annual Conference of Parties in Paris, India had announced its ambitious INDCs. It
pledged a reduction in its emissions intensity of its Gross Domestic Product by 33 to 35 per cent by 2030,
below 2005 levels. Crucially, it announced that by 2030, it would increase the share of non-fossil fuels in the
installed energy capacity to 40 per cent. As part of its commitment to the Paris Agreement, the United States
had promised to reduce its emissions by 26 per cent to 28 per cent by the year 2025 from 2005 levels. The
Paris agreement came into force on November 4, 2016.

FeedBack Bookmark

https://www.aspiration.ai/LAW/sis/Solution.jsp?qsetId=Lw5e7noFooA=&qsetName=LST Mock 20 2021 (CLAT) 80/208


12/18/2020 Mock Analysis

Passage-03

The US formally exited the Paris Climate Agreement amid election uncertainty, three years after President
Donald Trump announced his intent to remove the country from participating in the landmark global pact to
reduce greenhouse gas emissions.

President Trump originally announced his intention to withdraw from the agreement in 2017 and formally
noti ed the United Nations last year. The US exited the pact after a mandatory year-long waiting period ended.

The historic accord seeks to limit global warming to less than [1] Celsius; the value that climate scientists have
determined will have disastrous consequences if exceeded.

Mr Trump has repeatedly criticised the agreement as economically detrimental and claimed it could cost the
country 2.5 million jobs by 2025. He also said it gave other major emitters, like China and India, a free pass.

The US is the only country to withdraw from the global pact. It can still attend negotiations and give opinions,
but is relegated to observer status.

Mr Trump stated that he intended to renegotiate the details of the United States’ membership within the Paris
Agreement that can better protect US workers in industries like coal, paper, and steel. The US is the second
leading producer of all carbon dioxide emissions globally, behind [2].

A possible second Trump term would make clear that an international effort to slow the Earth’s warming will not
include the US government. Democratic presidential candidate Joe Biden, meanwhile, has vowed to rejoin the
Paris accord as soon as he is inaugurated and to make the US a global leader on climate action.

Republicans have criticised Mr Biden’s climate plan as being too expensive, with Vice President Mike Pence
describing it as “a USD 2 trillion version of the Green New Deal” during his debate with Democratic rival Senator
Kamala Harris last month.

Q.46 [30465398]
What did India commit to achieve under the Paris Climate Agreement (2015) as outlined in its Intended
Nationally Determined Contributions (INDC) report submitted to the UNFCCC?

a Reduce the pesticide intensity of its GDP by 33 to 35 percent by 2030 from 2005 levels

b Reduce the emissions intensity of its GDP by 33 to 35 percent by 2030 from 2005 levels

c Reduce the material intensity of its GDP by 33 to 35 percent by 2030 from 2005 levels

d Reduce the energy intensity of its GDP by 33 to 35 percent by 2030 from 2005 levels

https://www.aspiration.ai/LAW/sis/Solution.jsp?qsetId=Lw5e7noFooA=&qsetName=LST Mock 20 2021 (CLAT) 81/208


12/18/2020 Mock Analysis

Solution:
 Answer key/Solution
Correct Answer : b
Your Answer : b
India submitted its INDC to the UNFCCC in October 2015, committing to cut the
emissions intensity of GDP by 33–35% by 2030 from 2005 levels. On its submission, India wrote that it needs
"at least USD 2.5 trillion" to achieve its 2015-2030 goals, and that its "international climate nance needs" will
be the difference over "what can be made available from domestic sources."

FeedBack Bookmark

Passage-04

India’s economy rebounded sharply in the wake of the reopening from lockdowns, slowing the pace of its
contraction to 8.6% in the second quarter, a ‘nowcast’ in the RBI’s monthly bulletin showed.

Still, the estimate implies that India is likely to have entered a technical recession in the rst half of 2020-21 for
the rst time in its history with two successive quarters of GDP contraction, wrote an RBI o cial who authored
an article in the bulletin.

An Economic Activity Index, which tracks real-time data from 27 monthly indicators, showed the economy had
rebounded sharply from May/June, with industry normalising faster than contact-intensive service sectors,
pointing to a short-lived contraction.

“The index nowcasts GDP growth at (-) 8.6% in Q2, implying that India is likely to have entered a technical
recession in the rst half,” Pankaj Kumar of the RBI’s Monetary Policy Department wrote in the article titled ‘An
Economic Activity Index for India’.

“At a time when global economic activity is besieged by the outbreak of the second wave of COVID-19... data for
the month of October 2020 have brightened the near-term outlook for the Indian economy and stirred up
consumer and business con dence,” the RBI noted. “There are, however, formidable downside risks that
confront the path of recovery,” it added.

Observing that “the unrelenting pressure of in ation” was the foremost risk, the RBI warned that a hit to external
demand from the second wave and nancially stressed households and companies could also undermine the
recovery.

Q.47 [30465398]
If there is lack of money supply in comparison to the supply of the goods and services, then the possible
consequence would be:

a In ation

b De ation

c Hyperin ation

https://www.aspiration.ai/LAW/sis/Solution.jsp?qsetId=Lw5e7noFooA=&qsetName=LST Mock 20 2021 (CLAT) 82/208


12/18/2020 Mock Analysis

d Devaluation

Solution:
 Answer key/Solution
Correct Answer : b
Your Answer : a
De ation is generally the decline in the prices for goods and services that occur
when the rate of in ation falls below 0%. De ation will take place naturally, if and when the money supply of
an economy is limited. De ation in an economy indicates deteriorating conditions. De ation is normally linked
with signi cant unemployment and low productivity levels of good and services. The term “De ation” is often
mistaken with “disin ation.” While de ation refers to a decrease in the prices of goods and services in an
economy, disin ation is when in ation increases at a slower rate.

FeedBack Bookmark

Passage-04

India’s economy rebounded sharply in the wake of the reopening from lockdowns, slowing the pace of its
contraction to 8.6% in the second quarter, a ‘nowcast’ in the RBI’s monthly bulletin showed.

Still, the estimate implies that India is likely to have entered a technical recession in the rst half of 2020-21 for
the rst time in its history with two successive quarters of GDP contraction, wrote an RBI o cial who authored
an article in the bulletin.

An Economic Activity Index, which tracks real-time data from 27 monthly indicators, showed the economy had
rebounded sharply from May/June, with industry normalising faster than contact-intensive service sectors,
pointing to a short-lived contraction.

“The index nowcasts GDP growth at (-) 8.6% in Q2, implying that India is likely to have entered a technical
recession in the rst half,” Pankaj Kumar of the RBI’s Monetary Policy Department wrote in the article titled ‘An
Economic Activity Index for India’.

“At a time when global economic activity is besieged by the outbreak of the second wave of COVID-19... data for
the month of October 2020 have brightened the near-term outlook for the Indian economy and stirred up
consumer and business con dence,” the RBI noted. “There are, however, formidable downside risks that
confront the path of recovery,” it added.

Observing that “the unrelenting pressure of in ation” was the foremost risk, the RBI warned that a hit to external
demand from the second wave and nancially stressed households and companies could also undermine the
recovery.

Q.48 [30465398]
In ation is caused by multiple factors, which of the following is not the reason for in ation?

a Money Supply

https://www.aspiration.ai/LAW/sis/Solution.jsp?qsetId=Lw5e7noFooA=&qsetName=LST Mock 20 2021 (CLAT) 83/208


12/18/2020 Mock Analysis

b National Debt

c Demand-Pull Effect

d Stagnation

Solution:
 Answer key/Solution
Correct Answer : d
In ation is caused by multiple factors, they are 1. Money Supply: Excess currency
(money) supply in an economy is one of the primary cause of in ation. 2. National
Debt: In a situation where a country’s debt increases, the respective country is left with two options: Taxes can
be raised internally, Additional money can be printed to pay off the debt 3. Demand-Pull Effect: The demand-
pull effect states that in a growing economy as wages increase within an economy, people will have more
money to spend on goods and services. The increase in demand for goods and services will result in
companies to raise prices that consumers will bear in order to balance supply and demand. 4. Cost-Push
Effect: This theory states that when companies face increased input costs on raw materials and wages for
manufacturing consumer goods, they will preserve their pro tability by passing the increased production cost
to the end consumer in the form of increased prices.

FeedBack Bookmark

Passage-04

India’s economy rebounded sharply in the wake of the reopening from lockdowns, slowing the pace of its
contraction to 8.6% in the second quarter, a ‘nowcast’ in the RBI’s monthly bulletin showed.

Still, the estimate implies that India is likely to have entered a technical recession in the rst half of 2020-21 for
the rst time in its history with two successive quarters of GDP contraction, wrote an RBI o cial who authored
an article in the bulletin.

An Economic Activity Index, which tracks real-time data from 27 monthly indicators, showed the economy had
rebounded sharply from May/June, with industry normalising faster than contact-intensive service sectors,
pointing to a short-lived contraction.

“The index nowcasts GDP growth at (-) 8.6% in Q2, implying that India is likely to have entered a technical
recession in the rst half,” Pankaj Kumar of the RBI’s Monetary Policy Department wrote in the article titled ‘An
Economic Activity Index for India’.

“At a time when global economic activity is besieged by the outbreak of the second wave of COVID-19... data for
the month of October 2020 have brightened the near-term outlook for the Indian economy and stirred up
consumer and business con dence,” the RBI noted. “There are, however, formidable downside risks that
confront the path of recovery,” it added.

Observing that “the unrelenting pressure of in ation” was the foremost risk, the RBI warned that a hit to external
demand from the second wave and nancially stressed households and companies could also undermine the
recovery.

https://www.aspiration.ai/LAW/sis/Solution.jsp?qsetId=Lw5e7noFooA=&qsetName=LST Mock 20 2021 (CLAT) 84/208


12/18/2020 Mock Analysis

Q.49 [30465398]
If the Cash Reserve Ratio (CRR) is reduced by the RBI, then it would lead to ___________.

a Decrease in in ation

b Increase in in ation

c No effect in in ation

d No effect of banks money supply

Solution:
 Answer key/Solution
Correct Answer : b
Your Answer : a
If the Cash Reserve Ratio (CRR) is reduced by the RBI, then it would lead to
increase in in ation because it allows banks to lend more money in the market and it will lead to increase in
the money supply in the market and automatically it leads to increase in in ation.

FeedBack Bookmark

https://www.aspiration.ai/LAW/sis/Solution.jsp?qsetId=Lw5e7noFooA=&qsetName=LST Mock 20 2021 (CLAT) 85/208


12/18/2020 Mock Analysis

Passage-04

India’s economy rebounded sharply in the wake of the reopening from lockdowns, slowing the pace of its
contraction to 8.6% in the second quarter, a ‘nowcast’ in the RBI’s monthly bulletin showed.

Still, the estimate implies that India is likely to have entered a technical recession in the rst half of 2020-21 for
the rst time in its history with two successive quarters of GDP contraction, wrote an RBI o cial who authored
an article in the bulletin.

An Economic Activity Index, which tracks real-time data from 27 monthly indicators, showed the economy had
rebounded sharply from May/June, with industry normalising faster than contact-intensive service sectors,
pointing to a short-lived contraction.

“The index nowcasts GDP growth at (-) 8.6% in Q2, implying that India is likely to have entered a technical
recession in the rst half,” Pankaj Kumar of the RBI’s Monetary Policy Department wrote in the article titled ‘An
Economic Activity Index for India’.

“At a time when global economic activity is besieged by the outbreak of the second wave of COVID-19... data for
the month of October 2020 have brightened the near-term outlook for the Indian economy and stirred up
consumer and business con dence,” the RBI noted. “There are, however, formidable downside risks that
confront the path of recovery,” it added.

Observing that “the unrelenting pressure of in ation” was the foremost risk, the RBI warned that a hit to external
demand from the second wave and nancially stressed households and companies could also undermine the
recovery.

Q.50 [30465398]
The situation with increasing unemployment and in ation is termed as:

a Hyperin ation

b Stag ation

c Re ation

d De ation

Solution:
 Answer key/Solution
Correct Answer : b
Stag ation is the combination of high in ation, high unemployment and sluggish
economic growth.

FeedBack Bookmark

https://www.aspiration.ai/LAW/sis/Solution.jsp?qsetId=Lw5e7noFooA=&qsetName=LST Mock 20 2021 (CLAT) 86/208


12/18/2020 Mock Analysis

Passage-04

India’s economy rebounded sharply in the wake of the reopening from lockdowns, slowing the pace of its
contraction to 8.6% in the second quarter, a ‘nowcast’ in the RBI’s monthly bulletin showed.

Still, the estimate implies that India is likely to have entered a technical recession in the rst half of 2020-21 for
the rst time in its history with two successive quarters of GDP contraction, wrote an RBI o cial who authored
an article in the bulletin.

An Economic Activity Index, which tracks real-time data from 27 monthly indicators, showed the economy had
rebounded sharply from May/June, with industry normalising faster than contact-intensive service sectors,
pointing to a short-lived contraction.

“The index nowcasts GDP growth at (-) 8.6% in Q2, implying that India is likely to have entered a technical
recession in the rst half,” Pankaj Kumar of the RBI’s Monetary Policy Department wrote in the article titled ‘An
Economic Activity Index for India’.

“At a time when global economic activity is besieged by the outbreak of the second wave of COVID-19... data for
the month of October 2020 have brightened the near-term outlook for the Indian economy and stirred up
consumer and business con dence,” the RBI noted. “There are, however, formidable downside risks that
confront the path of recovery,” it added.

Observing that “the unrelenting pressure of in ation” was the foremost risk, the RBI warned that a hit to external
demand from the second wave and nancially stressed households and companies could also undermine the
recovery.

Q.51 [30465398]
Which of the following released the ‘Financial Stability Report (FSR)’ for the month of July 2020?

a Reserve Bank of India

b International Monetary Fund

c Ministry of Finance

d NITI Aayog

https://www.aspiration.ai/LAW/sis/Solution.jsp?qsetId=Lw5e7noFooA=&qsetName=LST Mock 20 2021 (CLAT) 87/208


12/18/2020 Mock Analysis

Solution:
 Answer key/Solution
Correct Answer : a
The Reserve Bank of India (RBI) released its Financial Stability Report (FSR) for
the month of July 2020. The FSR re ects the collective assessment of the Sub-
Committee of the Financial Stability and Development Council (FSDC - headed by the Governor of RBI) on
risks to nancial stability and the resilience of the nancial system. The RBI warned that the Gross Non-
performing Assets (GNPA) ratio of all Scheduled Commercial Banks (SCBs) may increase from 8.5% in March
2020 to 12.5% by March 2021. The GNPA ratio may also worsen to as high as 14.7% by the end of the current
nancial year, if the adverse economic impact of the Covid-19 pandemic would be ‘very severe’. The Covid-19
lockdown had a signi cant impact on all industrial activities in the economy resulting in major income loss.
This has impacted their loan repayment ability. This may lead to Gross Domestic Product (GDP) contraction by
8.9% in 2020-21.
FeedBack Bookmark

https://www.aspiration.ai/LAW/sis/Solution.jsp?qsetId=Lw5e7noFooA=&qsetName=LST Mock 20 2021 (CLAT) 88/208


12/18/2020 Mock Analysis

Passage-05

The [1] government issued an order withdrawing general consent to the Central Bureau of Investigation (CBI) to
investigate cases without prior permission.

According to sources, the move implies that the CBI will not have general consent to exercise its powers and
jurisdiction in the state and will have to get the state government’s permission to probe any matter.

According to [1] CMO sources, the state government was unhappy with CBI intervention in Life Mission Housing
project an initiative of the [1] government to provide shelter and housing to the homeless.

The project was stalled temporarily following the probe after which the government approached the high court.
The high court stayed the CBI probe. The decision was taken in a cabinet meeting held.

With this, [1] has joined the list of non-BJP-ruled states West Bengal, Rajasthan and Maharashtra in withdrawing
general consent to CBI.

In October 2020, the Maharashtra government had issued on order withdrawing consent accorded to the
members of the Delhi Special Police Establishment to exercise the powers and jurisdiction under an act in the
state.

The move implied that the CBI will not have general consent, accorded by the Maharashtra government by an
order on February 22, 1989, to exercise its powers and jurisdiction in the state and will have to get the state
governments permission to probe any matter.

The governments of Maharashtra, West Bengal, Rajasthan and Chhattisgarh recently withdrew the general
consent to the Central Bureau of Investigation. The consent is akin to a blanket nod for the agency to probe
scheduled offences speci ed in the [2].

The CBI also lacks a "general consent" from Mizoram. Unlike the National Investigation Agency (NIA), which has
countrywide jurisdiction to take over any case related to terrorism, the CBI requires the consent of the state
government concerned under Section [3] of the DPSE Act, the law that governs the agency's functioning.

Q.52 [30465398]
In November 2020, which of the following states has issued an order withdrawing general consent to the Central
Bureau of Investigation (CBI) to investigate cases without prior permission whose name has been redacted with
[1] in the passage above?

a Tripura

b West Bengal

c Kerala

d Odisha

https://www.aspiration.ai/LAW/sis/Solution.jsp?qsetId=Lw5e7noFooA=&qsetName=LST Mock 20 2021 (CLAT) 89/208


12/18/2020 Mock Analysis

Solution:
 Answer key/Solution
Correct Answer : c
Your Answer : c
The Kerala government issued an order withdrawing general consent to the
Central Bureau of Investigation (CBI) to investigate cases without prior permission. According to sources, the
move implies that the CBI will not have general consent to exercise its powers and jurisdiction in the state and
will have to get the state government’s permission to probe any matter. According to Kerala CMO sources, the
state government was unhappy with CBI intervention in Life Mission Housing project an initiative of the Kerala
government to provide shelter and housing to the homeless.
FeedBack Bookmark

Passage-05

The [1] government issued an order withdrawing general consent to the Central Bureau of Investigation (CBI) to
investigate cases without prior permission.

According to sources, the move implies that the CBI will not have general consent to exercise its powers and
jurisdiction in the state and will have to get the state government’s permission to probe any matter.

According to [1] CMO sources, the state government was unhappy with CBI intervention in Life Mission Housing
project an initiative of the [1] government to provide shelter and housing to the homeless.

The project was stalled temporarily following the probe after which the government approached the high court.
The high court stayed the CBI probe. The decision was taken in a cabinet meeting held.

With this, [1] has joined the list of non-BJP-ruled states West Bengal, Rajasthan and Maharashtra in withdrawing
general consent to CBI.

In October 2020, the Maharashtra government had issued on order withdrawing consent accorded to the
members of the Delhi Special Police Establishment to exercise the powers and jurisdiction under an act in the
state.

The move implied that the CBI will not have general consent, accorded by the Maharashtra government by an
order on February 22, 1989, to exercise its powers and jurisdiction in the state and will have to get the state
governments permission to probe any matter.

The governments of Maharashtra, West Bengal, Rajasthan and Chhattisgarh recently withdrew the general
consent to the Central Bureau of Investigation. The consent is akin to a blanket nod for the agency to probe
scheduled offences speci ed in the [2].

The CBI also lacks a "general consent" from Mizoram. Unlike the National Investigation Agency (NIA), which has
countrywide jurisdiction to take over any case related to terrorism, the CBI requires the consent of the state
government concerned under Section [3] of the DPSE Act, the law that governs the agency's functioning.

https://www.aspiration.ai/LAW/sis/Solution.jsp?qsetId=Lw5e7noFooA=&qsetName=LST Mock 20 2021 (CLAT) 90/208


12/18/2020 Mock Analysis

Q.53 [30465398]
The general consent is akin to a blanket nod for the CBI to probe scheduled offences speci ed in the [2]. Which
of the following acts has been redacted with [2] in the passage above?

a Delhi Special Police Establishment Act (DPSE) Act, 1940

b Delhi Special Police Establishment Act (DPSE) Act, 1946

c Delhi Special Police Establishment Act (DPSE) Act, 1956

d Delhi Special Police Establishment Act (DPSE) Act, 1966

Solution:
 Answer key/Solution
Correct Answer : b
Your Answer : b
The governments of Maharashtra, West Bengal, Rajasthan and Chhattisgarh
recently withdrew the general consent to the Central Bureau of Investigation. The consent is akin to a blanket
nod for the agency to probe scheduled offences speci ed in the Delhi Special Police Establishment Act (DPSE)
Act, 1946.
FeedBack Bookmark

https://www.aspiration.ai/LAW/sis/Solution.jsp?qsetId=Lw5e7noFooA=&qsetName=LST Mock 20 2021 (CLAT) 91/208


12/18/2020 Mock Analysis

Passage-05

The [1] government issued an order withdrawing general consent to the Central Bureau of Investigation (CBI) to
investigate cases without prior permission.

According to sources, the move implies that the CBI will not have general consent to exercise its powers and
jurisdiction in the state and will have to get the state government’s permission to probe any matter.

According to [1] CMO sources, the state government was unhappy with CBI intervention in Life Mission Housing
project an initiative of the [1] government to provide shelter and housing to the homeless.

The project was stalled temporarily following the probe after which the government approached the high court.
The high court stayed the CBI probe. The decision was taken in a cabinet meeting held.

With this, [1] has joined the list of non-BJP-ruled states West Bengal, Rajasthan and Maharashtra in withdrawing
general consent to CBI.

In October 2020, the Maharashtra government had issued on order withdrawing consent accorded to the
members of the Delhi Special Police Establishment to exercise the powers and jurisdiction under an act in the
state.

The move implied that the CBI will not have general consent, accorded by the Maharashtra government by an
order on February 22, 1989, to exercise its powers and jurisdiction in the state and will have to get the state
governments permission to probe any matter.

The governments of Maharashtra, West Bengal, Rajasthan and Chhattisgarh recently withdrew the general
consent to the Central Bureau of Investigation. The consent is akin to a blanket nod for the agency to probe
scheduled offences speci ed in the [2].

The CBI also lacks a "general consent" from Mizoram. Unlike the National Investigation Agency (NIA), which has
countrywide jurisdiction to take over any case related to terrorism, the CBI requires the consent of the state
government concerned under Section [3] of the DPSE Act, the law that governs the agency's functioning.

Q.54 [30465398]
In exercise of power conferred by Section [3] of the act [2], the state governments can withdraw the general
consent accorded. Which of the following sections has been redacted with [3] in the passage above?

a Section 2

b Section 4

c Section 6

d Section 8

https://www.aspiration.ai/LAW/sis/Solution.jsp?qsetId=Lw5e7noFooA=&qsetName=LST Mock 20 2021 (CLAT) 92/208


12/18/2020 Mock Analysis

Solution:
 Answer key/Solution
Correct Answer : c
Your Answer : c
In exercise of power conferred by Section 6 of the Delhi Special Police
Establishment Act, 1946, the state governments can withdraw the general consent accorded.
FeedBack Bookmark

Passage-05

The [1] government issued an order withdrawing general consent to the Central Bureau of Investigation (CBI) to
investigate cases without prior permission.

According to sources, the move implies that the CBI will not have general consent to exercise its powers and
jurisdiction in the state and will have to get the state government’s permission to probe any matter.

According to [1] CMO sources, the state government was unhappy with CBI intervention in Life Mission Housing
project an initiative of the [1] government to provide shelter and housing to the homeless.

The project was stalled temporarily following the probe after which the government approached the high court.
The high court stayed the CBI probe. The decision was taken in a cabinet meeting held.

With this, [1] has joined the list of non-BJP-ruled states West Bengal, Rajasthan and Maharashtra in withdrawing
general consent to CBI.

In October 2020, the Maharashtra government had issued on order withdrawing consent accorded to the
members of the Delhi Special Police Establishment to exercise the powers and jurisdiction under an act in the
state.

The move implied that the CBI will not have general consent, accorded by the Maharashtra government by an
order on February 22, 1989, to exercise its powers and jurisdiction in the state and will have to get the state
governments permission to probe any matter.

The governments of Maharashtra, West Bengal, Rajasthan and Chhattisgarh recently withdrew the general
consent to the Central Bureau of Investigation. The consent is akin to a blanket nod for the agency to probe
scheduled offences speci ed in the [2].

The CBI also lacks a "general consent" from Mizoram. Unlike the National Investigation Agency (NIA), which has
countrywide jurisdiction to take over any case related to terrorism, the CBI requires the consent of the state
government concerned under Section [3] of the DPSE Act, the law that governs the agency's functioning.

Q.55 [30465398]
Which of the following statements is not true with regard to Central Bureau of Investigation (CBI)?

a CBI was set up as a non-statutory body to investigate bribery and governmental corruption.

https://www.aspiration.ai/LAW/sis/Solution.jsp?qsetId=Lw5e7noFooA=&qsetName=LST Mock 20 2021 (CLAT) 93/208


12/18/2020 Mock Analysis

b CBI is exempted from the provisions of the Right to Information Act.

c CBI is India's o cially designated single point of contact for liaison with the United Nations and FATF.

d Rishi Kumar Shukla is the current director of the CBI.

Solution:
 Answer key/Solution
Correct Answer : c
Your Answer : c
The Central Bureau of Investigation (CBI) is the premier investigating agency of
India. Operating under the jurisdiction of the Ministry of Home Affairs (India), Originally set up as a non-
statutory body to investigate bribery and governmental corruption, in 1965 it received expanded jurisdiction to
investigate breaches of central laws enforceable by the Government of India, multi-state organised crime,
multi-agency or international cases. The agency has been known to investigate several economic crimes,
special crimes, cases of corruption and other cases. CBI is exempted from the provisions of the Right to
Information Act. CBI is India's o cially designated single point of contact for liaison with the Interpol. The CBI
headquarter is located in CGO Complex, near Jawaharlal Nehru Stadium in New Delhi. Rishi Kumar Shukla is
the current director of the CBI.

FeedBack Bookmark

https://www.aspiration.ai/LAW/sis/Solution.jsp?qsetId=Lw5e7noFooA=&qsetName=LST Mock 20 2021 (CLAT) 94/208


12/18/2020 Mock Analysis

Passage-05

The [1] government issued an order withdrawing general consent to the Central Bureau of Investigation (CBI) to
investigate cases without prior permission.

According to sources, the move implies that the CBI will not have general consent to exercise its powers and
jurisdiction in the state and will have to get the state government’s permission to probe any matter.

According to [1] CMO sources, the state government was unhappy with CBI intervention in Life Mission Housing
project an initiative of the [1] government to provide shelter and housing to the homeless.

The project was stalled temporarily following the probe after which the government approached the high court.
The high court stayed the CBI probe. The decision was taken in a cabinet meeting held.

With this, [1] has joined the list of non-BJP-ruled states West Bengal, Rajasthan and Maharashtra in withdrawing
general consent to CBI.

In October 2020, the Maharashtra government had issued on order withdrawing consent accorded to the
members of the Delhi Special Police Establishment to exercise the powers and jurisdiction under an act in the
state.

The move implied that the CBI will not have general consent, accorded by the Maharashtra government by an
order on February 22, 1989, to exercise its powers and jurisdiction in the state and will have to get the state
governments permission to probe any matter.

The governments of Maharashtra, West Bengal, Rajasthan and Chhattisgarh recently withdrew the general
consent to the Central Bureau of Investigation. The consent is akin to a blanket nod for the agency to probe
scheduled offences speci ed in the [2].

The CBI also lacks a "general consent" from Mizoram. Unlike the National Investigation Agency (NIA), which has
countrywide jurisdiction to take over any case related to terrorism, the CBI requires the consent of the state
government concerned under Section [3] of the DPSE Act, the law that governs the agency's functioning.

Q.56 [30465398]
The Bureau of Investigation traces its origins to the Special Police Establishment, a Central Government Police
force, which was set up in 1941 by the Government of India to investigate bribery and corruption in transactions
with the___________.

a War and Supply Department of India

b Ministry of Finance

c Last Secretary of State for India, Earl of Listowel

d Last Viceroy of India, Lord Mountbatten

https://www.aspiration.ai/LAW/sis/Solution.jsp?qsetId=Lw5e7noFooA=&qsetName=LST Mock 20 2021 (CLAT) 95/208


12/18/2020 Mock Analysis

Solution:
 Answer key/Solution
Correct Answer : a
Your Answer : a
The Bureau of Investigation traces its origins to the Special Police Establishment,
a Central Government Police force, which was set up in 1941 by the Government of India to investigate bribery
and corruption in transactions with the War and Supply Department of India. It had its headquarters in Lahore.
The Superintendent of the SPE was Qurban Ali Khan, who later opted for Pakistan during the Partition of India.
The rst legal adviser of the War Department was Rai Sahib Karam Chand Jain. After the end of the war, there
was a continued need for a central governmental agency to investigate bribery and corruption by central-
government employees. Sahib Karam Chand Jain remained its legal advisor when the department was
transferred to the Home Department by the 1946 Delhi Special Police Establishment Act.
FeedBack Bookmark

Passage-06

The Nobel Peace Prize 2020 was awarded to the [1] for feeding millions of people from Yemen to North Korea,
with the coronavirus pandemic seen pushing millions more into hunger.

The [1] was honoured for “its efforts to combat hunger, for its contribution to bettering conditions for peace in
con ict-affected areas and for acting as a driving force in efforts to prevent the use of hunger as a weapon of
war and con ict,” Nobel committee chairwoman Berit Reiss-Andersen said on unveiling the winner in Oslo.

Whether delivering food by helicopter or on the back of an elephant or a camel, the [1] prides itself on being “the
leading humanitarian organisation” in a world where, by its own estimates, some 690 million people -- one in 11 -
- go to bed on an empty stomach.

“With this year's award, the Norwegian Nobel Committee wishes to turn the eyes of the world towards the
millions of people who suffer from or face the threat of hunger,” Reiss-Andersen said.

Founded in 1961, the UN organisation helped 97 million people last year, distributing 15 billion rations to people
in 88 countries last year. The numbers are dizzying but only a fraction of the total number in need.

Despite making progress over the past three decades, the UN's goal to eradicate hunger by 2030 appears out of
reach if current trends continue, according to experts.

In 75 years, the United Nations, its specialised agencies, related agencies, funds, programmes and staff were
awarded the prestigious Nobel Peace Prize [2] times, more than any other laureate.

The virus will also affect the Nobel Peace Prize ceremony on December 10 in Oslo, which has been scaled back
due to corona restrictions. The award consists of a gold medal, a diploma and a cheque for 10 million Swedish
kronor (950,000 euros, $1.1 million).

Q.57 [30465398]
The Nobel Peace Prize 2020 was awarded to the [1] for feeding millions of people from Yemen to North Korea.
Which of the following has been redacted with [1] in the passage above?

https://www.aspiration.ai/LAW/sis/Solution.jsp?qsetId=Lw5e7noFooA=&qsetName=LST Mock 20 2021 (CLAT) 96/208


12/18/2020 Mock Analysis

a World Food Programme

b United Nations Development Programme

c UNICEF

d Food and Agriculture Organization

Solution:
 Answer key/Solution
Correct Answer : a
Your Answer : a
The Nobel Peace Prize 2020 was awarded to the World Food Programme for
feeding millions of people from Yemen to North Korea, with the coronavirus pandemic seen pushing millions
more into hunger. The World Food Programme aims to eradicate hunger and malnutrition. It is the world’s
largest humanitarian agency. Every year, the programme feeds almost 80 million people in around 75
countries.
FeedBack Bookmark

https://www.aspiration.ai/LAW/sis/Solution.jsp?qsetId=Lw5e7noFooA=&qsetName=LST Mock 20 2021 (CLAT) 97/208


12/18/2020 Mock Analysis

Passage-06

The Nobel Peace Prize 2020 was awarded to the [1] for feeding millions of people from Yemen to North Korea,
with the coronavirus pandemic seen pushing millions more into hunger.

The [1] was honoured for “its efforts to combat hunger, for its contribution to bettering conditions for peace in
con ict-affected areas and for acting as a driving force in efforts to prevent the use of hunger as a weapon of
war and con ict,” Nobel committee chairwoman Berit Reiss-Andersen said on unveiling the winner in Oslo.

Whether delivering food by helicopter or on the back of an elephant or a camel, the [1] prides itself on being “the
leading humanitarian organisation” in a world where, by its own estimates, some 690 million people -- one in 11 -
- go to bed on an empty stomach.

“With this year's award, the Norwegian Nobel Committee wishes to turn the eyes of the world towards the
millions of people who suffer from or face the threat of hunger,” Reiss-Andersen said.

Founded in 1961, the UN organisation helped 97 million people last year, distributing 15 billion rations to people
in 88 countries last year. The numbers are dizzying but only a fraction of the total number in need.

Despite making progress over the past three decades, the UN's goal to eradicate hunger by 2030 appears out of
reach if current trends continue, according to experts.

In 75 years, the United Nations, its specialised agencies, related agencies, funds, programmes and staff were
awarded the prestigious Nobel Peace Prize [2] times, more than any other laureate.

The virus will also affect the Nobel Peace Prize ceremony on December 10 in Oslo, which has been scaled back
due to corona restrictions. The award consists of a gold medal, a diploma and a cheque for 10 million Swedish
kronor (950,000 euros, $1.1 million).

Q.58 [30465398]
In 75 years, the United Nations, its specialised agencies, related agencies, funds, programmes and staff were
awarded the prestigious Nobel Peace Prize [2] times, more than any other laureate. Which of the following has
been redacted with [2] in the passage above?

a 8

b 10

c 12

d 15

https://www.aspiration.ai/LAW/sis/Solution.jsp?qsetId=Lw5e7noFooA=&qsetName=LST Mock 20 2021 (CLAT) 98/208


12/18/2020 Mock Analysis

Solution:
 Answer key/Solution
Correct Answer : c
Your Answer : a
This is the 12th time the Noble Peace Prize has gone to the UN, one of its
agencies or personalities — more than any other laureate. In 75 years, the United Nations, its specialised
agencies, related agencies, funds, programmes and staff were awarded the prestigious Nobel Peace Prize
twelve times. One agency, the United Nations High Commissioner for Refugees (UNHCR) received the famous
prize in both 1954 and 1981.
FeedBack Bookmark

Passage-06

The Nobel Peace Prize 2020 was awarded to the [1] for feeding millions of people from Yemen to North Korea,
with the coronavirus pandemic seen pushing millions more into hunger.

The [1] was honoured for “its efforts to combat hunger, for its contribution to bettering conditions for peace in
con ict-affected areas and for acting as a driving force in efforts to prevent the use of hunger as a weapon of
war and con ict,” Nobel committee chairwoman Berit Reiss-Andersen said on unveiling the winner in Oslo.

Whether delivering food by helicopter or on the back of an elephant or a camel, the [1] prides itself on being “the
leading humanitarian organisation” in a world where, by its own estimates, some 690 million people -- one in 11 -
- go to bed on an empty stomach.

“With this year's award, the Norwegian Nobel Committee wishes to turn the eyes of the world towards the
millions of people who suffer from or face the threat of hunger,” Reiss-Andersen said.

Founded in 1961, the UN organisation helped 97 million people last year, distributing 15 billion rations to people
in 88 countries last year. The numbers are dizzying but only a fraction of the total number in need.

Despite making progress over the past three decades, the UN's goal to eradicate hunger by 2030 appears out of
reach if current trends continue, according to experts.

In 75 years, the United Nations, its specialised agencies, related agencies, funds, programmes and staff were
awarded the prestigious Nobel Peace Prize [2] times, more than any other laureate.

The virus will also affect the Nobel Peace Prize ceremony on December 10 in Oslo, which has been scaled back
due to corona restrictions. The award consists of a gold medal, a diploma and a cheque for 10 million Swedish
kronor (950,000 euros, $1.1 million).

Q.59 [30465398]
Name the two Secretaries-General of United Nations who were awarded the Noble Peace Prize for their work by
the Norwegian Nobel Committee.

a Ko Annan and Dag Hammarskjold

https://www.aspiration.ai/LAW/sis/Solution.jsp?qsetId=Lw5e7noFooA=&qsetName=LST Mock 20 2021 (CLAT) 99/208


12/18/2020 Mock Analysis

b Antonio Guterres and Ban Ki-moon

c Ban Ki-moon and Ruud Lubbers

d Boutros Boutros-Ghali and António Guterres

Solution:
 Answer key/Solution
Correct Answer : a
Your Answer : a
Two Secretaries-General, Ko Annan and Dag Hammarskjold, were also honoured
for their work by the Norwegian Nobel Committee. After being awarded the Noble Peace Prize jointly with the
world body, Secretary-General Ko Annan in 2001 told UN staff he hoped that winning the prize "will urge us
forward and encourage all of us to tackle our tasks with even greater determination".
FeedBack Bookmark

Passage-06

The Nobel Peace Prize 2020 was awarded to the [1] for feeding millions of people from Yemen to North Korea,
with the coronavirus pandemic seen pushing millions more into hunger.

The [1] was honoured for “its efforts to combat hunger, for its contribution to bettering conditions for peace in
con ict-affected areas and for acting as a driving force in efforts to prevent the use of hunger as a weapon of
war and con ict,” Nobel committee chairwoman Berit Reiss-Andersen said on unveiling the winner in Oslo.

Whether delivering food by helicopter or on the back of an elephant or a camel, the [1] prides itself on being “the
leading humanitarian organisation” in a world where, by its own estimates, some 690 million people -- one in 11 -
- go to bed on an empty stomach.

“With this year's award, the Norwegian Nobel Committee wishes to turn the eyes of the world towards the
millions of people who suffer from or face the threat of hunger,” Reiss-Andersen said.

Founded in 1961, the UN organisation helped 97 million people last year, distributing 15 billion rations to people
in 88 countries last year. The numbers are dizzying but only a fraction of the total number in need.

Despite making progress over the past three decades, the UN's goal to eradicate hunger by 2030 appears out of
reach if current trends continue, according to experts.

In 75 years, the United Nations, its specialised agencies, related agencies, funds, programmes and staff were
awarded the prestigious Nobel Peace Prize [2] times, more than any other laureate.

The virus will also affect the Nobel Peace Prize ceremony on December 10 in Oslo, which has been scaled back
due to corona restrictions. The award consists of a gold medal, a diploma and a cheque for 10 million Swedish
kronor (950,000 euros, $1.1 million).

https://www.aspiration.ai/LAW/sis/Solution.jsp?qsetId=Lw5e7noFooA=&qsetName=LST Mock 20 2021 (CLAT) 100/208


12/18/2020 Mock Analysis

Q.60 [30465398]
Who among the following received a third nomination for the Nobel Peace Prize 2020 for his recent role in
helping broker relations between Israel and the United Arab Emirates (UAE)?

a Vladimir Putin

b Donald Trump

c Kumi Naidoo

d Emmanuel Macron

Solution:
 Answer key/Solution
Correct Answer : b
Your Answer : b
The US President Donald Trump has received a third nomination for the Nobel
Peace Prize. According to Sky News Australia, four Australian law professors recently nominated the
president for the high honor, with one of them, David Flint, citing his recent role in helping broker relations
between Israel and the United Arab Emirates (UAE). Trump also received a nomination for the award from a
member of the Swedish Parliament and a member of the Norwegian Parliament earlier this month.
FeedBack Bookmark

https://www.aspiration.ai/LAW/sis/Solution.jsp?qsetId=Lw5e7noFooA=&qsetName=LST Mock 20 2021 (CLAT) 101/208


12/18/2020 Mock Analysis

Passage-06

The Nobel Peace Prize 2020 was awarded to the [1] for feeding millions of people from Yemen to North Korea,
with the coronavirus pandemic seen pushing millions more into hunger.

The [1] was honoured for “its efforts to combat hunger, for its contribution to bettering conditions for peace in
con ict-affected areas and for acting as a driving force in efforts to prevent the use of hunger as a weapon of
war and con ict,” Nobel committee chairwoman Berit Reiss-Andersen said on unveiling the winner in Oslo.

Whether delivering food by helicopter or on the back of an elephant or a camel, the [1] prides itself on being “the
leading humanitarian organisation” in a world where, by its own estimates, some 690 million people -- one in 11 -
- go to bed on an empty stomach.

“With this year's award, the Norwegian Nobel Committee wishes to turn the eyes of the world towards the
millions of people who suffer from or face the threat of hunger,” Reiss-Andersen said.

Founded in 1961, the UN organisation helped 97 million people last year, distributing 15 billion rations to people
in 88 countries last year. The numbers are dizzying but only a fraction of the total number in need.

Despite making progress over the past three decades, the UN's goal to eradicate hunger by 2030 appears out of
reach if current trends continue, according to experts.

In 75 years, the United Nations, its specialised agencies, related agencies, funds, programmes and staff were
awarded the prestigious Nobel Peace Prize [2] times, more than any other laureate.

The virus will also affect the Nobel Peace Prize ceremony on December 10 in Oslo, which has been scaled back
due to corona restrictions. The award consists of a gold medal, a diploma and a cheque for 10 million Swedish
kronor (950,000 euros, $1.1 million).

Q.61 [30465398]
He was nominated for the Nobel Peace Prize in 1937, 1938, 1939, 1947 and, nally, a few days before he was
murdered in January 1948 but never awarded the Peace Prize. Who was he?

a Winston Churchill

b Mahatma Gandhi

c Franklin D. Roosevelt

d Swami Vivekananda

https://www.aspiration.ai/LAW/sis/Solution.jsp?qsetId=Lw5e7noFooA=&qsetName=LST Mock 20 2021 (CLAT) 102/208


12/18/2020 Mock Analysis

Solution:
 Answer key/Solution
Correct Answer : b
Your Answer : b
Mahatma Gandhi was nominated in 1937, 1938, 1939, 1947 and, nally, a few
days before he was murdered in January 1948. The omission has been publicly regretted by later members of
the Nobel Committee; when the Dalai Lama was awarded the Peace Prize in 1989, the chairman of the
committee said that this was “in part a tribute to the memory of Mahatma Gandhi”. However, the committee
has never commented on the speculations as to why Gandhi was not awarded the prize, and until recently the
sources which might shed some light on the matter were unavailable.

FeedBack Bookmark

Passage-07

“In space, we always talk about rsts,” space technologist Stewart Bain muses. “[1] would be the rst dedicated
satellite constellation looking at managing the situation in space such as congestion, threats of collisions, and
addressing space tra c management.”

Stewart is the CEO of Canada-based space company NorthStar Earth & Space that made headlines for its
efforts to address the threat of space debris, which has increased over the recent years. NorthStar plans to
provide safe and sustainable solutions through its [1] satellite constellation designed speci cally for Space
Situational Awareness (SSA).

[2] was the rst human-made object to orbit the Earth. Its launch created new opportunities in space. However,
63 years later, it has also added 29,000 objects as debris (larger than 10 centimetres in diameter) in addition to
the 2,500-plus active satellites in space.

Other companies such as SpaceX and House Space Force Caucus have been vocal about the problem of debris.
Since Russian cosmonaut [3]’s rst human space ight in 1961, numerous space missions have contributed to
the build-up of debris.

Most ‘space junk’ can reach a speed of nearly 29,000 kilometres per hour. Because of the amount of debris in
LEO and the speed at which they are moving, current and future space-based services, explorations, and
operations pose a safety risk to people and property in space and on Earth, according to NASA.

A space-based satellite system has certain advantages over ground-based systems, including having multiple
points of view for an object; not reliant on weather changes and atmospheric interferences; and the ability to
make frequent revisits. Stewart adds, “It picks up changes quickly, if two objects collide and create debris. You
want to know what is happening and don’t want to wait long for a new map of all this new debris. So, in space
and from space, those things are more achievable.”

The system works on NorthStar’s algorithms to provide space tra c data and collision avoidance navigation
services, and the satellites are needed to get the data from the right perspective.

https://www.aspiration.ai/LAW/sis/Solution.jsp?qsetId=Lw5e7noFooA=&qsetName=LST Mock 20 2021 (CLAT) 103/208


12/18/2020 Mock Analysis

Q.62 [30465398]
NorthStar plans to provide safe and sustainable solutions through its [1] satellite constellation designed
speci cally for Space Situational Awareness (SSA). What is the name of the satellite redacted with [1] in the
passage above?

a Skylark

b IKONOS

c Quickbird

d AURA

Solution:
 Answer key/Solution
Correct Answer : a
NorthStar plans to provide safe and sustainable solutions through its Skylark
satellite constellation designed speci cally for Space Situational Awareness
(SSA). “In space, we always talk about rsts,” space technologist Stewart Bain muses. “Skylark would be the
rst dedicated satellite constellation looking at managing the situation in space such as congestion, threats of
collisions, and addressing space tra c management.”

FeedBack Bookmark

https://www.aspiration.ai/LAW/sis/Solution.jsp?qsetId=Lw5e7noFooA=&qsetName=LST Mock 20 2021 (CLAT) 104/208


12/18/2020 Mock Analysis

Passage-07

“In space, we always talk about rsts,” space technologist Stewart Bain muses. “[1] would be the rst dedicated
satellite constellation looking at managing the situation in space such as congestion, threats of collisions, and
addressing space tra c management.”

Stewart is the CEO of Canada-based space company NorthStar Earth & Space that made headlines for its
efforts to address the threat of space debris, which has increased over the recent years. NorthStar plans to
provide safe and sustainable solutions through its [1] satellite constellation designed speci cally for Space
Situational Awareness (SSA).

[2] was the rst human-made object to orbit the Earth. Its launch created new opportunities in space. However,
63 years later, it has also added 29,000 objects as debris (larger than 10 centimetres in diameter) in addition to
the 2,500-plus active satellites in space.

Other companies such as SpaceX and House Space Force Caucus have been vocal about the problem of debris.
Since Russian cosmonaut [3]’s rst human space ight in 1961, numerous space missions have contributed to
the build-up of debris.

Most ‘space junk’ can reach a speed of nearly 29,000 kilometres per hour. Because of the amount of debris in
LEO and the speed at which they are moving, current and future space-based services, explorations, and
operations pose a safety risk to people and property in space and on Earth, according to NASA.

A space-based satellite system has certain advantages over ground-based systems, including having multiple
points of view for an object; not reliant on weather changes and atmospheric interferences; and the ability to
make frequent revisits. Stewart adds, “It picks up changes quickly, if two objects collide and create debris. You
want to know what is happening and don’t want to wait long for a new map of all this new debris. So, in space
and from space, those things are more achievable.”

The system works on NorthStar’s algorithms to provide space tra c data and collision avoidance navigation
services, and the satellites are needed to get the data from the right perspective.

Q.63 [30465398]
What is the name of the rst human-made object to orbit the Earth whose name has been redacted with [2] in
the passage above?

a Explorer 1

b Sputnik 1

c Apollo

d Voyager 1

https://www.aspiration.ai/LAW/sis/Solution.jsp?qsetId=Lw5e7noFooA=&qsetName=LST Mock 20 2021 (CLAT) 105/208


12/18/2020 Mock Analysis

Solution:
 Answer key/Solution
Correct Answer : b
Your Answer : b
Sputnik 1 was the rst arti cial Earth satellite. The Soviet Union launched it into
an elliptical low Earth orbit on 4 October 1957.

FeedBack Bookmark

Passage-07

“In space, we always talk about rsts,” space technologist Stewart Bain muses. “[1] would be the rst dedicated
satellite constellation looking at managing the situation in space such as congestion, threats of collisions, and
addressing space tra c management.”

Stewart is the CEO of Canada-based space company NorthStar Earth & Space that made headlines for its
efforts to address the threat of space debris, which has increased over the recent years. NorthStar plans to
provide safe and sustainable solutions through its [1] satellite constellation designed speci cally for Space
Situational Awareness (SSA).

[2] was the rst human-made object to orbit the Earth. Its launch created new opportunities in space. However,
63 years later, it has also added 29,000 objects as debris (larger than 10 centimetres in diameter) in addition to
the 2,500-plus active satellites in space.

Other companies such as SpaceX and House Space Force Caucus have been vocal about the problem of debris.
Since Russian cosmonaut [3]’s rst human space ight in 1961, numerous space missions have contributed to
the build-up of debris.

Most ‘space junk’ can reach a speed of nearly 29,000 kilometres per hour. Because of the amount of debris in
LEO and the speed at which they are moving, current and future space-based services, explorations, and
operations pose a safety risk to people and property in space and on Earth, according to NASA.

A space-based satellite system has certain advantages over ground-based systems, including having multiple
points of view for an object; not reliant on weather changes and atmospheric interferences; and the ability to
make frequent revisits. Stewart adds, “It picks up changes quickly, if two objects collide and create debris. You
want to know what is happening and don’t want to wait long for a new map of all this new debris. So, in space
and from space, those things are more achievable.”

The system works on NorthStar’s algorithms to provide space tra c data and collision avoidance navigation
services, and the satellites are needed to get the data from the right perspective.

Q.64 [30465398]
Who was the rst human to journey into outer space whose name has been redacted with [3] in the passage
above?

a Michael Collins

https://www.aspiration.ai/LAW/sis/Solution.jsp?qsetId=Lw5e7noFooA=&qsetName=LST Mock 20 2021 (CLAT) 106/208


12/18/2020 Mock Analysis

b Buzz Aldrin

c Yuri Gagarin

d Neil Armstrong

Solution:
 Answer key/Solution
Correct Answer : c
Your Answer : c
Yuri Gagarin was a Soviet Air Forces pilot and cosmonaut who became the rst
human to journey into outer space, achieving a major milestone in the Space Race; his capsule, Vostok 1,
completed one orbit of Earth on 12 April 1961. Gagarin became an international celebrity and was awarded
many medals and titles, including Hero of the Soviet Union, his nation's highest honour.

FeedBack Bookmark

https://www.aspiration.ai/LAW/sis/Solution.jsp?qsetId=Lw5e7noFooA=&qsetName=LST Mock 20 2021 (CLAT) 107/208


12/18/2020 Mock Analysis

Passage-07

“In space, we always talk about rsts,” space technologist Stewart Bain muses. “[1] would be the rst dedicated
satellite constellation looking at managing the situation in space such as congestion, threats of collisions, and
addressing space tra c management.”

Stewart is the CEO of Canada-based space company NorthStar Earth & Space that made headlines for its
efforts to address the threat of space debris, which has increased over the recent years. NorthStar plans to
provide safe and sustainable solutions through its [1] satellite constellation designed speci cally for Space
Situational Awareness (SSA).

[2] was the rst human-made object to orbit the Earth. Its launch created new opportunities in space. However,
63 years later, it has also added 29,000 objects as debris (larger than 10 centimetres in diameter) in addition to
the 2,500-plus active satellites in space.

Other companies such as SpaceX and House Space Force Caucus have been vocal about the problem of debris.
Since Russian cosmonaut [3]’s rst human space ight in 1961, numerous space missions have contributed to
the build-up of debris.

Most ‘space junk’ can reach a speed of nearly 29,000 kilometres per hour. Because of the amount of debris in
LEO and the speed at which they are moving, current and future space-based services, explorations, and
operations pose a safety risk to people and property in space and on Earth, according to NASA.

A space-based satellite system has certain advantages over ground-based systems, including having multiple
points of view for an object; not reliant on weather changes and atmospheric interferences; and the ability to
make frequent revisits. Stewart adds, “It picks up changes quickly, if two objects collide and create debris. You
want to know what is happening and don’t want to wait long for a new map of all this new debris. So, in space
and from space, those things are more achievable.”

The system works on NorthStar’s algorithms to provide space tra c data and collision avoidance navigation
services, and the satellites are needed to get the data from the right perspective.

Q.65 [30465398]
The International Space Station (ISS) is a multinational collaborative project between ve participating space
agencies, which of the following space agencies is not a participating space agency?

a Roscosmos

b JAXA

c Canadian Space Agency

d ISRO

https://www.aspiration.ai/LAW/sis/Solution.jsp?qsetId=Lw5e7noFooA=&qsetName=LST Mock 20 2021 (CLAT) 108/208


12/18/2020 Mock Analysis

Solution:
 Answer key/Solution
Correct Answer : d
Your Answer : d
The International Space Station (ISS) is a modular space station (habitable
arti cial satellite) in low Earth orbit. It is a multinational collaborative project between ve participating space
agencies: NASA (United States), Roscosmos (Russia), JAXA (Japan), ESA (Europe), and CSA (Canada). The
ownership and use of the space station is established by intergovernmental treaties and agreements.
FeedBack Bookmark

Passage-07

“In space, we always talk about rsts,” space technologist Stewart Bain muses. “[1] would be the rst dedicated
satellite constellation looking at managing the situation in space such as congestion, threats of collisions, and
addressing space tra c management.”

Stewart is the CEO of Canada-based space company NorthStar Earth & Space that made headlines for its
efforts to address the threat of space debris, which has increased over the recent years. NorthStar plans to
provide safe and sustainable solutions through its [1] satellite constellation designed speci cally for Space
Situational Awareness (SSA).

[2] was the rst human-made object to orbit the Earth. Its launch created new opportunities in space. However,
63 years later, it has also added 29,000 objects as debris (larger than 10 centimetres in diameter) in addition to
the 2,500-plus active satellites in space.

Other companies such as SpaceX and House Space Force Caucus have been vocal about the problem of debris.
Since Russian cosmonaut [3]’s rst human space ight in 1961, numerous space missions have contributed to
the build-up of debris.

Most ‘space junk’ can reach a speed of nearly 29,000 kilometres per hour. Because of the amount of debris in
LEO and the speed at which they are moving, current and future space-based services, explorations, and
operations pose a safety risk to people and property in space and on Earth, according to NASA.

A space-based satellite system has certain advantages over ground-based systems, including having multiple
points of view for an object; not reliant on weather changes and atmospheric interferences; and the ability to
make frequent revisits. Stewart adds, “It picks up changes quickly, if two objects collide and create debris. You
want to know what is happening and don’t want to wait long for a new map of all this new debris. So, in space
and from space, those things are more achievable.”

The system works on NorthStar’s algorithms to provide space tra c data and collision avoidance navigation
services, and the satellites are needed to get the data from the right perspective.

Q.66 [30465398]
In November 2020, as part of steps to deepen cooperation in civil space activities, the space agencies of India
and ____________were working together to position temporarily Indian tracking facilities.

https://www.aspiration.ai/LAW/sis/Solution.jsp?qsetId=Lw5e7noFooA=&qsetName=LST Mock 20 2021 (CLAT) 109/208


12/18/2020 Mock Analysis

a Australia

b USA

c Canada

d Russia

Solution:
 Answer key/Solution
Correct Answer : a
Your Answer : a
As part of steps to deepen cooperation in civil space activities, the space
agencies of India and Australia were working together to position temporarily Indian tracking facilities in
Australia, said Australian Minister for Industry, Science and Technology Karen Andrews. This would support
India’s planned human space ight program.
FeedBack Bookmark

Sec 3

https://www.aspiration.ai/LAW/sis/Solution.jsp?qsetId=Lw5e7noFooA=&qsetName=LST Mock 20 2021 (CLAT) 110/208


12/18/2020 Mock Analysis

Directions for questions 67 to 105: You have been given some passages followed by questions based on each
passage. You are required to choose the most appropriate option which follows from the passage. Only the
information given in the passage should be used for choosing the answer and no external knowledge of law
howsoever prominent is to be applied.

Passage – 1

Duress can be de ned as unlawful threat or coercion that causes another person to commit acts that he would
otherwise not commit. Initially the doctrine of duress was only con ned to actual or threatened violence. Over
the years, this doctrine has evolved to include various forms of duress including economic duress. Duress is the
inducement of a transaction in which one party exercises a degree of illegal domination on the other, to obtain
and derive undue bene ts.

Two elements are essential to establish economic duress namely:


(i) the exertion of illegitimate pressure by one party on the other: the pressure should be of such high capacity
that it should be capable of provoking a sense of moral outrage and appear to be beyond the norms of ordinary
commercial practice thus rendering it to be unconscionable enough to be at par with such conduct that the law
expressly recognizes as illegal and unlawful, for example, violation of ant-trust laws of consumer protection. It
must be shown that the pressure was of such nature that the victim's consent to the contract was an involuntary
act on his part. Therefore, the illegitimate pressure must have been such as actually caused the making of the
agreement, in the sense that it would not otherwise have been made either at all or, at least, in the terms in
which it was made.
(ii) signi cant causation i.e. a signi cant cause compelling or pressurizing the other party to act as he did: The
courts must apply the test of a reasonable man to see whether an ordinary man of prudence would have acted
in a similar manner in a similar situation, and if so, then would he have done so because he had no practical
choice but to submit, or due to other reasons. The minimum basic test of subjective causation in economic
duress ought, it appears to be a "but for" test. In other words, the victim must show that, "but for" the threat, he
would not have entered the contract.
The result of the conclusion that a party had consented to a contract or transaction under duress is that it
becomes voidable on part of that party, i.e. it is enforceable only if that party wished it to be so.

Q.67 [30465398]
A, a manufacturing company entered into a contract for carriage of goods with B, a shipping company. A day
before A's cargo was to be loaded on the ship, B company asked A to increase the payment by ten percent over
the stipulated price. As the cargo was of perishable nature and there was no time to nd another shipping
company, A agreed to B's demands. B completed the contract and asked for the full payment, which A declined.
Decide.

a A can decline the entire payment as he consented to the contract under duress.

b A can refuse to pay only the additional amount, as he consented to the construct under duress.

c A cannot refuse to pay the amount as he would have agreed to the same price even without duress.

d A cannot refuse to pay the amount as there was no illegitimate pressure exerted upon him.

https://www.aspiration.ai/LAW/sis/Solution.jsp?qsetId=Lw5e7noFooA=&qsetName=LST Mock 20 2021 (CLAT) 111/208


12/18/2020 Mock Analysis

Solution:
 Answer key/Solution
Correct Answer : d
B did not threaten to do or actually do any illegal act so as to put A under duress.
His demand was ful lled by A readily and without any threat of terminating the
contract, thus it cannot be termed as duress.

FeedBack Bookmark

Directions for questions 67 to 105: You have been given some passages followed by questions based on each
passage. You are required to choose the most appropriate option which follows from the passage. Only the
information given in the passage should be used for choosing the answer and no external knowledge of law
howsoever prominent is to be applied.

Passage – 1

Duress can be de ned as unlawful threat or coercion that causes another person to commit acts that he would
otherwise not commit. Initially the doctrine of duress was only con ned to actual or threatened violence. Over
the years, this doctrine has evolved to include various forms of duress including economic duress. Duress is the
inducement of a transaction in which one party exercises a degree of illegal domination on the other, to obtain
and derive undue bene ts.

Two elements are essential to establish economic duress namely:


(i) the exertion of illegitimate pressure by one party on the other: the pressure should be of such high capacity
that it should be capable of provoking a sense of moral outrage and appear to be beyond the norms of ordinary
commercial practice thus rendering it to be unconscionable enough to be at par with such conduct that the law
expressly recognizes as illegal and unlawful, for example, violation of ant-trust laws of consumer protection. It
must be shown that the pressure was of such nature that the victim's consent to the contract was an involuntary
act on his part. Therefore, the illegitimate pressure must have been such as actually caused the making of the
agreement, in the sense that it would not otherwise have been made either at all or, at least, in the terms in
which it was made.
(ii) signi cant causation i.e. a signi cant cause compelling or pressurizing the other party to act as he did: The
courts must apply the test of a reasonable man to see whether an ordinary man of prudence would have acted
in a similar manner in a similar situation, and if so, then would he have done so because he had no practical
choice but to submit, or due to other reasons. The minimum basic test of subjective causation in economic
duress ought, it appears to be a "but for" test. In other words, the victim must show that, "but for" the threat, he
would not have entered the contract.
The result of the conclusion that a party had consented to a contract or transaction under duress is that it
becomes voidable on part of that party, i.e. it is enforceable only if that party wished it to be so.

Q.68 [30465398]
A entered into a contract with a broadcasting company whereby he agreed that in case of termination of
employment with the said company, he would not work for any other broadcasting company for ve years
following such termination. After A is in fact terminated, he tries to avoid the contract alleging duress, and
states that he agreed to the same only because he was in nancial distress and badly needed a job. The
company argues that it was a standard contract signed by all its employees. Decide.

https://www.aspiration.ai/LAW/sis/Solution.jsp?qsetId=Lw5e7noFooA=&qsetName=LST Mock 20 2021 (CLAT) 112/208


12/18/2020 Mock Analysis

a A can avoid the contract as it is apparently beyond the norms of ordinary commercial practice.

b A cannot avoid the contract as there was no illegitimate pressure on him to sign the contract.

c A can avoid the contract as he would not have signed it but for duress.

d A cannot avoid the contract as the illegitimate pressure did not cause him to sing the contract.

Solution:
 Answer key/Solution
Correct Answer : b
As the contract signed by A was a standard contract signed by all the employees,
it cannot be said to be beyond the norms of ordinary commercial practice.
Similarly, nancial distress cannot be pleaded as entering into that particular contract was not the only option
available. Therefore, option (b) is the correct answer. Option (d), though technically correct, implies that there
was illegitimate pressure in the rst place, thus is the wrong answer.
FeedBack Bookmark

https://www.aspiration.ai/LAW/sis/Solution.jsp?qsetId=Lw5e7noFooA=&qsetName=LST Mock 20 2021 (CLAT) 113/208


12/18/2020 Mock Analysis

Directions for questions 67 to 105: You have been given some passages followed by questions based on each
passage. You are required to choose the most appropriate option which follows from the passage. Only the
information given in the passage should be used for choosing the answer and no external knowledge of law
howsoever prominent is to be applied.

Passage – 1

Duress can be de ned as unlawful threat or coercion that causes another person to commit acts that he would
otherwise not commit. Initially the doctrine of duress was only con ned to actual or threatened violence. Over
the years, this doctrine has evolved to include various forms of duress including economic duress. Duress is the
inducement of a transaction in which one party exercises a degree of illegal domination on the other, to obtain
and derive undue bene ts.

Two elements are essential to establish economic duress namely:


(i) the exertion of illegitimate pressure by one party on the other: the pressure should be of such high capacity
that it should be capable of provoking a sense of moral outrage and appear to be beyond the norms of ordinary
commercial practice thus rendering it to be unconscionable enough to be at par with such conduct that the law
expressly recognizes as illegal and unlawful, for example, violation of ant-trust laws of consumer protection. It
must be shown that the pressure was of such nature that the victim's consent to the contract was an involuntary
act on his part. Therefore, the illegitimate pressure must have been such as actually caused the making of the
agreement, in the sense that it would not otherwise have been made either at all or, at least, in the terms in
which it was made.
(ii) signi cant causation i.e. a signi cant cause compelling or pressurizing the other party to act as he did: The
courts must apply the test of a reasonable man to see whether an ordinary man of prudence would have acted
in a similar manner in a similar situation, and if so, then would he have done so because he had no practical
choice but to submit, or due to other reasons. The minimum basic test of subjective causation in economic
duress ought, it appears to be a "but for" test. In other words, the victim must show that, "but for" the threat, he
would not have entered the contract.
The result of the conclusion that a party had consented to a contract or transaction under duress is that it
becomes voidable on part of that party, i.e. it is enforceable only if that party wished it to be so.

Q.69 [30465398]
A construction company promised X to build an exhibition stand within a month, for an event to be held at the
end of the month. Twenty days after the contract was made and construction had started, the workmen from
the construction company went on strike and the company expressed its inability to ful ll the contract in time.
As the completion of construction within the month was absolutely essential, X agreed under the same contract,
to pay twice the originally stipulated amount, as contribution to the payment of additional wages demanded by
the employees of the company. A new contract was drawn and only upon that did the company build X's
construction. Decide X's position at the time of forming the new contract.

a X was under duress as he would not have agreed to twice the original sum but for the threat.

b X was under duress as no reasonable man would pay twice the sum to get a construction completed.

c X was not under duress as his consent to the contract was not involuntary.

d Both (a) and (b).

https://www.aspiration.ai/LAW/sis/Solution.jsp?qsetId=Lw5e7noFooA=&qsetName=LST Mock 20 2021 (CLAT) 114/208


12/18/2020 Mock Analysis

Solution:
 Answer key/Solution
Correct Answer : d
As X needed the construction to be completed within the stipulated time frame,
his paying double the originally agreed amount goes far beyond commercial
practice, cannot be said to be voluntary or an act of a reasonable person and would not have been the case but
for the pressure exerted by the impending threat of the work not being completed on time. Thus, both (a) and
(b) are correct answers.

FeedBack Bookmark

Directions for questions 67 to 105: You have been given some passages followed by questions based on each
passage. You are required to choose the most appropriate option which follows from the passage. Only the
information given in the passage should be used for choosing the answer and no external knowledge of law
howsoever prominent is to be applied.

Passage – 1

Duress can be de ned as unlawful threat or coercion that causes another person to commit acts that he would
otherwise not commit. Initially the doctrine of duress was only con ned to actual or threatened violence. Over
the years, this doctrine has evolved to include various forms of duress including economic duress. Duress is the
inducement of a transaction in which one party exercises a degree of illegal domination on the other, to obtain
and derive undue bene ts.

Two elements are essential to establish economic duress namely:


(i) the exertion of illegitimate pressure by one party on the other: the pressure should be of such high capacity
that it should be capable of provoking a sense of moral outrage and appear to be beyond the norms of ordinary
commercial practice thus rendering it to be unconscionable enough to be at par with such conduct that the law
expressly recognizes as illegal and unlawful, for example, violation of ant-trust laws of consumer protection. It
must be shown that the pressure was of such nature that the victim's consent to the contract was an involuntary
act on his part. Therefore, the illegitimate pressure must have been such as actually caused the making of the
agreement, in the sense that it would not otherwise have been made either at all or, at least, in the terms in
which it was made.
(ii) signi cant causation i.e. a signi cant cause compelling or pressurizing the other party to act as he did: The
courts must apply the test of a reasonable man to see whether an ordinary man of prudence would have acted
in a similar manner in a similar situation, and if so, then would he have done so because he had no practical
choice but to submit, or due to other reasons. The minimum basic test of subjective causation in economic
duress ought, it appears to be a "but for" test. In other words, the victim must show that, "but for" the threat, he
would not have entered the contract.
The result of the conclusion that a party had consented to a contract or transaction under duress is that it
becomes voidable on part of that party, i.e. it is enforceable only if that party wished it to be so.

https://www.aspiration.ai/LAW/sis/Solution.jsp?qsetId=Lw5e7noFooA=&qsetName=LST Mock 20 2021 (CLAT) 115/208


12/18/2020 Mock Analysis

Q.70 [30465398]
A was a farmer who sold his produce to a mill owner, B. B owed A payments for several transactions, all
amounting to twenty lakhs in total. As the season for cropping was upon him, A desperately needed money to
buy seeds. Upon asking from B, B offered to pay him fteen lakh as full and nal settlement of his claims, or
else offered to pay the money after six months. As A had no time to enforce his claim in courts, he agreed to the
settlement offered by B and signed the contract. Decide.

a B owes the entire amount to A.

b B owes only fteen lakhs to B as per under the contract.

c B owes no money to A as the contract is invalid due to duress.

d B's obligation to pay depends upon A's wishes.

Solution:
 Answer key/Solution
Correct Answer : d
As the contract was signed by A under duress, it is not valid, option (b) is
incorrect. However, it is not void or invalid either, thus option (a) and (c) are
incorrect. The fate of the contract depends upon A's choice to enforce it, thus option (d) is correct.
FeedBack Bookmark

https://www.aspiration.ai/LAW/sis/Solution.jsp?qsetId=Lw5e7noFooA=&qsetName=LST Mock 20 2021 (CLAT) 116/208


12/18/2020 Mock Analysis

Directions for questions 67 to 105: You have been given some passages followed by questions based on each
passage. You are required to choose the most appropriate option which follows from the passage. Only the
information given in the passage should be used for choosing the answer and no external knowledge of law
howsoever prominent is to be applied.

Passage – 1

Duress can be de ned as unlawful threat or coercion that causes another person to commit acts that he would
otherwise not commit. Initially the doctrine of duress was only con ned to actual or threatened violence. Over
the years, this doctrine has evolved to include various forms of duress including economic duress. Duress is the
inducement of a transaction in which one party exercises a degree of illegal domination on the other, to obtain
and derive undue bene ts.

Two elements are essential to establish economic duress namely:


(i) the exertion of illegitimate pressure by one party on the other: the pressure should be of such high capacity
that it should be capable of provoking a sense of moral outrage and appear to be beyond the norms of ordinary
commercial practice thus rendering it to be unconscionable enough to be at par with such conduct that the law
expressly recognizes as illegal and unlawful, for example, violation of ant-trust laws of consumer protection. It
must be shown that the pressure was of such nature that the victim's consent to the contract was an involuntary
act on his part. Therefore, the illegitimate pressure must have been such as actually caused the making of the
agreement, in the sense that it would not otherwise have been made either at all or, at least, in the terms in
which it was made.
(ii) signi cant causation i.e. a signi cant cause compelling or pressurizing the other party to act as he did: The
courts must apply the test of a reasonable man to see whether an ordinary man of prudence would have acted
in a similar manner in a similar situation, and if so, then would he have done so because he had no practical
choice but to submit, or due to other reasons. The minimum basic test of subjective causation in economic
duress ought, it appears to be a "but for" test. In other words, the victim must show that, "but for" the threat, he
would not have entered the contract.
The result of the conclusion that a party had consented to a contract or transaction under duress is that it
becomes voidable on part of that party, i.e. it is enforceable only if that party wished it to be so.

Q.71 [30465398]
A was starting a business of publishing. He wanted a particular kind of paper manufactured by B, an established
manufacturer. B entered into a contract with A for supply of paper at the rates that were double the prevailing
market rates elsewhere for the same kind of paper. Later upon nding out about the real rates in the market, A
tried to avoid the contract by alleging duress. Can he do so?

a Yes, as but for the threat by B, A would not have entered into the contract.

b Yes, as no reasonable man would have agreed to such high rates.

c No, agreeing to double the rates was the only way to get the desired paper for A.

d No, as A had other practical choices than to submit to the contract with B.

https://www.aspiration.ai/LAW/sis/Solution.jsp?qsetId=Lw5e7noFooA=&qsetName=LST Mock 20 2021 (CLAT) 117/208


12/18/2020 Mock Analysis

Solution:
 Answer key/Solution
Correct Answer : d
As there were other practically available alternatives in the market, A's ignorance
of them cannot avail him of the defence of duress. There was no threat of
illegitimate nature from B and thus A cannot avoid the construct.
FeedBack Bookmark

Directions for questions 67 to 105: You have been given some passages followed by questions based on each
passage. You are required to choose the most appropriate option which follows from the passage. Only the
information given in the passage should be used for choosing the answer and no external knowledge of law
howsoever prominent is to be applied.

Passage – 2

The principle of the Doctrine of Agency of Necessity originates from a situation where an agent exceeds his
authority by acting on behalf of the principal in an emergency situation. This occurs when one party, the agent,
is faced with an emergency which poses an imminent threat to the interests or the property of another party, the
Principal, and there is insu cient time or means for the agent to seek for the Principal's directions or authority
regarding the matter.

There are three essentials for the application of this doctrine, viz., there must be a genuine necessity, it must be
impossible for the agent to get the principal's instruction, and the agent must have acted bona de.

Historically, the Doctrine of Agency of Necessity was applicable only to the carriage of goods by the sea which
involved the captain's action to save a ship in the course of a voyage, or the cargo it might be carrying in
threatening situations. Gradually, similar cases that involved comparable carriage of goods on land and some
other forms of agent-principal relations came about to be in the ambit of the Doctrine.

Historically, the Common law principles did not require pre-existing contractual relations for the doctrine of
agency of necessity to be applicable. However Indian law mandates for a contractual relation to be present
between the principal and agent for the doctrine to be applicable as stated in Section 189 of The Indian Contract
Act, 18724.

Q.72 [30465398]
Which of the following could be the purpose behind the Indian law mandating the existence of contractual
relationship for the application of the doctrine of agency by necessity?

a Protection of the agent from liability of acts done outside of the scope of his agency.

b Protection of the principal from incurring liability for acts done by his agents outside of their authority.

c Protection of persons from incurring liability for acts done by others who are not their agents.

d Encouraging people to enter into contracts of agency.

https://www.aspiration.ai/LAW/sis/Solution.jsp?qsetId=Lw5e7noFooA=&qsetName=LST Mock 20 2021 (CLAT) 118/208


12/18/2020 Mock Analysis

Solution:
 Answer key/Solution
Correct Answer : c
Your Answer : a
As this doctrine allows the agent to act beyond his authority in emergency
situations, the insistence on existence of contractual relations limits the people who can act in this way on
behalf of a certain principal, thereby protecting the principal from incurring liability.

FeedBack Bookmark

Directions for questions 67 to 105: You have been given some passages followed by questions based on each
passage. You are required to choose the most appropriate option which follows from the passage. Only the
information given in the passage should be used for choosing the answer and no external knowledge of law
howsoever prominent is to be applied.

Passage – 2

The principle of the Doctrine of Agency of Necessity originates from a situation where an agent exceeds his
authority by acting on behalf of the principal in an emergency situation. This occurs when one party, the agent,
is faced with an emergency which poses an imminent threat to the interests or the property of another party, the
Principal, and there is insu cient time or means for the agent to seek for the Principal's directions or authority
regarding the matter.

There are three essentials for the application of this doctrine, viz., there must be a genuine necessity, it must be
impossible for the agent to get the principal's instruction, and the agent must have acted bona de.

Historically, the Doctrine of Agency of Necessity was applicable only to the carriage of goods by the sea which
involved the captain's action to save a ship in the course of a voyage, or the cargo it might be carrying in
threatening situations. Gradually, similar cases that involved comparable carriage of goods on land and some
other forms of agent-principal relations came about to be in the ambit of the Doctrine.

Historically, the Common law principles did not require pre-existing contractual relations for the doctrine of
agency of necessity to be applicable. However Indian law mandates for a contractual relation to be present
between the principal and agent for the doctrine to be applicable as stated in Section 189 of The Indian Contract
Act, 18724.

Q.73 [30465398]
Which of the following can be said to be the principle underlying the fact that a principal is liable to the agent if
the agent makes any provisions or spends any money in emergency situations for protecting the interests of
such principal?

a A person acting bona de for another should be compensated for his expenses.

b No person should be made liable for an act committed by his agent.

https://www.aspiration.ai/LAW/sis/Solution.jsp?qsetId=Lw5e7noFooA=&qsetName=LST Mock 20 2021 (CLAT) 119/208


12/18/2020 Mock Analysis

c The principal is vicariously liable to third parties for all the acts done by his agent.

d Principal's liability is co-extensive with the liability of the agent in emergency situations.

Solution:
 Answer key/Solution
Correct Answer : a
As principal is liable to pay to the agent, the principle is that of restitution through
compensation, thus making good to the agent for the efforts taken by him for
protection of interests of the principal.
FeedBack Bookmark

Directions for questions 67 to 105: You have been given some passages followed by questions based on each
passage. You are required to choose the most appropriate option which follows from the passage. Only the
information given in the passage should be used for choosing the answer and no external knowledge of law
howsoever prominent is to be applied.

Passage – 2

The principle of the Doctrine of Agency of Necessity originates from a situation where an agent exceeds his
authority by acting on behalf of the principal in an emergency situation. This occurs when one party, the agent,
is faced with an emergency which poses an imminent threat to the interests or the property of another party, the
Principal, and there is insu cient time or means for the agent to seek for the Principal's directions or authority
regarding the matter.

There are three essentials for the application of this doctrine, viz., there must be a genuine necessity, it must be
impossible for the agent to get the principal's instruction, and the agent must have acted bona de.

Historically, the Doctrine of Agency of Necessity was applicable only to the carriage of goods by the sea which
involved the captain's action to save a ship in the course of a voyage, or the cargo it might be carrying in
threatening situations. Gradually, similar cases that involved comparable carriage of goods on land and some
other forms of agent-principal relations came about to be in the ambit of the Doctrine.

Historically, the Common law principles did not require pre-existing contractual relations for the doctrine of
agency of necessity to be applicable. However Indian law mandates for a contractual relation to be present
between the principal and agent for the doctrine to be applicable as stated in Section 189 of The Indian Contract
Act, 18724.

Q.74 [30465398]
A was the captain of a ship and was carrying cargo owned by various persons with him. The ship lost its way
due to bad weather and the crew was stranded on it for weeks. As there was a shortage of food on the ship, A
opened one of the cargo boxes and the crew ate the items in it. The cargo that was consumed was owned by B,
who sued A for conversion of his property. A pleads the defence of agency of necessity. According to the
position of English common law, will A be successful in his defence?

a Yes, as no contractual relation is required for the application of the doctrine.

https://www.aspiration.ai/LAW/sis/Solution.jsp?qsetId=Lw5e7noFooA=&qsetName=LST Mock 20 2021 (CLAT) 120/208


12/18/2020 Mock Analysis

b Yes, as there was an imminent threat of starvation to the crew.

c No, as the captain of the ship cannot be an agent of the owner of the goods that he is carrying.

d No, as the act done by A was not for protection of interests of the principal or a third party.

Solution:
 Answer key/Solution
Correct Answer : d
Your Answer : d
The principle of agency by necessity implies an imminent threat to interests of
the principal. In this case, there was no threat to the goods owned by B and thus A's actions cannot be saved
by this doctrine.

FeedBack Bookmark

Directions for questions 67 to 105: You have been given some passages followed by questions based on each
passage. You are required to choose the most appropriate option which follows from the passage. Only the
information given in the passage should be used for choosing the answer and no external knowledge of law
howsoever prominent is to be applied.

Passage – 2

The principle of the Doctrine of Agency of Necessity originates from a situation where an agent exceeds his
authority by acting on behalf of the principal in an emergency situation. This occurs when one party, the agent,
is faced with an emergency which poses an imminent threat to the interests or the property of another party, the
Principal, and there is insu cient time or means for the agent to seek for the Principal's directions or authority
regarding the matter.

There are three essentials for the application of this doctrine, viz., there must be a genuine necessity, it must be
impossible for the agent to get the principal's instruction, and the agent must have acted bona de.

Historically, the Doctrine of Agency of Necessity was applicable only to the carriage of goods by the sea which
involved the captain's action to save a ship in the course of a voyage, or the cargo it might be carrying in
threatening situations. Gradually, similar cases that involved comparable carriage of goods on land and some
other forms of agent-principal relations came about to be in the ambit of the Doctrine.

Historically, the Common law principles did not require pre-existing contractual relations for the doctrine of
agency of necessity to be applicable. However Indian law mandates for a contractual relation to be present
between the principal and agent for the doctrine to be applicable as stated in Section 189 of The Indian Contract
Act, 18724.

https://www.aspiration.ai/LAW/sis/Solution.jsp?qsetId=Lw5e7noFooA=&qsetName=LST Mock 20 2021 (CLAT) 121/208


12/18/2020 Mock Analysis

Q.75 [30465398]
A was the agent of P and was sent to another country to buy some cloth on behalf of his principal. However,
after he had bought the cloth and before he could come home, a war broke out between the two countries. A
tried to contact P to seek directions as to what was to be done with the cloth, but the communications were
disconnected. After waiting for a year for the war to end, he started selling the cloth because its value was
appreciating. After the war ended, P sued A for exceeding his authority. Does the doctrine of agency of
necessity apply in this case?

a Yes, as it was impossible for A to get the P's directions.

b No, as there was no imminent threat to the cloth.

c No, as the A should have waited for P to give instructions.

d Yes, as the A acted in good faith in order to increase the pro t for P.

Solution:
 Answer key/Solution
Correct Answer : b
Your Answer : b
Increasing the pro ts cannot be called as an imminent threat to the goods and
thus the doctrine does not apply in this case.
FeedBack Bookmark

https://www.aspiration.ai/LAW/sis/Solution.jsp?qsetId=Lw5e7noFooA=&qsetName=LST Mock 20 2021 (CLAT) 122/208


12/18/2020 Mock Analysis

Directions for questions 67 to 105: You have been given some passages followed by questions based on each
passage. You are required to choose the most appropriate option which follows from the passage. Only the
information given in the passage should be used for choosing the answer and no external knowledge of law
howsoever prominent is to be applied.

Passage – 2

The principle of the Doctrine of Agency of Necessity originates from a situation where an agent exceeds his
authority by acting on behalf of the principal in an emergency situation. This occurs when one party, the agent,
is faced with an emergency which poses an imminent threat to the interests or the property of another party, the
Principal, and there is insu cient time or means for the agent to seek for the Principal's directions or authority
regarding the matter.

There are three essentials for the application of this doctrine, viz., there must be a genuine necessity, it must be
impossible for the agent to get the principal's instruction, and the agent must have acted bona de.

Historically, the Doctrine of Agency of Necessity was applicable only to the carriage of goods by the sea which
involved the captain's action to save a ship in the course of a voyage, or the cargo it might be carrying in
threatening situations. Gradually, similar cases that involved comparable carriage of goods on land and some
other forms of agent-principal relations came about to be in the ambit of the Doctrine.

Historically, the Common law principles did not require pre-existing contractual relations for the doctrine of
agency of necessity to be applicable. However Indian law mandates for a contractual relation to be present
between the principal and agent for the doctrine to be applicable as stated in Section 189 of The Indian Contract
Act, 18724.

Q.76 [30465398]
A was tasked with carrying B's cargo containing tomatoes from Chennai to Mumbai and then to Nagpur. After
reaching Mumbai, B found that the workers at the dock were on strike, due to which the unloading of the cargo
was delayed by three days. Considering that the journey from Mumbai to Nagpur would take another day, B
decided to sell the tomatoes in Mumbai itself. Which of the following facts would not support the application of
the doctrine of agency by necessity in A's favour?

a That there was no contract of agency between A and B.

b That there was no channel of communication to contact B instantly.

c That A honestly believed that the tomatoes would be spoiled in another day.

d That the tomatoes would have been spoilt in less than a day.

https://www.aspiration.ai/LAW/sis/Solution.jsp?qsetId=Lw5e7noFooA=&qsetName=LST Mock 20 2021 (CLAT) 123/208


12/18/2020 Mock Analysis

Solution:
 Answer key/Solution
Correct Answer : a
Your Answer : c
All the other three options support the application of the doctrine. However, in
India, as a contractual relationship is required for the doctrine to apply, option (a) is the correct answer.
FeedBack Bookmark

Directions for questions 67 to 105: You have been given some passages followed by questions based on each
passage. You are required to choose the most appropriate option which follows from the passage. Only the
information given in the passage should be used for choosing the answer and no external knowledge of law
howsoever prominent is to be applied.

Passage – 3

Doctrine of Election has its source in Section 3 of the transfer of property act 1882 and sections 180-190 of
Indian succession act. Election means a choice between two alternative or con icting rights and is based on the
principle that the person taking the bene t should also bear the burden. It states that when a party transfers a
property over which he does not hold any right of transfer and entailed in that transaction is the bene t
conferred upon the original owner of the property, such owner must elect his option to either validate such
transfer of property or reject it; upon rejection, the bene t shall be reverted back to the transferor. This rule is
subject to two limitations, in which case the person who is supposed to elect either of the options can refuse to
do so, keep both his property and the bene t and can make good to the disappointed transferee by paying him
the amount or market value of the property that was attempted to be transferred to him. This exception is
available in two cases:

1. Where the transfer has been through gratuitous means and the transferor has died or become incapable of
making a fresh transfer.
2. In all cases where the transfer is for consideration.
The election by the owner can either be direct or indirect. In direct election, it is simply through communication
about the elected choice or option. Bene t shall be deemed to be accepted by the person who is supposed to
elect, if he is aware of his duty to elect and has enjoyed the bene t for two years without doing any act to
express dissent or, has enjoyed the property in such a way that it is impossible for the transferee to enjoy the
property in the same way, for example, by exhausting its resources.

There are certain quali cations to this doctrine. When the owner who is considering the election between
retaining the property and accepting a particular bene t, chooses the former, he is not bound to relinquish any
other additional bene t that he gains through the transaction.

Q.77 [30465398]
A transferred a house owned by B worth Rs.10 lakh, to C and through the same instrument, gave B Rs.15 lakh in
lieu of his house. However, before B could decide whether he wanted to keep the house or take the Rs. 15 lakh,
A died. Decide the correct course of action for B.

a B's position is unchanged upon A's death.

https://www.aspiration.ai/LAW/sis/Solution.jsp?qsetId=Lw5e7noFooA=&qsetName=LST Mock 20 2021 (CLAT) 124/208


12/18/2020 Mock Analysis

b B can keep both the house and Rs. 15 lakh.

c B can keep both the property and the bene t upon paying C Rs. 10 lakh.

d B can keep both the property and the bene t upon paying C Rs. 15 lakh.

Solution:
 Answer key/Solution
Correct Answer : c
Your Answer : a
As stated in the passage, if the transferor died before the owner could make
election, the owner is exempted from making the election and can keep both his own property and the bene t
transferred by the transferor by paying the disappointed transferee the value of the property. Here, the value of
the property is Rs. 10 lakh, and B can keep both the bene t and property upon paying C the value of the
property.

FeedBack Bookmark

https://www.aspiration.ai/LAW/sis/Solution.jsp?qsetId=Lw5e7noFooA=&qsetName=LST Mock 20 2021 (CLAT) 125/208


12/18/2020 Mock Analysis

Directions for questions 67 to 105: You have been given some passages followed by questions based on each
passage. You are required to choose the most appropriate option which follows from the passage. Only the
information given in the passage should be used for choosing the answer and no external knowledge of law
howsoever prominent is to be applied.

Passage – 3

Doctrine of Election has its source in Section 3 of the transfer of property act 1882 and sections 180-190 of
Indian succession act. Election means a choice between two alternative or con icting rights and is based on the
principle that the person taking the bene t should also bear the burden. It states that when a party transfers a
property over which he does not hold any right of transfer and entailed in that transaction is the bene t
conferred upon the original owner of the property, such owner must elect his option to either validate such
transfer of property or reject it; upon rejection, the bene t shall be reverted back to the transferor. This rule is
subject to two limitations, in which case the person who is supposed to elect either of the options can refuse to
do so, keep both his property and the bene t and can make good to the disappointed transferee by paying him
the amount or market value of the property that was attempted to be transferred to him. This exception is
available in two cases:

1. Where the transfer has been through gratuitous means and the transferor has died or become incapable of
making a fresh transfer.
2. In all cases where the transfer is for consideration.
The election by the owner can either be direct or indirect. In direct election, it is simply through communication
about the elected choice or option. Bene t shall be deemed to be accepted by the person who is supposed to
elect, if he is aware of his duty to elect and has enjoyed the bene t for two years without doing any act to
express dissent or, has enjoyed the property in such a way that it is impossible for the transferee to enjoy the
property in the same way, for example, by exhausting its resources.

There are certain quali cations to this doctrine. When the owner who is considering the election between
retaining the property and accepting a particular bene t, chooses the former, he is not bound to relinquish any
other additional bene t that he gains through the transaction.

Q.78 [30465398]
A and B jointly owned a house. A transferred B's share in the house to C and through the same instrument,
transferred Rs.15 lakh to B in lieu of his share. Additionally, he gifted B his own share in the house. B refuses to
take the Rs.15 lakh and retains his share in the house. Can he be compelled to give up A's share in the house?

a Yes, as he cannot both keep the property and take the bene t.

b Yes, but only upon paying C the value of his share in the house.

c No, as he has relinquished the bene t conferred upon him by refusing to take the Rs. 15 lakh.

d No, as he is not bound to relinquish the additional bene t.

https://www.aspiration.ai/LAW/sis/Solution.jsp?qsetId=Lw5e7noFooA=&qsetName=LST Mock 20 2021 (CLAT) 126/208


12/18/2020 Mock Analysis

Solution:
 Answer key/Solution
Correct Answer : d
Your Answer : a
The Rs. 15 lakh refused by B were in lieu of his own share in the house. By
electing to refuse that amount, he retains his own share in the house. However, A made an additional gift of
his own share in the house to B, and as stated in the passage, he is not bound to relinquish such other bene t
conferred upon him.
FeedBack Bookmark

Directions for questions 67 to 105: You have been given some passages followed by questions based on each
passage. You are required to choose the most appropriate option which follows from the passage. Only the
information given in the passage should be used for choosing the answer and no external knowledge of law
howsoever prominent is to be applied.

Passage – 3

Doctrine of Election has its source in Section 3 of the transfer of property act 1882 and sections 180-190 of
Indian succession act. Election means a choice between two alternative or con icting rights and is based on the
principle that the person taking the bene t should also bear the burden. It states that when a party transfers a
property over which he does not hold any right of transfer and entailed in that transaction is the bene t
conferred upon the original owner of the property, such owner must elect his option to either validate such
transfer of property or reject it; upon rejection, the bene t shall be reverted back to the transferor. This rule is
subject to two limitations, in which case the person who is supposed to elect either of the options can refuse to
do so, keep both his property and the bene t and can make good to the disappointed transferee by paying him
the amount or market value of the property that was attempted to be transferred to him. This exception is
available in two cases:

1. Where the transfer has been through gratuitous means and the transferor has died or become incapable of
making a fresh transfer.
2. In all cases where the transfer is for consideration.
The election by the owner can either be direct or indirect. In direct election, it is simply through communication
about the elected choice or option. Bene t shall be deemed to be accepted by the person who is supposed to
elect, if he is aware of his duty to elect and has enjoyed the bene t for two years without doing any act to
express dissent or, has enjoyed the property in such a way that it is impossible for the transferee to enjoy the
property in the same way, for example, by exhausting its resources.

There are certain quali cations to this doctrine. When the owner who is considering the election between
retaining the property and accepting a particular bene t, chooses the former, he is not bound to relinquish any
other additional bene t that he gains through the transaction.

https://www.aspiration.ai/LAW/sis/Solution.jsp?qsetId=Lw5e7noFooA=&qsetName=LST Mock 20 2021 (CLAT) 127/208


12/18/2020 Mock Analysis

Q.79 [30465398]
A transferred a house owned by B worth Rs.10 lakh, to C and through the same instrument, transferred some
coal mines owned by him to B in lieu of his house. B did not directly elect either of the properties, but took
possession of coal mines and extracted all the coal from them within one year from the transfer. After one year
had passed, he communicated to A and C that he wished to elect the house and relinquish the mines. Can B
elect to retain his property at this point?

a Yes, as he has not yet enjoyed the bene t for two years.

b No, as it is impossible for C to enjoy the coal mines now.

c No, as B has only enjoyed the additional bene t from the transaction.

d Yes, but only by paying C the value of the coal mines.

Solution:
 Answer key/Solution
Correct Answer : b
When the owner of the property enjoys the bene t in a way that makes it
impossible for the transferee to enjoy it in the same way, he is deemed to have
elected the bene t and relinquished the property. Thus, option (b) is correct.

FeedBack Bookmark

https://www.aspiration.ai/LAW/sis/Solution.jsp?qsetId=Lw5e7noFooA=&qsetName=LST Mock 20 2021 (CLAT) 128/208


12/18/2020 Mock Analysis

Directions for questions 67 to 105: You have been given some passages followed by questions based on each
passage. You are required to choose the most appropriate option which follows from the passage. Only the
information given in the passage should be used for choosing the answer and no external knowledge of law
howsoever prominent is to be applied.

Passage – 3

Doctrine of Election has its source in Section 3 of the transfer of property act 1882 and sections 180-190 of
Indian succession act. Election means a choice between two alternative or con icting rights and is based on the
principle that the person taking the bene t should also bear the burden. It states that when a party transfers a
property over which he does not hold any right of transfer and entailed in that transaction is the bene t
conferred upon the original owner of the property, such owner must elect his option to either validate such
transfer of property or reject it; upon rejection, the bene t shall be reverted back to the transferor. This rule is
subject to two limitations, in which case the person who is supposed to elect either of the options can refuse to
do so, keep both his property and the bene t and can make good to the disappointed transferee by paying him
the amount or market value of the property that was attempted to be transferred to him. This exception is
available in two cases:

1. Where the transfer has been through gratuitous means and the transferor has died or become incapable of
making a fresh transfer.
2. In all cases where the transfer is for consideration.
The election by the owner can either be direct or indirect. In direct election, it is simply through communication
about the elected choice or option. Bene t shall be deemed to be accepted by the person who is supposed to
elect, if he is aware of his duty to elect and has enjoyed the bene t for two years without doing any act to
express dissent or, has enjoyed the property in such a way that it is impossible for the transferee to enjoy the
property in the same way, for example, by exhausting its resources.

There are certain quali cations to this doctrine. When the owner who is considering the election between
retaining the property and accepting a particular bene t, chooses the former, he is not bound to relinquish any
other additional bene t that he gains through the transaction.

Q.80 [30465398]
A sold B's house to C, for which C paid fteen lakh rupees to A. The market value of the house was ten lakh
rupees. Through the same instrument, A transferred some other property in B's name in lieu of his house. Which
of the following is not a course of action available to B in this case?

a To choose the house and relinquish the property conferred by A.

b To relinquish the house and take the property given by A.

c To keep both the house and property and pay C fteen lakh rupees.

d To keep both the house and property and pay C ten lakh rupees.

https://www.aspiration.ai/LAW/sis/Solution.jsp?qsetId=Lw5e7noFooA=&qsetName=LST Mock 20 2021 (CLAT) 129/208


12/18/2020 Mock Analysis

Solution:
 Answer key/Solution
Correct Answer : c
As the transfer is for a consideration, B can compensate C, the disappointed
transferee with the amount equal to the market value of the property and take
both the property and keep his house.

FeedBack Bookmark

Directions for questions 67 to 105: You have been given some passages followed by questions based on each
passage. You are required to choose the most appropriate option which follows from the passage. Only the
information given in the passage should be used for choosing the answer and no external knowledge of law
howsoever prominent is to be applied.

Passage – 3

Doctrine of Election has its source in Section 3 of the transfer of property act 1882 and sections 180-190 of
Indian succession act. Election means a choice between two alternative or con icting rights and is based on the
principle that the person taking the bene t should also bear the burden. It states that when a party transfers a
property over which he does not hold any right of transfer and entailed in that transaction is the bene t
conferred upon the original owner of the property, such owner must elect his option to either validate such
transfer of property or reject it; upon rejection, the bene t shall be reverted back to the transferor. This rule is
subject to two limitations, in which case the person who is supposed to elect either of the options can refuse to
do so, keep both his property and the bene t and can make good to the disappointed transferee by paying him
the amount or market value of the property that was attempted to be transferred to him. This exception is
available in two cases:

1. Where the transfer has been through gratuitous means and the transferor has died or become incapable of
making a fresh transfer.
2. In all cases where the transfer is for consideration.
The election by the owner can either be direct or indirect. In direct election, it is simply through communication
about the elected choice or option. Bene t shall be deemed to be accepted by the person who is supposed to
elect, if he is aware of his duty to elect and has enjoyed the bene t for two years without doing any act to
express dissent or, has enjoyed the property in such a way that it is impossible for the transferee to enjoy the
property in the same way, for example, by exhausting its resources.

There are certain quali cations to this doctrine. When the owner who is considering the election between
retaining the property and accepting a particular bene t, chooses the former, he is not bound to relinquish any
other additional bene t that he gains through the transaction.

Q.81 [30465398]
In the case above, B did not elect either of the choices for three years after the transfer made A, in spite of being
aware of the same. After three years, C changed his mind and tried to take the properties back and rescind the
sale of B's house with C. Can A do so?

a Yes, as upon rejection, the bene t must revert back to the transferor.

https://www.aspiration.ai/LAW/sis/Solution.jsp?qsetId=Lw5e7noFooA=&qsetName=LST Mock 20 2021 (CLAT) 130/208


12/18/2020 Mock Analysis

b Yes, as the transfer was for compensation and is exempt from election by the owner.

c No, as the bene t is deemed to be elected by B after expiry of two years.

d No, as B has enjoyed the property in a way that A cannot do it in the same way.

Solution:
 Answer key/Solution
Correct Answer : c
Indirect election means that without directly electing, B is deemed to have taken
the bene t if he does not deny it within two years. As he has not done so in this
case, the properties belong to B and A cannot take them back.

FeedBack Bookmark

Directions for questions 67 to 105: You have been given some passages followed by questions based on each
passage. You are required to choose the most appropriate option which follows from the passage. Only the
information given in the passage should be used for choosing the answer and no external knowledge of law
howsoever prominent is to be applied.

Passage – 4

The term injunction has been de ned as "a judicial process whereby a party is required to do, or refrain from
doing, any particular act." Injunctions could be classi ed as prohibitory or mandatory. Prohibitory is when one of
the parties is ordered to restrain from doing a wrongful act, which would infringe some legal or equitable right of
the Plaintiff. Mandatory is where a party is actually compelled to do something. The requirements for granting
of a temporary injunction, i.e. injunction before the decision of the case is made on merit, are: prima facie case,
balance of convenience and irreparable injury.

Mareva Injunction means injunctions that restrain the Defendant from removing assets from the jurisdiction or
dealing with or parting with the assets in such a way as to render the execution of the decree if given in favor of
the Plaintiff a mere brutum fulmen (an ineffectual legal judgment). It is granted to maintain status quo till the
Court adjudicates upon the rights and liabilities of the parties. Mareva injunction is mainly used to aid in the
recovery of debts so as to help the creditor by preventing the debtor from absconding or disposing of his assets
overseas. The three conditions required to be ful lled before issuing this injunction are:
(1) The Plaintiff needs to have a legal or an equitable right justiciable in Court. Evidence needs to be shown that
the Defendant is trying to dispose of his assets so as to defeat the Judgement.
(2) In case the injunction is not granted, it would cause an irreparable loss to the Plaintiff;
(3) The balance of convenience should lie with the Plaintiff.

Q.82 [30465398]
In a dispute between A and B regarding the line of boundary between their lands, the court decided the boundary
line and ordered that A was injuncted from allowing his cattle pass the delineated line and must compensate for
any loss that may be caused to B's crops due to such crossing of the cattle. What is the nature of the injunction
given by the court?

a Mandatory temporary injunction.

https://www.aspiration.ai/LAW/sis/Solution.jsp?qsetId=Lw5e7noFooA=&qsetName=LST Mock 20 2021 (CLAT) 131/208


12/18/2020 Mock Analysis

b Mandatory permanent injunction.

c ProhibItory temporary injunction.

d Both mandatory and prohibitory injunction.

Solution:
 Answer key/Solution
Correct Answer : d
Your Answer : c
As A is ordered to do something and also restrained from doing another act, the
injunction is both mandatory and prohibitory in nature.

FeedBack Bookmark

Directions for questions 67 to 105: You have been given some passages followed by questions based on each
passage. You are required to choose the most appropriate option which follows from the passage. Only the
information given in the passage should be used for choosing the answer and no external knowledge of law
howsoever prominent is to be applied.

Passage – 4

The term injunction has been de ned as "a judicial process whereby a party is required to do, or refrain from
doing, any particular act." Injunctions could be classi ed as prohibitory or mandatory. Prohibitory is when one of
the parties is ordered to restrain from doing a wrongful act, which would infringe some legal or equitable right of
the Plaintiff. Mandatory is where a party is actually compelled to do something. The requirements for granting
of a temporary injunction, i.e. injunction before the decision of the case is made on merit, are: prima facie case,
balance of convenience and irreparable injury.

Mareva Injunction means injunctions that restrain the Defendant from removing assets from the jurisdiction or
dealing with or parting with the assets in such a way as to render the execution of the decree if given in favor of
the Plaintiff a mere brutum fulmen (an ineffectual legal judgment). It is granted to maintain status quo till the
Court adjudicates upon the rights and liabilities of the parties. Mareva injunction is mainly used to aid in the
recovery of debts so as to help the creditor by preventing the debtor from absconding or disposing of his assets
overseas. The three conditions required to be ful lled before issuing this injunction are:
(1) The Plaintiff needs to have a legal or an equitable right justiciable in Court. Evidence needs to be shown that
the Defendant is trying to dispose of his assets so as to defeat the Judgement.
(2) In case the injunction is not granted, it would cause an irreparable loss to the Plaintiff;
(3) The balance of convenience should lie with the Plaintiff.

Q.83 [30465398]
In the above case, while the suit proceedings were going on in the court, B argued that A's cattle grazed freely in
his land and caused a lot of damage to his crops. B sought a temporary injunction against the same. Which of
the following is not an argument on A's part for opposing the injunction sought by B?

https://www.aspiration.ai/LAW/sis/Solution.jsp?qsetId=Lw5e7noFooA=&qsetName=LST Mock 20 2021 (CLAT) 132/208


12/18/2020 Mock Analysis

a That B has not proved a prima facie case for his right on his land.

b That it would be more convenient to let his cattle graze freely on B's land.

c That no injunction can be granted until the case is heard on merit.

d That there is no irreparable injury being caused to B by allowing A's cattle to graze in his land.

Solution:
 Answer key/Solution
Correct Answer : c
Your Answer : c
As all the other three options provide arguments which nullify the requirements
for granting a temporary injunction, option (c) is the correct answer.

FeedBack Bookmark

Directions for questions 67 to 105: You have been given some passages followed by questions based on each
passage. You are required to choose the most appropriate option which follows from the passage. Only the
information given in the passage should be used for choosing the answer and no external knowledge of law
howsoever prominent is to be applied.

Passage – 4

The term injunction has been de ned as "a judicial process whereby a party is required to do, or refrain from
doing, any particular act." Injunctions could be classi ed as prohibitory or mandatory. Prohibitory is when one of
the parties is ordered to restrain from doing a wrongful act, which would infringe some legal or equitable right of
the Plaintiff. Mandatory is where a party is actually compelled to do something. The requirements for granting
of a temporary injunction, i.e. injunction before the decision of the case is made on merit, are: prima facie case,
balance of convenience and irreparable injury.

Mareva Injunction means injunctions that restrain the Defendant from removing assets from the jurisdiction or
dealing with or parting with the assets in such a way as to render the execution of the decree if given in favor of
the Plaintiff a mere brutum fulmen (an ineffectual legal judgment). It is granted to maintain status quo till the
Court adjudicates upon the rights and liabilities of the parties. Mareva injunction is mainly used to aid in the
recovery of debts so as to help the creditor by preventing the debtor from absconding or disposing of his assets
overseas. The three conditions required to be ful lled before issuing this injunction are:
(1) The Plaintiff needs to have a legal or an equitable right justiciable in Court. Evidence needs to be shown that
the Defendant is trying to dispose of his assets so as to defeat the Judgement.
(2) In case the injunction is not granted, it would cause an irreparable loss to the Plaintiff;
(3) The balance of convenience should lie with the Plaintiff.

https://www.aspiration.ai/LAW/sis/Solution.jsp?qsetId=Lw5e7noFooA=&qsetName=LST Mock 20 2021 (CLAT) 133/208


12/18/2020 Mock Analysis

Q.84 [30465398]
The court was to adjudicate upon a dispute between A and B regarding A's liability for repaying a loan of ve
lakhs taken by him from B. A owns several properties within the city amounting to more than fty lakhs.
Additionally, he owns several properties outside of the city. While the case is being decided, A enters into a
contract with C for the sale of one of his farmhouses out of the city. Can B successfully apply for a Mareva
injunction to be granted to stop the sale?

a Yes, as he has a legal right to recover his loan amount from A.

b No, as selling the farmhouse will not render the decree ineffective even if made in B's favour.

c Yes, as if it is more convenient to recover the debt if A does not sell the farmhouse.

d No, as the farmhouse is not within the jurisdiction of the court.

Solution:
 Answer key/Solution
Correct Answer : b
Your Answer : b
As the amount of A's properties far exceeds his liability, there is no possibility that
a decree for payment of debt in B's favour will be ineffectual even after A has sold the farmhouse. Therefore,
an injunction will not be granted in B's favour.

FeedBack Bookmark

https://www.aspiration.ai/LAW/sis/Solution.jsp?qsetId=Lw5e7noFooA=&qsetName=LST Mock 20 2021 (CLAT) 134/208


12/18/2020 Mock Analysis

Directions for questions 67 to 105: You have been given some passages followed by questions based on each
passage. You are required to choose the most appropriate option which follows from the passage. Only the
information given in the passage should be used for choosing the answer and no external knowledge of law
howsoever prominent is to be applied.

Passage – 4

The term injunction has been de ned as "a judicial process whereby a party is required to do, or refrain from
doing, any particular act." Injunctions could be classi ed as prohibitory or mandatory. Prohibitory is when one of
the parties is ordered to restrain from doing a wrongful act, which would infringe some legal or equitable right of
the Plaintiff. Mandatory is where a party is actually compelled to do something. The requirements for granting
of a temporary injunction, i.e. injunction before the decision of the case is made on merit, are: prima facie case,
balance of convenience and irreparable injury.

Mareva Injunction means injunctions that restrain the Defendant from removing assets from the jurisdiction or
dealing with or parting with the assets in such a way as to render the execution of the decree if given in favor of
the Plaintiff a mere brutum fulmen (an ineffectual legal judgment). It is granted to maintain status quo till the
Court adjudicates upon the rights and liabilities of the parties. Mareva injunction is mainly used to aid in the
recovery of debts so as to help the creditor by preventing the debtor from absconding or disposing of his assets
overseas. The three conditions required to be ful lled before issuing this injunction are:
(1) The Plaintiff needs to have a legal or an equitable right justiciable in Court. Evidence needs to be shown that
the Defendant is trying to dispose of his assets so as to defeat the Judgement.
(2) In case the injunction is not granted, it would cause an irreparable loss to the Plaintiff;
(3) The balance of convenience should lie with the Plaintiff.

Q.85 [30465398]
The court was to adjudicate upon a dispute between A and B regarding A's liability for repaying a loan of ve
lakhs taken by him from B. Which of the following facts, if proven, support B's application for granting of a
Mareva injunction?

a A is planning to leave the country for a pre-scheduled business trip.

b The total amount of A's assets is less than ve lakh rupees.

c A is planning to le for bankruptcy.

d A is planning to shift his entire business to another country.

https://www.aspiration.ai/LAW/sis/Solution.jsp?qsetId=Lw5e7noFooA=&qsetName=LST Mock 20 2021 (CLAT) 135/208


12/18/2020 Mock Analysis

Solution:
 Answer key/Solution
Correct Answer : d
Your Answer : b
Other than shifting of the business overseas, none of the options speak of an
action which may amount to a situation where a decree in B's favour will be inef fectual. Option (a) cannot
necessarily amount to absconding, while option (c) does not exempt A from the liability to pay his debts to B.
Thus, option (d) is correct.
FeedBack Bookmark

Directions for questions 67 to 105: You have been given some passages followed by questions based on each
passage. You are required to choose the most appropriate option which follows from the passage. Only the
information given in the passage should be used for choosing the answer and no external knowledge of law
howsoever prominent is to be applied.

Passage – 4

The term injunction has been de ned as "a judicial process whereby a party is required to do, or refrain from
doing, any particular act." Injunctions could be classi ed as prohibitory or mandatory. Prohibitory is when one of
the parties is ordered to restrain from doing a wrongful act, which would infringe some legal or equitable right of
the Plaintiff. Mandatory is where a party is actually compelled to do something. The requirements for granting
of a temporary injunction, i.e. injunction before the decision of the case is made on merit, are: prima facie case,
balance of convenience and irreparable injury.

Mareva Injunction means injunctions that restrain the Defendant from removing assets from the jurisdiction or
dealing with or parting with the assets in such a way as to render the execution of the decree if given in favor of
the Plaintiff a mere brutum fulmen (an ineffectual legal judgment). It is granted to maintain status quo till the
Court adjudicates upon the rights and liabilities of the parties. Mareva injunction is mainly used to aid in the
recovery of debts so as to help the creditor by preventing the debtor from absconding or disposing of his assets
overseas. The three conditions required to be ful lled before issuing this injunction are:
(1) The Plaintiff needs to have a legal or an equitable right justiciable in Court. Evidence needs to be shown that
the Defendant is trying to dispose of his assets so as to defeat the Judgement.
(2) In case the injunction is not granted, it would cause an irreparable loss to the Plaintiff;
(3) The balance of convenience should lie with the Plaintiff.

Q.86 [30465398]
In the case above, suppose A planned to rent out his sole property to C, in order to recover rent. B led for a
Mareva injunction against such rental agreement. Which of the following can be the reason for non-granting of
such injunction?

a B does not have any legal right on A's property.

b Non-granting of injunction will cause no irreparable injury to B.

c Status quo must be maintained till the court adjudicates upon rights of the parties.

https://www.aspiration.ai/LAW/sis/Solution.jsp?qsetId=Lw5e7noFooA=&qsetName=LST Mock 20 2021 (CLAT) 136/208


12/18/2020 Mock Analysis

d B has not proven a prima facie case for granting of an injunction in his favour.

Solution:
 Answer key/Solution
Correct Answer : b
Your Answer : b
As A is merely renting the property out, there is no possibility of irreparable injury
in case injunction is not granted. Thus, option (b) is correct.

FeedBack Bookmark

https://www.aspiration.ai/LAW/sis/Solution.jsp?qsetId=Lw5e7noFooA=&qsetName=LST Mock 20 2021 (CLAT) 137/208


12/18/2020 Mock Analysis

Directions for questions 67 to 105: You have been given some passages followed by questions based on each
passage. You are required to choose the most appropriate option which follows from the passage. Only the
information given in the passage should be used for choosing the answer and no external knowledge of law
howsoever prominent is to be applied.

Passage – 5

Commercial globalisation has resulted in giving a big boost to our country. With the rapid increase in commerce
and trade, use of cheque also increased and so did the cheque bouncing disputes. The object of Sections 138-
142 of the Negotiable Instruments Act, 1881 is to promote the e cacy of banking operations and to ensure
credibility in transacting business through cheques.

Section 138 casts a criminal liability punishable with imprisonment or ne or with both on a person who issues
a cheque towards discharge of a debt or liability as a whole or in part and the cheque is dishonoured by the
bank on presentation. There is no requirement of mens rea for constituting this offence. However, with a view to
avert unnecessary prosecution of an honest drawer of the cheque and with a view to give an opportunity to him
to make amends, the prosecution under Section 138 of the Act has been made subject to certain conditions.
These conditions are stipulated in the proviso to Section 138.

The ingredients of the offence under Section 138 are:


(a) cheque is drawn by the accused on an account maintained by him with a banker;
(b) the cheque amount is in discharge of a debt or liability; and
(c) the cheque is returned unpaid for insu ciency of funds or that the amount exceeds the arrangement made
with the bank, the offence standing committed the moment the cheque is returned unpaid.

There are three distinct conditions precedent, which must be satis ed before the offence becomes punishable.
(i) The cheque ought to have been presented to the bank within a period of 6 months from the date on which it
is drawn or within the period of its validity, whichever is earlier.
(ii) The payee or the holder in due course of the cheque, as the case may be, ought to make a demand for the
payment of the said amount of money by giving a notice in writing, to the drawer of the cheque, within 30 days
of the receipt of information by him from the bank regarding the return of the cheque as unpaid.
(iii) The drawer of such a cheque should have failed to make payment of the said amount of money to the payee
or as the case may be, to the holder in due course of the cheque within 15 days of the receipt of the said notice.
Section 139 carves out the presumption that the holder of a cheque has received it for the discharge of any
liability. Section 140 clari es that it will not be available as a defence to the drawer that he had no reason to
believe, when he issued the cheque, that it would be dishonoured.

Q.87 [30465398]
A owed a certain amount to B. He issued a cheque in B's name for Rs.5 lakh on 20th January, 2020 and told him
not to encash the cheque before the end of the month. B encashed the cheque on 29st of January, 2020 and it
was returned unpaid as A's account had only Rs. 500. Can A be prosecuted and punished under section 138 of
the NI Act?

a Yes, as A drew a cheque in discharge of a debt.

b Yes, as the cheque was returned due to insu ciency of funds in A's account.

c No, as conditions precedent have not been ful lled.

https://www.aspiration.ai/LAW/sis/Solution.jsp?qsetId=Lw5e7noFooA=&qsetName=LST Mock 20 2021 (CLAT) 138/208


12/18/2020 Mock Analysis

d Both (a) and (b).

Solution:
 Answer key/Solution
Correct Answer : c
Your Answer : a
The passage clearly states when the offence is constituted and when a person
can be prosecuted. As both the essentials given in option (a) and (b) only constitute offence, for prosecution
of it, conditions precedent to it must be ful lled. Therefore, option (c) is correct.
FeedBack Bookmark

https://www.aspiration.ai/LAW/sis/Solution.jsp?qsetId=Lw5e7noFooA=&qsetName=LST Mock 20 2021 (CLAT) 139/208


12/18/2020 Mock Analysis

Directions for questions 67 to 105: You have been given some passages followed by questions based on each
passage. You are required to choose the most appropriate option which follows from the passage. Only the
information given in the passage should be used for choosing the answer and no external knowledge of law
howsoever prominent is to be applied.

Passage – 5

Commercial globalisation has resulted in giving a big boost to our country. With the rapid increase in commerce
and trade, use of cheque also increased and so did the cheque bouncing disputes. The object of Sections 138-
142 of the Negotiable Instruments Act, 1881 is to promote the e cacy of banking operations and to ensure
credibility in transacting business through cheques.

Section 138 casts a criminal liability punishable with imprisonment or ne or with both on a person who issues
a cheque towards discharge of a debt or liability as a whole or in part and the cheque is dishonoured by the
bank on presentation. There is no requirement of mens rea for constituting this offence. However, with a view to
avert unnecessary prosecution of an honest drawer of the cheque and with a view to give an opportunity to him
to make amends, the prosecution under Section 138 of the Act has been made subject to certain conditions.
These conditions are stipulated in the proviso to Section 138.

The ingredients of the offence under Section 138 are:


(a) cheque is drawn by the accused on an account maintained by him with a banker;
(b) the cheque amount is in discharge of a debt or liability; and
(c) the cheque is returned unpaid for insu ciency of funds or that the amount exceeds the arrangement made
with the bank, the offence standing committed the moment the cheque is returned unpaid.

There are three distinct conditions precedent, which must be satis ed before the offence becomes punishable.
(i) The cheque ought to have been presented to the bank within a period of 6 months from the date on which it
is drawn or within the period of its validity, whichever is earlier.
(ii) The payee or the holder in due course of the cheque, as the case may be, ought to make a demand for the
payment of the said amount of money by giving a notice in writing, to the drawer of the cheque, within 30 days
of the receipt of information by him from the bank regarding the return of the cheque as unpaid.
(iii) The drawer of such a cheque should have failed to make payment of the said amount of money to the payee
or as the case may be, to the holder in due course of the cheque within 15 days of the receipt of the said notice.
Section 139 carves out the presumption that the holder of a cheque has received it for the discharge of any
liability. Section 140 clari es that it will not be available as a defence to the drawer that he had no reason to
believe, when he issued the cheque, that it would be dishonoured.

Q.88 [30465398]
A owed a certain amount to B. He issued a cheque in B's name for Rs.5 lakh on 20th January, 2020 and told him
not to encash the cheque before the end of the month. B encashed the cheque on 29th of January, 2020 and it
was returned unpaid as A's account had only Rs. 500. B demanded the money from A on 30th of January
through a written notice, which A received on 5th of February. B sought to prosecute A on 21st February. Which
of the following defences can A plead in order to save himself from prosecution?

a There was no criminal intention on his part

b B encashed the cheque before the time given by A.

https://www.aspiration.ai/LAW/sis/Solution.jsp?qsetId=Lw5e7noFooA=&qsetName=LST Mock 20 2021 (CLAT) 140/208


12/18/2020 Mock Analysis

c The conditions precedent for the prosecution are not ful lled.

d None of the above.

Solution:
 Answer key/Solution
Correct Answer : d
Your Answer : d
The passage lays down three ingredients and three conditions for prosecution
u/s. 138, which are all ful lled in the case above. Thus, none of the defences are valid and option (d) is
correct.

FeedBack Bookmark

https://www.aspiration.ai/LAW/sis/Solution.jsp?qsetId=Lw5e7noFooA=&qsetName=LST Mock 20 2021 (CLAT) 141/208


12/18/2020 Mock Analysis

Directions for questions 67 to 105: You have been given some passages followed by questions based on each
passage. You are required to choose the most appropriate option which follows from the passage. Only the
information given in the passage should be used for choosing the answer and no external knowledge of law
howsoever prominent is to be applied.

Passage – 5

Commercial globalisation has resulted in giving a big boost to our country. With the rapid increase in commerce
and trade, use of cheque also increased and so did the cheque bouncing disputes. The object of Sections 138-
142 of the Negotiable Instruments Act, 1881 is to promote the e cacy of banking operations and to ensure
credibility in transacting business through cheques.

Section 138 casts a criminal liability punishable with imprisonment or ne or with both on a person who issues
a cheque towards discharge of a debt or liability as a whole or in part and the cheque is dishonoured by the
bank on presentation. There is no requirement of mens rea for constituting this offence. However, with a view to
avert unnecessary prosecution of an honest drawer of the cheque and with a view to give an opportunity to him
to make amends, the prosecution under Section 138 of the Act has been made subject to certain conditions.
These conditions are stipulated in the proviso to Section 138.

The ingredients of the offence under Section 138 are:


(a) cheque is drawn by the accused on an account maintained by him with a banker;
(b) the cheque amount is in discharge of a debt or liability; and
(c) the cheque is returned unpaid for insu ciency of funds or that the amount exceeds the arrangement made
with the bank, the offence standing committed the moment the cheque is returned unpaid.

There are three distinct conditions precedent, which must be satis ed before the offence becomes punishable.
(i) The cheque ought to have been presented to the bank within a period of 6 months from the date on which it
is drawn or within the period of its validity, whichever is earlier.
(ii) The payee or the holder in due course of the cheque, as the case may be, ought to make a demand for the
payment of the said amount of money by giving a notice in writing, to the drawer of the cheque, within 30 days
of the receipt of information by him from the bank regarding the return of the cheque as unpaid.
(iii) The drawer of such a cheque should have failed to make payment of the said amount of money to the payee
or as the case may be, to the holder in due course of the cheque within 15 days of the receipt of the said notice.
Section 139 carves out the presumption that the holder of a cheque has received it for the discharge of any
liability. Section 140 clari es that it will not be available as a defence to the drawer that he had no reason to
believe, when he issued the cheque, that it would be dishonoured.

Q.89 [30465398]
In the case above, which of the following facts can save A from the prosecution under the NI Act?

a A's joint account holder had withdrawn Rs.5 lakhs on 29th of January without telling A.

b A had no means of knowing the actual balance amount in his bank account.

c The cheque was a gift given by A to B.

d B had promised A that he would not encash the cheque before the end of the month.

https://www.aspiration.ai/LAW/sis/Solution.jsp?qsetId=Lw5e7noFooA=&qsetName=LST Mock 20 2021 (CLAT) 142/208


12/18/2020 Mock Analysis

Solution:
 Answer key/Solution
Correct Answer : c
Your Answer : d
Only option (c) shows that an ingredient for sec. 138 was not ful lled, i.e. the
cheque was not for discharge of any liability but was for a gift and thus it would be a good defence.
FeedBack Bookmark

Directions for questions 67 to 105: You have been given some passages followed by questions based on each
passage. You are required to choose the most appropriate option which follows from the passage. Only the
information given in the passage should be used for choosing the answer and no external knowledge of law
howsoever prominent is to be applied.

Passage – 5

Commercial globalisation has resulted in giving a big boost to our country. With the rapid increase in commerce
and trade, use of cheque also increased and so did the cheque bouncing disputes. The object of Sections 138-
142 of the Negotiable Instruments Act, 1881 is to promote the e cacy of banking operations and to ensure
credibility in transacting business through cheques.

Section 138 casts a criminal liability punishable with imprisonment or ne or with both on a person who issues
a cheque towards discharge of a debt or liability as a whole or in part and the cheque is dishonoured by the
bank on presentation. There is no requirement of mens rea for constituting this offence. However, with a view to
avert unnecessary prosecution of an honest drawer of the cheque and with a view to give an opportunity to him
to make amends, the prosecution under Section 138 of the Act has been made subject to certain conditions.
These conditions are stipulated in the proviso to Section 138.

The ingredients of the offence under Section 138 are:


(a) cheque is drawn by the accused on an account maintained by him with a banker;
(b) the cheque amount is in discharge of a debt or liability; and
(c) the cheque is returned unpaid for insu ciency of funds or that the amount exceeds the arrangement made
with the bank, the offence standing committed the moment the cheque is returned unpaid.

There are three distinct conditions precedent, which must be satis ed before the offence becomes punishable.
(i) The cheque ought to have been presented to the bank within a period of 6 months from the date on which it
is drawn or within the period of its validity, whichever is earlier.
(ii) The payee or the holder in due course of the cheque, as the case may be, ought to make a demand for the
payment of the said amount of money by giving a notice in writing, to the drawer of the cheque, within 30 days
of the receipt of information by him from the bank regarding the return of the cheque as unpaid.
(iii) The drawer of such a cheque should have failed to make payment of the said amount of money to the payee
or as the case may be, to the holder in due course of the cheque within 15 days of the receipt of the said notice.
Section 139 carves out the presumption that the holder of a cheque has received it for the discharge of any
liability. Section 140 clari es that it will not be available as a defence to the drawer that he had no reason to
believe, when he issued the cheque, that it would be dishonoured.

https://www.aspiration.ai/LAW/sis/Solution.jsp?qsetId=Lw5e7noFooA=&qsetName=LST Mock 20 2021 (CLAT) 143/208


12/18/2020 Mock Analysis

Q.90 [30465398]
A gave a cheque for his account at the Indian Bank to B, for an amount of Rs. 5 lakhs on 20th of January.
However, upon presentation of the cheque at the bank, it was returned unpaid as A had closed his account at
that bank on 19th of January. Has the offence under sec. 138 been constituted?

a Yes, as the cheque was drawn by A on his account maintained at the India Bank.

b No, as the cheque was not drawn on an account maintained by A.

c No, as the cheque was not returned for insu ciency of funds.

d Both (b) and (c).

Solution:
 Answer key/Solution
Correct Answer : d
Your Answer : d
When A drew the cheque, he had already closed the account, thus the rst
ingredient is not ful lled and option (b) is correct. The reason for the return of the cheque was also not
insu ciency of funds per se, thus option (c) is also correct.
FeedBack Bookmark

https://www.aspiration.ai/LAW/sis/Solution.jsp?qsetId=Lw5e7noFooA=&qsetName=LST Mock 20 2021 (CLAT) 144/208


12/18/2020 Mock Analysis

Directions for questions 67 to 105: You have been given some passages followed by questions based on each
passage. You are required to choose the most appropriate option which follows from the passage. Only the
information given in the passage should be used for choosing the answer and no external knowledge of law
howsoever prominent is to be applied.

Passage – 5

Commercial globalisation has resulted in giving a big boost to our country. With the rapid increase in commerce
and trade, use of cheque also increased and so did the cheque bouncing disputes. The object of Sections 138-
142 of the Negotiable Instruments Act, 1881 is to promote the e cacy of banking operations and to ensure
credibility in transacting business through cheques.

Section 138 casts a criminal liability punishable with imprisonment or ne or with both on a person who issues
a cheque towards discharge of a debt or liability as a whole or in part and the cheque is dishonoured by the
bank on presentation. There is no requirement of mens rea for constituting this offence. However, with a view to
avert unnecessary prosecution of an honest drawer of the cheque and with a view to give an opportunity to him
to make amends, the prosecution under Section 138 of the Act has been made subject to certain conditions.
These conditions are stipulated in the proviso to Section 138.

The ingredients of the offence under Section 138 are:


(a) cheque is drawn by the accused on an account maintained by him with a banker;
(b) the cheque amount is in discharge of a debt or liability; and
(c) the cheque is returned unpaid for insu ciency of funds or that the amount exceeds the arrangement made
with the bank, the offence standing committed the moment the cheque is returned unpaid.

There are three distinct conditions precedent, which must be satis ed before the offence becomes punishable.
(i) The cheque ought to have been presented to the bank within a period of 6 months from the date on which it
is drawn or within the period of its validity, whichever is earlier.
(ii) The payee or the holder in due course of the cheque, as the case may be, ought to make a demand for the
payment of the said amount of money by giving a notice in writing, to the drawer of the cheque, within 30 days
of the receipt of information by him from the bank regarding the return of the cheque as unpaid.
(iii) The drawer of such a cheque should have failed to make payment of the said amount of money to the payee
or as the case may be, to the holder in due course of the cheque within 15 days of the receipt of the said notice.
Section 139 carves out the presumption that the holder of a cheque has received it for the discharge of any
liability. Section 140 clari es that it will not be available as a defence to the drawer that he had no reason to
believe, when he issued the cheque, that it would be dishonoured.

Q.91 [30465398]
A was B's tenant. As he was leaving town, on 1st of January, he wrote twelve cheques to his landlord B, for the
payment of rent. He put the rst day of each month as the drawing date. For the rent of July, B encashed the
cheque drawn on 1st of July on 2nd of July, which was returned for insu ciency of funds. Has the offence
under sec. 138 been constituted?

a No, as the amount was not in discharge of a debt or liability.

b No, as B encashed the cheque after the expiry of six months from when it was drawn.

c Yes, as the cheque was returned for insu ciency of funds.

https://www.aspiration.ai/LAW/sis/Solution.jsp?qsetId=Lw5e7noFooA=&qsetName=LST Mock 20 2021 (CLAT) 145/208


12/18/2020 Mock Analysis

d Both (a) and (b).

Solution:
 Answer key/Solution
Correct Answer : c
Your Answer : c
The cheque was drawn on the rst of July and it is immaterial when it was written.
Payment of rent is a liability. Thus, options (a), (b) and (d) are incorrect.
FeedBack Bookmark

Directions for questions 67 to 105: You have been given some passages followed by questions based on each
passage. You are required to choose the most appropriate option which follows from the passage. Only the
information given in the passage should be used for choosing the answer and no external knowledge of law
howsoever prominent is to be applied.

Passage – 6

The Public Trust Doctrine primarily rests on the principle that certain resources like air, sea, waters and the
forests have such a great importance to the people as a whole that it would be wholly unjusti ed to make them
a subject of private ownership. The said resources being a gift of nature, they should be made freely available to
everyone irrespective of the status in life. The doctrine enjoins upon the Government to protect the resources as
trustees for the enjoyment of the general public rather than to permit their use for private ownership or
commercial purposes.

Three types of restrictions on governmental authority are often thought to be imposed by the Public Trust: rst,
the property subject to the trust must not only be used for a public purpose, but it must be held available for use
by the general public; second, the property may not be sold, even for a fair cash equivalent; and third the
property must be maintained for particular types of uses.

Our legal system- based on English Common law- includes the Public Trust Doctrine as a part of its
jurisprudence. The State is the trustee of all-natural resources which are by nature meant for public use and
enjoyment. Public at large is the bene ciary of the seashore, running waters, airs, forests and ecologically fragile
lands. The State as a trustee is under a legal duty to protect the natural resources; these resources are meant
for public use and cannot be converted into private ownership. The aesthetic use and the pristine glory cannot
be permitted to be eroded for private, commercial or any other use unless the courts nd it necessary in good
faith, for public good and in public interest to encroach upon the said resource.

Q.92 [30465398]
Oscorp is a company engaged in building luxury hotels at tourist destinations. It applied for a ninety-nineyear
lease to the government of Assam, to build a ve-star hotel on the banks of the river Brahmaputra. The
proposed site of the hotel is currently a reserved forest. The environmentalist groups in the country protested
against the grant of such lease. Which of the following would be a strong argument for the environmentalists?

a That water is a gift of nature and should be available to all.

https://www.aspiration.ai/LAW/sis/Solution.jsp?qsetId=Lw5e7noFooA=&qsetName=LST Mock 20 2021 (CLAT) 146/208


12/18/2020 Mock Analysis

b That government cannot sell the land subject to trust.

c That the reserved forest should be freely accessible to the general public.

d That the government has a legal duty to protect the forest.

Solution:
 Answer key/Solution
Correct Answer : d
Your Answer : c
As stated by the paragraph, the property in question which is the subject of trust
here is the forest and under the PTD, the government has a duty to protect the same. Although option (c) is
factually true, the granting of lease would not restrict public access to the forest and thus would not be a
strong argument.
FeedBack Bookmark

Directions for questions 67 to 105: You have been given some passages followed by questions based on each
passage. You are required to choose the most appropriate option which follows from the passage. Only the
information given in the passage should be used for choosing the answer and no external knowledge of law
howsoever prominent is to be applied.

Passage – 6

The Public Trust Doctrine primarily rests on the principle that certain resources like air, sea, waters and the
forests have such a great importance to the people as a whole that it would be wholly unjusti ed to make them
a subject of private ownership. The said resources being a gift of nature, they should be made freely available to
everyone irrespective of the status in life. The doctrine enjoins upon the Government to protect the resources as
trustees for the enjoyment of the general public rather than to permit their use for private ownership or
commercial purposes.

Three types of restrictions on governmental authority are often thought to be imposed by the Public Trust: rst,
the property subject to the trust must not only be used for a public purpose, but it must be held available for use
by the general public; second, the property may not be sold, even for a fair cash equivalent; and third the
property must be maintained for particular types of uses.

Our legal system- based on English Common law- includes the Public Trust Doctrine as a part of its
jurisprudence. The State is the trustee of all-natural resources which are by nature meant for public use and
enjoyment. Public at large is the bene ciary of the seashore, running waters, airs, forests and ecologically fragile
lands. The State as a trustee is under a legal duty to protect the natural resources; these resources are meant
for public use and cannot be converted into private ownership. The aesthetic use and the pristine glory cannot
be permitted to be eroded for private, commercial or any other use unless the courts nd it necessary in good
faith, for public good and in public interest to encroach upon the said resource.

https://www.aspiration.ai/LAW/sis/Solution.jsp?qsetId=Lw5e7noFooA=&qsetName=LST Mock 20 2021 (CLAT) 147/208


12/18/2020 Mock Analysis

Q.93 [30465398]
Which of the following facts can save the construction project undertaken by Oscorp from the application of
public trust doctrine?

a Oscorp promised to the government to preserve the forest on the land.

b Oscorp promised to the government to preserve the forest on the land.

c The general public is not in a position to enjoy the forests in its current condition.

d None of the above.

Solution:
 Answer key/Solution
Correct Answer : d
Your Answer : c
As given in the passage, the natural resources should be preserved and should
not be made subject to private ownership. Thus, none of three facts override the principle that the reserved
forest must be held by the government as a trustee for public purposes.
FeedBack Bookmark

https://www.aspiration.ai/LAW/sis/Solution.jsp?qsetId=Lw5e7noFooA=&qsetName=LST Mock 20 2021 (CLAT) 148/208


12/18/2020 Mock Analysis

Directions for questions 67 to 105: You have been given some passages followed by questions based on each
passage. You are required to choose the most appropriate option which follows from the passage. Only the
information given in the passage should be used for choosing the answer and no external knowledge of law
howsoever prominent is to be applied.

Passage – 6

The Public Trust Doctrine primarily rests on the principle that certain resources like air, sea, waters and the
forests have such a great importance to the people as a whole that it would be wholly unjusti ed to make them
a subject of private ownership. The said resources being a gift of nature, they should be made freely available to
everyone irrespective of the status in life. The doctrine enjoins upon the Government to protect the resources as
trustees for the enjoyment of the general public rather than to permit their use for private ownership or
commercial purposes.

Three types of restrictions on governmental authority are often thought to be imposed by the Public Trust: rst,
the property subject to the trust must not only be used for a public purpose, but it must be held available for use
by the general public; second, the property may not be sold, even for a fair cash equivalent; and third the
property must be maintained for particular types of uses.

Our legal system- based on English Common law- includes the Public Trust Doctrine as a part of its
jurisprudence. The State is the trustee of all-natural resources which are by nature meant for public use and
enjoyment. Public at large is the bene ciary of the seashore, running waters, airs, forests and ecologically fragile
lands. The State as a trustee is under a legal duty to protect the natural resources; these resources are meant
for public use and cannot be converted into private ownership. The aesthetic use and the pristine glory cannot
be permitted to be eroded for private, commercial or any other use unless the courts nd it necessary in good
faith, for public good and in public interest to encroach upon the said resource.

Q.94 [30465398]
Which of the following duties of a trustee is not in line with the government's position under the public trust
doctrine?

a A trustee must take steps to preserve and protect the assets in the trust fund.

b A trustee should exercise care and skill in administering the trust.

c A trustee should convert perishable trust property into property of permanent and immediately pro table
character.

d The trustee must not deal with the trust property for personal nancial reward.

https://www.aspiration.ai/LAW/sis/Solution.jsp?qsetId=Lw5e7noFooA=&qsetName=LST Mock 20 2021 (CLAT) 149/208


12/18/2020 Mock Analysis

Solution:
 Answer key/Solution
Correct Answer : c
Your Answer : b
As the government is under the duty to preserve the natural resources to be
enjoyed by the general public, there is no duty upon it to convert the perishable resources. Rather, such
conversion would destroy and defeat the purpose of the public trust.

FeedBack Bookmark

Directions for questions 67 to 105: You have been given some passages followed by questions based on each
passage. You are required to choose the most appropriate option which follows from the passage. Only the
information given in the passage should be used for choosing the answer and no external knowledge of law
howsoever prominent is to be applied.

Passage – 6

The Public Trust Doctrine primarily rests on the principle that certain resources like air, sea, waters and the
forests have such a great importance to the people as a whole that it would be wholly unjusti ed to make them
a subject of private ownership. The said resources being a gift of nature, they should be made freely available to
everyone irrespective of the status in life. The doctrine enjoins upon the Government to protect the resources as
trustees for the enjoyment of the general public rather than to permit their use for private ownership or
commercial purposes.

Three types of restrictions on governmental authority are often thought to be imposed by the Public Trust: rst,
the property subject to the trust must not only be used for a public purpose, but it must be held available for use
by the general public; second, the property may not be sold, even for a fair cash equivalent; and third the
property must be maintained for particular types of uses.

Our legal system- based on English Common law- includes the Public Trust Doctrine as a part of its
jurisprudence. The State is the trustee of all-natural resources which are by nature meant for public use and
enjoyment. Public at large is the bene ciary of the seashore, running waters, airs, forests and ecologically fragile
lands. The State as a trustee is under a legal duty to protect the natural resources; these resources are meant
for public use and cannot be converted into private ownership. The aesthetic use and the pristine glory cannot
be permitted to be eroded for private, commercial or any other use unless the courts nd it necessary in good
faith, for public good and in public interest to encroach upon the said resource.

Q.95 [30465398]
A municipal corporation entered into a contract with a construction company to build a public park on the site of
an empty eld. The eld is currently used by only children as a playground. Which of the following facts can
support the case against such a contract?

a The eld is a natural resource and should be enjoyed by all.

b The government must maintain the property for particular uses only.

c The eld must not be made subject to private ownership.

https://www.aspiration.ai/LAW/sis/Solution.jsp?qsetId=Lw5e7noFooA=&qsetName=LST Mock 20 2021 (CLAT) 150/208


12/18/2020 Mock Analysis

d The eld is a part of ecologically fragile land.

Solution:
 Answer key/Solution
Correct Answer : d
Your Answer : c
A mere piece of land does not come within any of the descriptions of properties
given in the passage, thus by itself it cannot be made subject to the public trust doctrine. However, it being a
part of ecologically sensitive land creates a duty on the government to protect and hold the same for the
public.

FeedBack Bookmark

Directions for questions 67 to 105: You have been given some passages followed by questions based on each
passage. You are required to choose the most appropriate option which follows from the passage. Only the
information given in the passage should be used for choosing the answer and no external knowledge of law
howsoever prominent is to be applied.

Passage – 6

The Public Trust Doctrine primarily rests on the principle that certain resources like air, sea, waters and the
forests have such a great importance to the people as a whole that it would be wholly unjusti ed to make them
a subject of private ownership. The said resources being a gift of nature, they should be made freely available to
everyone irrespective of the status in life. The doctrine enjoins upon the Government to protect the resources as
trustees for the enjoyment of the general public rather than to permit their use for private ownership or
commercial purposes.

Three types of restrictions on governmental authority are often thought to be imposed by the Public Trust: rst,
the property subject to the trust must not only be used for a public purpose, but it must be held available for use
by the general public; second, the property may not be sold, even for a fair cash equivalent; and third the
property must be maintained for particular types of uses.

Our legal system- based on English Common law- includes the Public Trust Doctrine as a part of its
jurisprudence. The State is the trustee of all-natural resources which are by nature meant for public use and
enjoyment. Public at large is the bene ciary of the seashore, running waters, airs, forests and ecologically fragile
lands. The State as a trustee is under a legal duty to protect the natural resources; these resources are meant
for public use and cannot be converted into private ownership. The aesthetic use and the pristine glory cannot
be permitted to be eroded for private, commercial or any other use unless the courts nd it necessary in good
faith, for public good and in public interest to encroach upon the said resource.

Q.96 [30465398]
There was a lake within the limits of a city, the municipal corporation of which, decided to commercially exploit
the water of that lake and started selling the same to industries in the city. Which of the following is a strong
argument against such practice?

https://www.aspiration.ai/LAW/sis/Solution.jsp?qsetId=Lw5e7noFooA=&qsetName=LST Mock 20 2021 (CLAT) 151/208


12/18/2020 Mock Analysis

a The water is a natural resource and must be available for the use of the general public.

b The water is a natural resource and cannot be sold even for a cash equivalent.

c The water is a natural resource and must be maintained for particular types of uses.

d The government must not permit the use of water from the lake for private purposes.

Solution:
 Answer key/Solution
Correct Answer : b
Your Answer : b
As stated in the passage, a natural resource like water is held by the government
on behalf of the public and cannot be commercially exploited even for an exchange price.
FeedBack Bookmark

Directions for questions 67 to 105: You have been given some passages followed by questions based on each
passage. You are required to choose the most appropriate option which follows from the passage. Only the
information given in the passage should be used for choosing the answer and no external knowledge of law
howsoever prominent is to be applied.

Passage – 7

The 3-judge bench of the Supreme Court recently held that it is not necessary that before a person is convicted
on the ground of common intention, he must be actively involved in the physical activity of assault. If the nature
of evidence displays a prearranged plan and acting in concert pursuant to the plan, common intention can be
inferred. According to Section 34 of the IPC, when a criminal act is done by several persons in furtherance of
common intention of all, each of such persons is liable for that act in the same manner as if it were done by him
alone.

The Court explained that common intention consists of several persons acting in unison to achieve a common
purpose, though their roles may be different. The role may be active or passive is irrelevant, once common
intention is established.

The foundation for conviction on the basis of common intention is based on the principle of vicarious
responsibility by which a person is held to be answerable for the acts of others with whom he shared the
common intention. If the nature of evidence displays a prearranged plan and acting in concert pursuant to the
plan, common intention can be inferred. A common intention to bring about a particular result may also develop
on the spot as between a number of persons deducible from the facts and circumstances of a particular case.

https://www.aspiration.ai/LAW/sis/Solution.jsp?qsetId=Lw5e7noFooA=&qsetName=LST Mock 20 2021 (CLAT) 152/208


12/18/2020 Mock Analysis

Q.97 [30465398]
A was an offender wanted by the police. When he was passing through a locality, B,C,D and E saw him and tried
to capture him. While B,C and D chased him, they sent E to call the police. Upon catching him, they beat him and
he died as a result of the wounds. All four of them are charged with culpable homicide. Can E be convicted on
the basis of having common intention with the rest?

a Yes, as he played a passive role in the criminal act.

b No, as he did not participate in the act of beating A.

c No, as he did not act to achieve a common purpose with the others.

d Yes, as he shall be vicariously responsible for the acts of others with whom he shared a common intention.

Solution:
 Answer key/Solution
Correct Answer : c
Your Answer : b
As E did not have a common intention of beating or harming A as did the rest of
the offenders, he cannot be convicted. Thus, options (a) and (d) are incorrect. Option (b) is incorrect as active
participation is not the necessary requirement for common intention. The correct reason for his innocence in
option (c).
FeedBack Bookmark

https://www.aspiration.ai/LAW/sis/Solution.jsp?qsetId=Lw5e7noFooA=&qsetName=LST Mock 20 2021 (CLAT) 153/208


12/18/2020 Mock Analysis

Directions for questions 67 to 105: You have been given some passages followed by questions based on each
passage. You are required to choose the most appropriate option which follows from the passage. Only the
information given in the passage should be used for choosing the answer and no external knowledge of law
howsoever prominent is to be applied.

Passage – 7

The 3-judge bench of the Supreme Court recently held that it is not necessary that before a person is convicted
on the ground of common intention, he must be actively involved in the physical activity of assault. If the nature
of evidence displays a prearranged plan and acting in concert pursuant to the plan, common intention can be
inferred. According to Section 34 of the IPC, when a criminal act is done by several persons in furtherance of
common intention of all, each of such persons is liable for that act in the same manner as if it were done by him
alone.

The Court explained that common intention consists of several persons acting in unison to achieve a common
purpose, though their roles may be different. The role may be active or passive is irrelevant, once common
intention is established.

The foundation for conviction on the basis of common intention is based on the principle of vicarious
responsibility by which a person is held to be answerable for the acts of others with whom he shared the
common intention. If the nature of evidence displays a prearranged plan and acting in concert pursuant to the
plan, common intention can be inferred. A common intention to bring about a particular result may also develop
on the spot as between a number of persons deducible from the facts and circumstances of a particular case.

Q.98 [30465398]
In the case above, along with B, C, D and E, F also saw A and decided to go to the police station to inform the
police. Upon seeing F go to inform the police, E followed him in order to stall him and prevent the police from
nding A so that the rest of the three could deal with him. In such a case, can E be charged for the offence of
causing harm to A?

a No, as there was no prearranged plan in place.

b Yes, as there is evidence of him acting in concert with the other three.

c No, as his act was not in furtherance of the common intention of the rest of the three.

d Yes, as he was as good as actively involved in the physical activity of assault.

Solution:
 Answer key/Solution
Correct Answer : b
Your Answer : d
The passage clearly states that common intention can be inferred upon the
evidence showing acts in concert. Here, E's intention of stalling the informant is in line with the common
intention of the rest and thus he can be made liable for their acts.
FeedBack Bookmark

https://www.aspiration.ai/LAW/sis/Solution.jsp?qsetId=Lw5e7noFooA=&qsetName=LST Mock 20 2021 (CLAT) 154/208


12/18/2020 Mock Analysis

Directions for questions 67 to 105: You have been given some passages followed by questions based on each
passage. You are required to choose the most appropriate option which follows from the passage. Only the
information given in the passage should be used for choosing the answer and no external knowledge of law
howsoever prominent is to be applied.

Passage – 7

The 3-judge bench of the Supreme Court recently held that it is not necessary that before a person is convicted
on the ground of common intention, he must be actively involved in the physical activity of assault. If the nature
of evidence displays a prearranged plan and acting in concert pursuant to the plan, common intention can be
inferred. According to Section 34 of the IPC, when a criminal act is done by several persons in furtherance of
common intention of all, each of such persons is liable for that act in the same manner as if it were done by him
alone.

The Court explained that common intention consists of several persons acting in unison to achieve a common
purpose, though their roles may be different. The role may be active or passive is irrelevant, once common
intention is established.

The foundation for conviction on the basis of common intention is based on the principle of vicarious
responsibility by which a person is held to be answerable for the acts of others with whom he shared the
common intention. If the nature of evidence displays a prearranged plan and acting in concert pursuant to the
plan, common intention can be inferred. A common intention to bring about a particular result may also develop
on the spot as between a number of persons deducible from the facts and circumstances of a particular case.

Q.99 [30465398]
A, B and C planned to commit a robbery together. A was tasked with keeping guard at the gates of the bank, B
with dealing with the guards and C with actually taking the money out of the vault of the bank. However, before
they reached the bank, police intercepted them and arrested A. B and C went ahead to the bank and committed
the robbery. Can A be charged with robbing the bank on the grounds of common intention?

a Yes, as he was acting in concert with B and C in pursuant of a common plan.

b No, as he had not yet done any act in pursuant to the plan.

c Yes, as he was playing a passive role in the plan.

d No, as he cannot be held vicariously responsible for the criminal acts of B and C.

https://www.aspiration.ai/LAW/sis/Solution.jsp?qsetId=Lw5e7noFooA=&qsetName=LST Mock 20 2021 (CLAT) 155/208


12/18/2020 Mock Analysis

Solution:
 Answer key/Solution
Correct Answer : b
Your Answer : a
It is given that the police arrested A even before he could reach the bank. He had
common intention no doubt, but he had not played any role at all, active or passive and cannot be convicted
under the same charge as B and C.

FeedBack Bookmark

Directions for questions 67 to 105: You have been given some passages followed by questions based on each
passage. You are required to choose the most appropriate option which follows from the passage. Only the
information given in the passage should be used for choosing the answer and no external knowledge of law
howsoever prominent is to be applied.

Passage – 7

The 3-judge bench of the Supreme Court recently held that it is not necessary that before a person is convicted
on the ground of common intention, he must be actively involved in the physical activity of assault. If the nature
of evidence displays a prearranged plan and acting in concert pursuant to the plan, common intention can be
inferred. According to Section 34 of the IPC, when a criminal act is done by several persons in furtherance of
common intention of all, each of such persons is liable for that act in the same manner as if it were done by him
alone.

The Court explained that common intention consists of several persons acting in unison to achieve a common
purpose, though their roles may be different. The role may be active or passive is irrelevant, once common
intention is established.

The foundation for conviction on the basis of common intention is based on the principle of vicarious
responsibility by which a person is held to be answerable for the acts of others with whom he shared the
common intention. If the nature of evidence displays a prearranged plan and acting in concert pursuant to the
plan, common intention can be inferred. A common intention to bring about a particular result may also develop
on the spot as between a number of persons deducible from the facts and circumstances of a particular case.

Q.100 [30465398]
In the case above, which of the following is a fact in favour of the prosecution in a trial against A?

a That A purposefully got arrested to avoid being charged with the same offence as B and C.

b That A had already disabled the bank's security system by hacking into it.

c That A was going to the bank pursuant to the prearranged plan.

d That A was unaware of the plan to rob the bank.

https://www.aspiration.ai/LAW/sis/Solution.jsp?qsetId=Lw5e7noFooA=&qsetName=LST Mock 20 2021 (CLAT) 156/208


12/18/2020 Mock Analysis

Solution:
 Answer key/Solution
Correct Answer : b
Your Answer : c
The fact that A had already done an act in pursuant to the common intention can
make him liable for all the acts committed by others in furtherance of the same intention, thus option (b) is
correct.

FeedBack Bookmark

Directions for questions 67 to 105: You have been given some passages followed by questions based on each
passage. You are required to choose the most appropriate option which follows from the passage. Only the
information given in the passage should be used for choosing the answer and no external knowledge of law
howsoever prominent is to be applied.

Passage – 7

The 3-judge bench of the Supreme Court recently held that it is not necessary that before a person is convicted
on the ground of common intention, he must be actively involved in the physical activity of assault. If the nature
of evidence displays a prearranged plan and acting in concert pursuant to the plan, common intention can be
inferred. According to Section 34 of the IPC, when a criminal act is done by several persons in furtherance of
common intention of all, each of such persons is liable for that act in the same manner as if it were done by him
alone.

The Court explained that common intention consists of several persons acting in unison to achieve a common
purpose, though their roles may be different. The role may be active or passive is irrelevant, once common
intention is established.

The foundation for conviction on the basis of common intention is based on the principle of vicarious
responsibility by which a person is held to be answerable for the acts of others with whom he shared the
common intention. If the nature of evidence displays a prearranged plan and acting in concert pursuant to the
plan, common intention can be inferred. A common intention to bring about a particular result may also develop
on the spot as between a number of persons deducible from the facts and circumstances of a particular case.

Q.101 [30465398]
A and B administered poison to C. Who of the following cannot be held as having common intention with A and
B?

a D, C's wife, who did not stop C from eating the poisoned food in spite of knowing that it was poisoned.

b Q, C's cook who prepared the food and put the rat poison in it.

c J, a shopkeeper who sold the ratpoison to A, knowing that the poison was lethal.

d N, C's brother who served the food to C after seeing it being poisoned by the cook.

https://www.aspiration.ai/LAW/sis/Solution.jsp?qsetId=Lw5e7noFooA=&qsetName=LST Mock 20 2021 (CLAT) 157/208


12/18/2020 Mock Analysis

Solution:
 Answer key/Solution
Correct Answer : c
Your Answer : c
Other than the shopkeeper, everyone else had the particular knowledge of the
plan of A and B and played either an active or passive role in the act of poisoning him. J, however, did not have
any pre-arranged or spontaneous common intention with A and B and merely knowledge of lethality of the
poison does not constitute an action in concert. Thus, he is the only one who cannot be charged with having
acted with common intention.

FeedBack Bookmark

https://www.aspiration.ai/LAW/sis/Solution.jsp?qsetId=Lw5e7noFooA=&qsetName=LST Mock 20 2021 (CLAT) 158/208


12/18/2020 Mock Analysis

Directions for questions 67 to 105: You have been given some passages followed by questions based on each
passage. You are required to choose the most appropriate option which follows from the passage. Only the
information given in the passage should be used for choosing the answer and no external knowledge of law
howsoever prominent is to be applied.

Passage – 8

The terms libel, slander, and defamation are frequently confused with each other. They are all similar in that they
all fall into the same general area of law that concerns false statements which harm a person's reputation. This
general area of law is called defamation law. Libel and slander are types of defamatory statements. Libel is a
defamatory statement that is written. Slander is a defamatory statement that is oral.

Historically, the distinction between libel and slander was signi cant and had real-world implications regarding
how a case was litigated including the elements that had to be proven and who had the burden of proof. Illinois
courts have changed their approach, however, as the Illinois Supreme Court explained in Bryson v. News
America Publication, Inc.:

At common law, libel and slander were analyzed under different sets of standards, with libel recognized as the
more serious wrong. Illinois law evolved, however, and rejected this bifurcated approach in favor of a single set
of rules for slander and libel. Libel and slander are now treated alike and the same rules apply to a defamatory
statement regardless of whether the statement is written or oral.

The tort of defamation (sometimes referred to as defamation of character) can be divided into claims involving
two distinct types of statements: defamatory per se statements and defamatory per quod statements.
Statements that are defamatory per se (sometimes referred to generically by courts as libel per se) are so
obviously and naturally harmful to one's reputation on their face that proof of injury is not required. Illinois law
recognizes ve types of statements that are considered defamatory per se:
1. Imputing that a person committed a crime;
2. Imputing that a person is infected with a loathsome communicable disease;
3. Imputing that a person is unable or lacks the integrity to perform one's employment duties;
4. Imputing that a person lack ability or otherwise prejudices one in one's profession; and
5. Imputing that a person has engaged in adultery or fornication.

Importantly, a statement can only be considered defamatory per se if the harmful effect is apparent on the face
of the statement itself. If extrinsic facts or additional information about the person being defamed is required to
understand the harmful effect of the statement, then it cannot be defamatory per se. That is not to say the
statement is not defamatory if extrinsic facts are required; it just cannot be defamatory per se.

If a defamatory statement does not fall into one of the defamatory per se categories or requires extrinsic facts,
then it is considered defamatory per quod. Unlike in cases involving defamation per se, defamation per quod
claims require the plaintiff to allege and prove special damages (also called "special harm" by some courts). The
term "special damages" or "special harm" is a legal term of art in defamation law that means the loss of
something with actual economic or pecuniary value. In other words, a plaintiff alleging defamation per quod
must be able to show speci cally how the defamation caused a speci c, quanti able loss of money such as the
commission from a lost sale or the salary from a lost job.

https://www.aspiration.ai/LAW/sis/Solution.jsp?qsetId=Lw5e7noFooA=&qsetName=LST Mock 20 2021 (CLAT) 159/208


12/18/2020 Mock Analysis

Q.102 [30465398]
A and B are business rivals. C, a client approaches B with his project, to discuss the same. B, while discussing
the project and the approach to be adopted, says that doing business with A takes a lot of time and patience. A
sues B for defamation. Decide.

a B is liable for defamation per se.

b B is liable for defamation per quod.

c B is not liable for defamation because this does not affect the reputation of A.

d B is not liable for defamation because it is an oral statement.

Solution:
 Answer key/Solution
Correct Answer : b
Your Answer : a
The statement is not explicitly harming by directly attacking the quality of the
work done by A. A will have to prove that the statement did indirectly harm A's reputation.

FeedBack Bookmark

https://www.aspiration.ai/LAW/sis/Solution.jsp?qsetId=Lw5e7noFooA=&qsetName=LST Mock 20 2021 (CLAT) 160/208


12/18/2020 Mock Analysis

Directions for questions 67 to 105: You have been given some passages followed by questions based on each
passage. You are required to choose the most appropriate option which follows from the passage. Only the
information given in the passage should be used for choosing the answer and no external knowledge of law
howsoever prominent is to be applied.

Passage – 8

The terms libel, slander, and defamation are frequently confused with each other. They are all similar in that they
all fall into the same general area of law that concerns false statements which harm a person's reputation. This
general area of law is called defamation law. Libel and slander are types of defamatory statements. Libel is a
defamatory statement that is written. Slander is a defamatory statement that is oral.

Historically, the distinction between libel and slander was signi cant and had real-world implications regarding
how a case was litigated including the elements that had to be proven and who had the burden of proof. Illinois
courts have changed their approach, however, as the Illinois Supreme Court explained in Bryson v. News
America Publication, Inc.:

At common law, libel and slander were analyzed under different sets of standards, with libel recognized as the
more serious wrong. Illinois law evolved, however, and rejected this bifurcated approach in favor of a single set
of rules for slander and libel. Libel and slander are now treated alike and the same rules apply to a defamatory
statement regardless of whether the statement is written or oral.

The tort of defamation (sometimes referred to as defamation of character) can be divided into claims involving
two distinct types of statements: defamatory per se statements and defamatory per quod statements.
Statements that are defamatory per se (sometimes referred to generically by courts as libel per se) are so
obviously and naturally harmful to one's reputation on their face that proof of injury is not required. Illinois law
recognizes ve types of statements that are considered defamatory per se:
1. Imputing that a person committed a crime;
2. Imputing that a person is infected with a loathsome communicable disease;
3. Imputing that a person is unable or lacks the integrity to perform one's employment duties;
4. Imputing that a person lack ability or otherwise prejudices one in one's profession; and
5. Imputing that a person has engaged in adultery or fornication.

Importantly, a statement can only be considered defamatory per se if the harmful effect is apparent on the face
of the statement itself. If extrinsic facts or additional information about the person being defamed is required to
understand the harmful effect of the statement, then it cannot be defamatory per se. That is not to say the
statement is not defamatory if extrinsic facts are required; it just cannot be defamatory per se.

If a defamatory statement does not fall into one of the defamatory per se categories or requires extrinsic facts,
then it is considered defamatory per quod. Unlike in cases involving defamation per se, defamation per quod
claims require the plaintiff to allege and prove special damages (also called "special harm" by some courts). The
term "special damages" or "special harm" is a legal term of art in defamation law that means the loss of
something with actual economic or pecuniary value. In other words, a plaintiff alleging defamation per quod
must be able to show speci cally how the defamation caused a speci c, quanti able loss of money such as the
commission from a lost sale or the salary from a lost job.

https://www.aspiration.ai/LAW/sis/Solution.jsp?qsetId=Lw5e7noFooA=&qsetName=LST Mock 20 2021 (CLAT) 161/208


12/18/2020 Mock Analysis

Q.103 [30465398]
A and B are business rivals. C, a client approaches B with his project, and meets the owner to discuss the same.
B's owner, while discussing the project and the approach to be adopted, says that A has been charged with
bribery and blackmail, which turns out to be a false statement. A sues B for defamation. Decide.

a B is liable for defamation per se.

b B is liable for defamation per quod.

c B is not liable for defamation because this does not affect the reputation of A.

d B is not liable for defamation because it is an oral statement.

Solution:
 Answer key/Solution
Correct Answer : a
Your Answer : a
This false statement was harmful to A's reputation on the face of it and also falls
within the list of statements that are covered under the law stated above.

FeedBack Bookmark

https://www.aspiration.ai/LAW/sis/Solution.jsp?qsetId=Lw5e7noFooA=&qsetName=LST Mock 20 2021 (CLAT) 162/208


12/18/2020 Mock Analysis

Directions for questions 67 to 105: You have been given some passages followed by questions based on each
passage. You are required to choose the most appropriate option which follows from the passage. Only the
information given in the passage should be used for choosing the answer and no external knowledge of law
howsoever prominent is to be applied.

Passage – 8

The terms libel, slander, and defamation are frequently confused with each other. They are all similar in that they
all fall into the same general area of law that concerns false statements which harm a person's reputation. This
general area of law is called defamation law. Libel and slander are types of defamatory statements. Libel is a
defamatory statement that is written. Slander is a defamatory statement that is oral.

Historically, the distinction between libel and slander was signi cant and had real-world implications regarding
how a case was litigated including the elements that had to be proven and who had the burden of proof. Illinois
courts have changed their approach, however, as the Illinois Supreme Court explained in Bryson v. News
America Publication, Inc.:

At common law, libel and slander were analyzed under different sets of standards, with libel recognized as the
more serious wrong. Illinois law evolved, however, and rejected this bifurcated approach in favor of a single set
of rules for slander and libel. Libel and slander are now treated alike and the same rules apply to a defamatory
statement regardless of whether the statement is written or oral.

The tort of defamation (sometimes referred to as defamation of character) can be divided into claims involving
two distinct types of statements: defamatory per se statements and defamatory per quod statements.
Statements that are defamatory per se (sometimes referred to generically by courts as libel per se) are so
obviously and naturally harmful to one's reputation on their face that proof of injury is not required. Illinois law
recognizes ve types of statements that are considered defamatory per se:
1. Imputing that a person committed a crime;
2. Imputing that a person is infected with a loathsome communicable disease;
3. Imputing that a person is unable or lacks the integrity to perform one's employment duties;
4. Imputing that a person lack ability or otherwise prejudices one in one's profession; and
5. Imputing that a person has engaged in adultery or fornication.

Importantly, a statement can only be considered defamatory per se if the harmful effect is apparent on the face
of the statement itself. If extrinsic facts or additional information about the person being defamed is required to
understand the harmful effect of the statement, then it cannot be defamatory per se. That is not to say the
statement is not defamatory if extrinsic facts are required; it just cannot be defamatory per se.

If a defamatory statement does not fall into one of the defamatory per se categories or requires extrinsic facts,
then it is considered defamatory per quod. Unlike in cases involving defamation per se, defamation per quod
claims require the plaintiff to allege and prove special damages (also called "special harm" by some courts). The
term "special damages" or "special harm" is a legal term of art in defamation law that means the loss of
something with actual economic or pecuniary value. In other words, a plaintiff alleging defamation per quod
must be able to show speci cally how the defamation caused a speci c, quanti able loss of money such as the
commission from a lost sale or the salary from a lost job.

https://www.aspiration.ai/LAW/sis/Solution.jsp?qsetId=Lw5e7noFooA=&qsetName=LST Mock 20 2021 (CLAT) 163/208


12/18/2020 Mock Analysis

Q.104 [30465398]
A robbery is committed at B's house by A. During the investigations, B's neighbour is asked if he saw any
stranger going in or leaving from B's house, to which the neighbour replied that he had not seen anyone within
the premises but saw A passing through the streets around the time when the robbery is suspected to have
been committed. A, on gaining knowledge of this, les a suit against B' neighbour for defamation. Decide.

a The neighbour is liable for defamation per se.

b The neighbour is liable for defamation per quod.

c The neighbour is not liable for defamation because this does not affect the reputation of A.

d The neighbour is not liable for defamation because the statement is not false.

Solution:
 Answer key/Solution
Correct Answer : d
Your Answer : d
As is clear from the facts, A committed the robbery and B's neighbour therefore
made a true claim of seeing A in the vicinity of the house around that time, and therefore cannot be held liable
for defamation.
FeedBack Bookmark

https://www.aspiration.ai/LAW/sis/Solution.jsp?qsetId=Lw5e7noFooA=&qsetName=LST Mock 20 2021 (CLAT) 164/208


12/18/2020 Mock Analysis

Directions for questions 67 to 105: You have been given some passages followed by questions based on each
passage. You are required to choose the most appropriate option which follows from the passage. Only the
information given in the passage should be used for choosing the answer and no external knowledge of law
howsoever prominent is to be applied.

Passage – 8

The terms libel, slander, and defamation are frequently confused with each other. They are all similar in that they
all fall into the same general area of law that concerns false statements which harm a person's reputation. This
general area of law is called defamation law. Libel and slander are types of defamatory statements. Libel is a
defamatory statement that is written. Slander is a defamatory statement that is oral.

Historically, the distinction between libel and slander was signi cant and had real-world implications regarding
how a case was litigated including the elements that had to be proven and who had the burden of proof. Illinois
courts have changed their approach, however, as the Illinois Supreme Court explained in Bryson v. News
America Publication, Inc.:

At common law, libel and slander were analyzed under different sets of standards, with libel recognized as the
more serious wrong. Illinois law evolved, however, and rejected this bifurcated approach in favor of a single set
of rules for slander and libel. Libel and slander are now treated alike and the same rules apply to a defamatory
statement regardless of whether the statement is written or oral.

The tort of defamation (sometimes referred to as defamation of character) can be divided into claims involving
two distinct types of statements: defamatory per se statements and defamatory per quod statements.
Statements that are defamatory per se (sometimes referred to generically by courts as libel per se) are so
obviously and naturally harmful to one's reputation on their face that proof of injury is not required. Illinois law
recognizes ve types of statements that are considered defamatory per se:
1. Imputing that a person committed a crime;
2. Imputing that a person is infected with a loathsome communicable disease;
3. Imputing that a person is unable or lacks the integrity to perform one's employment duties;
4. Imputing that a person lack ability or otherwise prejudices one in one's profession; and
5. Imputing that a person has engaged in adultery or fornication.

Importantly, a statement can only be considered defamatory per se if the harmful effect is apparent on the face
of the statement itself. If extrinsic facts or additional information about the person being defamed is required to
understand the harmful effect of the statement, then it cannot be defamatory per se. That is not to say the
statement is not defamatory if extrinsic facts are required; it just cannot be defamatory per se.

If a defamatory statement does not fall into one of the defamatory per se categories or requires extrinsic facts,
then it is considered defamatory per quod. Unlike in cases involving defamation per se, defamation per quod
claims require the plaintiff to allege and prove special damages (also called "special harm" by some courts). The
term "special damages" or "special harm" is a legal term of art in defamation law that means the loss of
something with actual economic or pecuniary value. In other words, a plaintiff alleging defamation per quod
must be able to show speci cally how the defamation caused a speci c, quanti able loss of money such as the
commission from a lost sale or the salary from a lost job.

https://www.aspiration.ai/LAW/sis/Solution.jsp?qsetId=Lw5e7noFooA=&qsetName=LST Mock 20 2021 (CLAT) 165/208


12/18/2020 Mock Analysis

Q.105 [30465398]
A, a doctor, published a letter in the local newspaper about B, also a doctor, which read, "B is untruthful". B reads
this letter and les a suit of defamation against A. Decide.

a A is liable for defamation per se.

b A is liable for defamation per quod.

c A is not liable for defamation because this does not affect the reputation of B.

d A is not liable for defamation because it is a written statement.

Solution:
 Answer key/Solution
Correct Answer : b
Your Answer : a
The statement states that B is untruthful, which harms B's reputation, but this
does not per se prove that A is targeting B's professional abilities.

FeedBack Bookmark

Sec 4
Directions for questions 106 to 130: Each set of questions in this section is based on the reasoning and
arguments set out in the preceding passage. Please answer each question on the basis of what is stated or
implied in the corresponding passage. Do not rely on any information or facts other than the ones supplied to
you. In some instances, more than one option may be the answer to the question; in such a case, please choose
the option that most accurately and comprehensively answers the question.

Passage – 1

Indian languages have never got the importance they deserve in the higher education system. Teaching and
learning have largely been in a foreign language. But a change is in the o ng, with the advent of the National
Education Policy (NEP) 2020. Prime Minister (PM) Narendra Modi recently tweeted: "Taking inspiration from the
National Education Policy, it will now be tried to teach all technical courses including medical, engineering in
mother tongue."

It is time we focused on the use of the mother tongue or regional language as a medium of instruction in higher
education. The discussion on the use of the mother tongue in education goes back to the time of Lord
Macaulay. The Radhakrishnan report recommended that English be replaced by an Indian language as the
medium of instruction for higher education as early as practicable. The report also pronounced that Indian
languages are a sine qua non for educational and cultural development since they strengthened equity in
education.

https://www.aspiration.ai/LAW/sis/Solution.jsp?qsetId=Lw5e7noFooA=&qsetName=LST Mock 20 2021 (CLAT) 166/208


12/18/2020 Mock Analysis

Q.106 [30465398]
In the light of the passage, it can be inferred that:

a Vernacular languages are integral in promoting fairness in mass media.

b Vernacular languages are integral in promoting fairness in education.

c Vernacular languages are integral in promoting fairness in politics.

d Vernacular languages are integral in promoting fairness in administration.

Solution:
 Answer key/Solution
Correct Answer : b
Your Answer : b
Refer to the last sentence of the passage where the author mentions that Indian
languages have an important role to play in educational development because they strengthened equity or
fairness. Therefore, option (b) is correct. Mass media, politics or administration are not mentioned
categorically in the passage and so, they are incorrect.

FeedBack Bookmark

Directions for questions 106 to 130: Each set of questions in this section is based on the reasoning and
arguments set out in the preceding passage. Please answer each question on the basis of what is stated or
implied in the corresponding passage. Do not rely on any information or facts other than the ones supplied to
you. In some instances, more than one option may be the answer to the question; in such a case, please choose
the option that most accurately and comprehensively answers the question.

Passage – 1

Indian languages have never got the importance they deserve in the higher education system. Teaching and
learning have largely been in a foreign language. But a change is in the o ng, with the advent of the National
Education Policy (NEP) 2020. Prime Minister (PM) Narendra Modi recently tweeted: "Taking inspiration from the
National Education Policy, it will now be tried to teach all technical courses including medical, engineering in
mother tongue."

It is time we focused on the use of the mother tongue or regional language as a medium of instruction in higher
education. The discussion on the use of the mother tongue in education goes back to the time of Lord
Macaulay. The Radhakrishnan report recommended that English be replaced by an Indian language as the
medium of instruction for higher education as early as practicable. The report also pronounced that Indian
languages are a sine qua non for educational and cultural development since they strengthened equity in
education.

Q.107 [30465398]
The author of the passage is most likely to agree with which of the following statements:

https://www.aspiration.ai/LAW/sis/Solution.jsp?qsetId=Lw5e7noFooA=&qsetName=LST Mock 20 2021 (CLAT) 167/208


12/18/2020 Mock Analysis

a Lord Macaulay wanted to promote vernacular literature in schools and colleges.

b Lord Macaulay loved Indians and so, favoured promoting vernacular literature.

c The discussion regarding the role of the mother tongue in development has been there for some time.

d The discussion regarding the role of the mother tongue in development has been there since the 19th
century.

Solution:
 Answer key/Solution
Correct Answer : c
Your Answer : d
Refer to the sentence where the author mentions that the role of the mother
tongue in development was discussed even by Lord Macaulay, a British o cer. Hence, option (c) is correct. No
time frame has been mentioned and so, option (d) is incorrect. Nothing has been mentioned in the passage to
substantiate options (a) and (b) and so, they are incorrect.

FeedBack Bookmark

Directions for questions 106 to 130: Each set of questions in this section is based on the reasoning and
arguments set out in the preceding passage. Please answer each question on the basis of what is stated or
implied in the corresponding passage. Do not rely on any information or facts other than the ones supplied to
you. In some instances, more than one option may be the answer to the question; in such a case, please choose
the option that most accurately and comprehensively answers the question.

Passage – 1

Indian languages have never got the importance they deserve in the higher education system. Teaching and
learning have largely been in a foreign language. But a change is in the o ng, with the advent of the National
Education Policy (NEP) 2020. Prime Minister (PM) Narendra Modi recently tweeted: "Taking inspiration from the
National Education Policy, it will now be tried to teach all technical courses including medical, engineering in
mother tongue."

It is time we focused on the use of the mother tongue or regional language as a medium of instruction in higher
education. The discussion on the use of the mother tongue in education goes back to the time of Lord
Macaulay. The Radhakrishnan report recommended that English be replaced by an Indian language as the
medium of instruction for higher education as early as practicable. The report also pronounced that Indian
languages are a sine qua non for educational and cultural development since they strengthened equity in
education.

Q.108 [30465398]
Out of the following options, which one can be inferred to be the central idea of the passage?

a Time to prioritise Indian languages in higher education

https://www.aspiration.ai/LAW/sis/Solution.jsp?qsetId=Lw5e7noFooA=&qsetName=LST Mock 20 2021 (CLAT) 168/208


12/18/2020 Mock Analysis

b Time to prioritise Indian languages in elementary education

c Time to prioritise Indian languages in international relations

d Time to prioritise Indian languages in engineering education only

Solution:
 Answer key/Solution
Correct Answer : a
Your Answer : a
The passage talks about the plans to introduce vernacular languages in higher
education. Hence, option (a) is the most appropriate. Option (b) is wrong because it talks about elementary
and not higher education. Option (c) is out of scope. Option (d) is narrow in scope because of the usage of
'only.'

FeedBack Bookmark

Directions for questions 106 to 130: Each set of questions in this section is based on the reasoning and
arguments set out in the preceding passage. Please answer each question on the basis of what is stated or
implied in the corresponding passage. Do not rely on any information or facts other than the ones supplied to
you. In some instances, more than one option may be the answer to the question; in such a case, please choose
the option that most accurately and comprehensively answers the question.

Passage – 1

Indian languages have never got the importance they deserve in the higher education system. Teaching and
learning have largely been in a foreign language. But a change is in the o ng, with the advent of the National
Education Policy (NEP) 2020. Prime Minister (PM) Narendra Modi recently tweeted: "Taking inspiration from the
National Education Policy, it will now be tried to teach all technical courses including medical, engineering in
mother tongue."

It is time we focused on the use of the mother tongue or regional language as a medium of instruction in higher
education. The discussion on the use of the mother tongue in education goes back to the time of Lord
Macaulay. The Radhakrishnan report recommended that English be replaced by an Indian language as the
medium of instruction for higher education as early as practicable. The report also pronounced that Indian
languages are a sine qua non for educational and cultural development since they strengthened equity in
education.

Q.109 [30465398]
One of the possible challenges in introducing vernacular languages in higher education can be:

a Lack of primary school teachers who can teach subjects in the vernacular

b Lack of cohesion among family members

https://www.aspiration.ai/LAW/sis/Solution.jsp?qsetId=Lw5e7noFooA=&qsetName=LST Mock 20 2021 (CLAT) 169/208


12/18/2020 Mock Analysis

c Availability of abundant study materials for higher studies in vernacular languages

d Lack of secondary reference and higher study materials in Indian languages

Solution:
 Answer key/Solution
Correct Answer : d
Your Answer : d
Option (d), if true, can pose a challenge to the plans of introducing vernacular
languages in higher education. Option (c) is strength and not a weakness. Option (a) is out of scope because
the passage does not talk about primary school teaching. The scope of the passage is higher education.
Option (b) is out of scope.
FeedBack Bookmark

Directions for questions 106 to 130: Each set of questions in this section is based on the reasoning and
arguments set out in the preceding passage. Please answer each question on the basis of what is stated or
implied in the corresponding passage. Do not rely on any information or facts other than the ones supplied to
you. In some instances, more than one option may be the answer to the question; in such a case, please choose
the option that most accurately and comprehensively answers the question.

Passage – 1

Indian languages have never got the importance they deserve in the higher education system. Teaching and
learning have largely been in a foreign language. But a change is in the o ng, with the advent of the National
Education Policy (NEP) 2020. Prime Minister (PM) Narendra Modi recently tweeted: "Taking inspiration from the
National Education Policy, it will now be tried to teach all technical courses including medical, engineering in
mother tongue."

It is time we focused on the use of the mother tongue or regional language as a medium of instruction in higher
education. The discussion on the use of the mother tongue in education goes back to the time of Lord
Macaulay. The Radhakrishnan report recommended that English be replaced by an Indian language as the
medium of instruction for higher education as early as practicable. The report also pronounced that Indian
languages are a sine qua non for educational and cultural development since they strengthened equity in
education.

Q.110 [30465398]
In the light of the passage, it can be understood that:

a English has played a role in unifying Indians because of its ubiquitous presence.

b English has been the lingua franca in the domain of higher education in India.

c English has been the lingua franca in the domain of higher education all over the world.

https://www.aspiration.ai/LAW/sis/Solution.jsp?qsetId=Lw5e7noFooA=&qsetName=LST Mock 20 2021 (CLAT) 170/208


12/18/2020 Mock Analysis

d English has been a very popular language in the north eastern part of India.

Solution:
 Answer key/Solution
Correct Answer : b
Your Answer : b
Refer to the sentence, "The Radhakrishnan report recommended that English be
replaced by an Indian language as the medium of instruction for higher education as early as practicable."
'Lingua franca' refers to a language that is adopted as a common language between speakers whose native
languages are different. We don't know if English is the lingua franca all over the world in the eld of higher
education. So, option (c) is wrong. Option (d) is out of scope. Option (a) is wrong because it is far-fetched and
not mentioned in the passage.
FeedBack Bookmark

Directions for questions 106 to 130: Each set of questions in this section is based on the reasoning and
arguments set out in the preceding passage. Please answer each question on the basis of what is stated or
implied in the corresponding passage. Do not rely on any information or facts other than the ones supplied to
you. In some instances, more than one option may be the answer to the question; in such a case, please choose
the option that most accurately and comprehensively answers the question.

Passage – 2

Global warming, along with the impact of other negative human activities, is devastating our oceans. This has
led to an alarming rise in sea levels that could displace millions of people. The ocean is turning warmer, less
predictable, and more acidic, causing a decline in sh stocks and the death of coral reefs. The millions of tonnes
of plastics dumped into the oceans every year contaminate at least 700 species of marine life. Unregulated
over shing has already driven many marine species to extinction and this could eventually threaten global food
security. Oceans produce half the planet's oxygen and absorb more than 90% of the anthropogenic heat. The
ocean-led economy is estimated to be contributing more than $1.5 trillion a year to the global economy, with
millions employed in related sectors. We must understand that if the resources of the ocean are sustainably
harnessed, it could multiply economic bene ts while protecting the natural ecosystem. The solution lies in
collaboration among governments, businesses and coastal communities.

Q.111 [30465398]
The author of the passage is most likely to agree with which of the following statements:

a The coastal communities pollute the ocean in the maximum possible manner.

b It is possible to save the marine ecosystem with the initiative of the government alone.

c It is impossible to save the marine ecosystem now.

d The marine ecosystem can be saved if every stakeholder is involved.

https://www.aspiration.ai/LAW/sis/Solution.jsp?qsetId=Lw5e7noFooA=&qsetName=LST Mock 20 2021 (CLAT) 171/208


12/18/2020 Mock Analysis

Solution:
 Answer key/Solution
Correct Answer : d
Your Answer : d
Refer to the last sentence of the passage where the author mentions that
governments, coastal communities and businesses have to be involved to save the marine ecosystem.
Therefore, option (d) is true. Option (b) is wrong in the light of the last sentence of the passage. Option (c) is
out of scope. Option (a) is wrong because it cannot be corroborated in the light of the passage.
FeedBack Bookmark

Directions for questions 106 to 130: Each set of questions in this section is based on the reasoning and
arguments set out in the preceding passage. Please answer each question on the basis of what is stated or
implied in the corresponding passage. Do not rely on any information or facts other than the ones supplied to
you. In some instances, more than one option may be the answer to the question; in such a case, please choose
the option that most accurately and comprehensively answers the question.

Passage – 2

Global warming, along with the impact of other negative human activities, is devastating our oceans. This has
led to an alarming rise in sea levels that could displace millions of people. The ocean is turning warmer, less
predictable, and more acidic, causing a decline in sh stocks and the death of coral reefs. The millions of tonnes
of plastics dumped into the oceans every year contaminate at least 700 species of marine life. Unregulated
over shing has already driven many marine species to extinction and this could eventually threaten global food
security. Oceans produce half the planet's oxygen and absorb more than 90% of the anthropogenic heat. The
ocean-led economy is estimated to be contributing more than $1.5 trillion a year to the global economy, with
millions employed in related sectors. We must understand that if the resources of the ocean are sustainably
harnessed, it could multiply economic bene ts while protecting the natural ecosystem. The solution lies in
collaboration among governments, businesses and coastal communities.

Q.112 [30465398]
In the light of the passage, it can be inferred that:

a Global warming is the only factor that is affecting oceans.

b Global warming is responsible for the extinction of many mammals.

c Degradation in the quality of ocean water can jeopardise the livelihood of many people.

d Mostly the Asian communities are dependent on oceans for their livelihood.

https://www.aspiration.ai/LAW/sis/Solution.jsp?qsetId=Lw5e7noFooA=&qsetName=LST Mock 20 2021 (CLAT) 172/208


12/18/2020 Mock Analysis

Solution:
 Answer key/Solution
Correct Answer : c
Your Answer : c
Refer to the sentence, "The ocean-led economy is estimated to be contributing
more than $1.5 trillion a year to the global economy, with millions employed in related sectors." Therefore,
option (c) is correct. Option (d) is out of scope. Option (a) is wrong in the light of the rst sentence. The author
mentions that other factors, along with global warming, are responsible for the damage to the oceans. Many
marine species have become extinct because of unregulated shing but we don't know if they are mammals.
So, option (b) is untrue.
FeedBack Bookmark

Directions for questions 106 to 130: Each set of questions in this section is based on the reasoning and
arguments set out in the preceding passage. Please answer each question on the basis of what is stated or
implied in the corresponding passage. Do not rely on any information or facts other than the ones supplied to
you. In some instances, more than one option may be the answer to the question; in such a case, please choose
the option that most accurately and comprehensively answers the question.

Passage – 2

Global warming, along with the impact of other negative human activities, is devastating our oceans. This has
led to an alarming rise in sea levels that could displace millions of people. The ocean is turning warmer, less
predictable, and more acidic, causing a decline in sh stocks and the death of coral reefs. The millions of tonnes
of plastics dumped into the oceans every year contaminate at least 700 species of marine life. Unregulated
over shing has already driven many marine species to extinction and this could eventually threaten global food
security. Oceans produce half the planet's oxygen and absorb more than 90% of the anthropogenic heat. The
ocean-led economy is estimated to be contributing more than $1.5 trillion a year to the global economy, with
millions employed in related sectors. We must understand that if the resources of the ocean are sustainably
harnessed, it could multiply economic bene ts while protecting the natural ecosystem. The solution lies in
collaboration among governments, businesses and coastal communities.

Q.113 [30465398]
Out of the following options, which one can be inferred to have captured the central idea of the passage?

a The importance of a sustainable ocean economy

b The problem of unregulated shing in oceans

c Plastic pollution and how it destroys the environment

d The importance of a balanced ecosystem

https://www.aspiration.ai/LAW/sis/Solution.jsp?qsetId=Lw5e7noFooA=&qsetName=LST Mock 20 2021 (CLAT) 173/208


12/18/2020 Mock Analysis

Solution:
 Answer key/Solution
Correct Answer : a
Your Answer : a
Option (a) is the most appropriate title of the passage. The passage talks about
the importance of the ocean economy and the role that it plays. Option (b) is narrow in scope while the other
two options are out of scope.

FeedBack Bookmark

Directions for questions 106 to 130: Each set of questions in this section is based on the reasoning and
arguments set out in the preceding passage. Please answer each question on the basis of what is stated or
implied in the corresponding passage. Do not rely on any information or facts other than the ones supplied to
you. In some instances, more than one option may be the answer to the question; in such a case, please choose
the option that most accurately and comprehensively answers the question.

Passage – 2

Global warming, along with the impact of other negative human activities, is devastating our oceans. This has
led to an alarming rise in sea levels that could displace millions of people. The ocean is turning warmer, less
predictable, and more acidic, causing a decline in sh stocks and the death of coral reefs. The millions of tonnes
of plastics dumped into the oceans every year contaminate at least 700 species of marine life. Unregulated
over shing has already driven many marine species to extinction and this could eventually threaten global food
security. Oceans produce half the planet's oxygen and absorb more than 90% of the anthropogenic heat. The
ocean-led economy is estimated to be contributing more than $1.5 trillion a year to the global economy, with
millions employed in related sectors. We must understand that if the resources of the ocean are sustainably
harnessed, it could multiply economic bene ts while protecting the natural ecosystem. The solution lies in
collaboration among governments, businesses and coastal communities.

Q.114 [30465398]
Which of the options, if true, would strengthen the author's belief as expressed in the passage?

a Oceanic resources comprise mainly of petroleum and natural gases.

b A viable cultivation of the ocean's resources can develop a country's economy.

c The oceanic resources surrounding a country can positively impact its culture.

d Sustainable management of the ocean's resources should be taken forward by NGOs.

https://www.aspiration.ai/LAW/sis/Solution.jsp?qsetId=Lw5e7noFooA=&qsetName=LST Mock 20 2021 (CLAT) 174/208


12/18/2020 Mock Analysis

Solution:
 Answer key/Solution
Correct Answer : b
Your Answer : b
Refer to the sentence, "We must understand that if the resources of the ocean are
sustainably harnessed, it could multiply economic bene ts while protecting the natural ecosystem." Hence,
option (b) is true. Option (c) cannot be substantiated in the light of the passage. The author has not gone into
detail when s/he described oceanic resources. Therefore, option (a) is untrue. Option (d) is out of scope
because the author does not mention anything about NGOs in the passage.
FeedBack Bookmark

Directions for questions 106 to 130: Each set of questions in this section is based on the reasoning and
arguments set out in the preceding passage. Please answer each question on the basis of what is stated or
implied in the corresponding passage. Do not rely on any information or facts other than the ones supplied to
you. In some instances, more than one option may be the answer to the question; in such a case, please choose
the option that most accurately and comprehensively answers the question.

Passage – 2

Global warming, along with the impact of other negative human activities, is devastating our oceans. This has
led to an alarming rise in sea levels that could displace millions of people. The ocean is turning warmer, less
predictable, and more acidic, causing a decline in sh stocks and the death of coral reefs. The millions of tonnes
of plastics dumped into the oceans every year contaminate at least 700 species of marine life. Unregulated
over shing has already driven many marine species to extinction and this could eventually threaten global food
security. Oceans produce half the planet's oxygen and absorb more than 90% of the anthropogenic heat. The
ocean-led economy is estimated to be contributing more than $1.5 trillion a year to the global economy, with
millions employed in related sectors. We must understand that if the resources of the ocean are sustainably
harnessed, it could multiply economic bene ts while protecting the natural ecosystem. The solution lies in
collaboration among governments, businesses and coastal communities.

Q.115 [30465398]
The author of the passage seems to imply that:

a Whales are prone to extinction.

b Corporate greed has led to ecological problems around the world.

c Multiple factors have led to the depletion of the ocean's resources.

d The world is facing the threat of food shortage in the coming century.

https://www.aspiration.ai/LAW/sis/Solution.jsp?qsetId=Lw5e7noFooA=&qsetName=LST Mock 20 2021 (CLAT) 175/208


12/18/2020 Mock Analysis

Solution:
 Answer key/Solution
Correct Answer : c
Your Answer : c
The author lists multiple factors such as unregulated shing and plastic wastes
as being responsible for the depletion of the ocean's resources. Therefore, option (c) is true. Option (b) is out
of context. Option (d) is wrong. Although the author mentions food security, s/ he hasn't mentioned any
timeframe. The author hasn't mentioned any particular marine species that is facing extinction. Hence, option
(a) is out of context.
FeedBack Bookmark

Directions for questions 106 to 130: Each set of questions in this section is based on the reasoning and
arguments set out in the preceding passage. Please answer each question on the basis of what is stated or
implied in the corresponding passage. Do not rely on any information or facts other than the ones supplied to
you. In some instances, more than one option may be the answer to the question; in such a case, please choose
the option that most accurately and comprehensively answers the question.

Passage – 3

Climate change continues to be a real and potent threat to agrobiodiversity, which will impact everything from
productivity to livelihoods across food and farm systems. Though India is dealing innovatively with climate
change - for example, through the development of drought and ood tolerant seed varieties, weather-based
agricultural advisories, promotion of millets, and small-scale irrigation - this year, we saw how climate-related
shocks made it di cult for farmers to deal with pest and locust attacks, as well as oods and cyclones.
Intensi ed food production systems with excessive use of chemicals and unsustainable farming practices
cause soil degradation, fast depletion of groundwater table and rapid loss of agro-biodiversity. These challenges
multiply with an increase in ________ of landholdings. In India, more than 86% farmers have less than two
hectares of land contributing around 60% of the total food grain production and over half the country's fruits and
vegetables.

Q.116 [30465398]
Which of the following is the central theme of the given passage?

a Farmers in India deal with climate change in an innovative way.

b Climate change impacts the agrobiodiversity and results in loss of agricultural yields.

c India is ill-equipped when it comes to dealing with pest attacks.

d In India, majority of farmers own very little land.

https://www.aspiration.ai/LAW/sis/Solution.jsp?qsetId=Lw5e7noFooA=&qsetName=LST Mock 20 2021 (CLAT) 176/208


12/18/2020 Mock Analysis

Solution:
 Answer key/Solution
Correct Answer : b
Your Answer : b
The given passage is primarily about the effect of climate change on agriculture
and its yield. Hence, option (b) is the answer. Options (a) and (c) are partially true and option (d) is true,
however, none of them is the central theme of the given passage.

FeedBack Bookmark

Directions for questions 106 to 130: Each set of questions in this section is based on the reasoning and
arguments set out in the preceding passage. Please answer each question on the basis of what is stated or
implied in the corresponding passage. Do not rely on any information or facts other than the ones supplied to
you. In some instances, more than one option may be the answer to the question; in such a case, please choose
the option that most accurately and comprehensively answers the question.

Passage – 3

Climate change continues to be a real and potent threat to agrobiodiversity, which will impact everything from
productivity to livelihoods across food and farm systems. Though India is dealing innovatively with climate
change - for example, through the development of drought and ood tolerant seed varieties, weather-based
agricultural advisories, promotion of millets, and small-scale irrigation - this year, we saw how climate-related
shocks made it di cult for farmers to deal with pest and locust attacks, as well as oods and cyclones.
Intensi ed food production systems with excessive use of chemicals and unsustainable farming practices
cause soil degradation, fast depletion of groundwater table and rapid loss of agro-biodiversity. These challenges
multiply with an increase in ________ of landholdings. In India, more than 86% farmers have less than two
hectares of land contributing around 60% of the total food grain production and over half the country's fruits and
vegetables.

Q.117 [30465398]
Which of the following, if true, would weaken the argument made by the author in the given passage?

a The excessive use of chemicals in farming does not affect the health of the soil.

b The use of fertilizers in farming increases the agriproducts.

c The excessive use of chemicals in farming affects the health of the soil.

d Intensi ed food production systems with unsustainable farming practices cause fast depletion of
groundwater table.

https://www.aspiration.ai/LAW/sis/Solution.jsp?qsetId=Lw5e7noFooA=&qsetName=LST Mock 20 2021 (CLAT) 177/208


12/18/2020 Mock Analysis

Solution:
 Answer key/Solution
Correct Answer : a
Your Answer : a
Option (b), (c) and (d) would strengthen the argument made by the author in the
given passage. Option (a) weakens the author's argument as according the passage the excessive use of
chemicals and unsustainable farming practices cause soil degradation, fast depletion of groundwater table
and rapid loss of agro-biodiversity.
FeedBack Bookmark

Directions for questions 106 to 130: Each set of questions in this section is based on the reasoning and
arguments set out in the preceding passage. Please answer each question on the basis of what is stated or
implied in the corresponding passage. Do not rely on any information or facts other than the ones supplied to
you. In some instances, more than one option may be the answer to the question; in such a case, please choose
the option that most accurately and comprehensively answers the question.

Passage – 3

Climate change continues to be a real and potent threat to agrobiodiversity, which will impact everything from
productivity to livelihoods across food and farm systems. Though India is dealing innovatively with climate
change - for example, through the development of drought and ood tolerant seed varieties, weather-based
agricultural advisories, promotion of millets, and small-scale irrigation - this year, we saw how climate-related
shocks made it di cult for farmers to deal with pest and locust attacks, as well as oods and cyclones.
Intensi ed food production systems with excessive use of chemicals and unsustainable farming practices
cause soil degradation, fast depletion of groundwater table and rapid loss of agro-biodiversity. These challenges
multiply with an increase in ________ of landholdings. In India, more than 86% farmers have less than two
hectares of land contributing around 60% of the total food grain production and over half the country's fruits and
vegetables.

Q.118 [30465398]
It can be inferred from the given passage that:

a India is fully equipped for any natural calamities that might be triggered by climate change.

b there are some crops that can be grown in any kind of climate.

c in spite of scienti c progress in agriculture India is not fully prepared to face the ill effects of climate
change.

d effects of climate change can be reversed.

https://www.aspiration.ai/LAW/sis/Solution.jsp?qsetId=Lw5e7noFooA=&qsetName=LST Mock 20 2021 (CLAT) 178/208


12/18/2020 Mock Analysis

Solution:
 Answer key/Solution
Correct Answer : c
Your Answer : c
Options (a), (b) and (d) cannot be inferred. Option (c) can be inferred from the
second sentence of the given passage. Refer to "Though India is dealing innovatively with climate change…
this year, we saw how climate-related shocks made it di cult for farmers to deal with pest and locust attacks,
as well as oods and cyclones."
FeedBack Bookmark

Directions for questions 106 to 130: Each set of questions in this section is based on the reasoning and
arguments set out in the preceding passage. Please answer each question on the basis of what is stated or
implied in the corresponding passage. Do not rely on any information or facts other than the ones supplied to
you. In some instances, more than one option may be the answer to the question; in such a case, please choose
the option that most accurately and comprehensively answers the question.

Passage – 3

Climate change continues to be a real and potent threat to agrobiodiversity, which will impact everything from
productivity to livelihoods across food and farm systems. Though India is dealing innovatively with climate
change - for example, through the development of drought and ood tolerant seed varieties, weather-based
agricultural advisories, promotion of millets, and small-scale irrigation - this year, we saw how climate-related
shocks made it di cult for farmers to deal with pest and locust attacks, as well as oods and cyclones.
Intensi ed food production systems with excessive use of chemicals and unsustainable farming practices
cause soil degradation, fast depletion of groundwater table and rapid loss of agro-biodiversity. These challenges
multiply with an increase in ________ of landholdings. In India, more than 86% farmers have less than two
hectares of land contributing around 60% of the total food grain production and over half the country's fruits and
vegetables.

Q.119 [30465398]
Which of the following is the most appropriate word to ll the blank in the given passage?

a formation

b development

c foundation

d fragmentation

https://www.aspiration.ai/LAW/sis/Solution.jsp?qsetId=Lw5e7noFooA=&qsetName=LST Mock 20 2021 (CLAT) 179/208


12/18/2020 Mock Analysis

Solution:
 Answer key/Solution
Correct Answer : d
Your Answer : b
The blank requires a word that would mean division. Hence, option (d) is the
correct answer. Other options have positive meanings and do not t in as the sentence starts with "These
challenges…".
FeedBack Bookmark

Directions for questions 106 to 130: Each set of questions in this section is based on the reasoning and
arguments set out in the preceding passage. Please answer each question on the basis of what is stated or
implied in the corresponding passage. Do not rely on any information or facts other than the ones supplied to
you. In some instances, more than one option may be the answer to the question; in such a case, please choose
the option that most accurately and comprehensively answers the question.

Passage – 3

Climate change continues to be a real and potent threat to agrobiodiversity, which will impact everything from
productivity to livelihoods across food and farm systems. Though India is dealing innovatively with climate
change - for example, through the development of drought and ood tolerant seed varieties, weather-based
agricultural advisories, promotion of millets, and small-scale irrigation - this year, we saw how climate-related
shocks made it di cult for farmers to deal with pest and locust attacks, as well as oods and cyclones.
Intensi ed food production systems with excessive use of chemicals and unsustainable farming practices
cause soil degradation, fast depletion of groundwater table and rapid loss of agro-biodiversity. These challenges
multiply with an increase in ________ of landholdings. In India, more than 86% farmers have less than two
hectares of land contributing around 60% of the total food grain production and over half the country's fruits and
vegetables.

Q.120 [30465398]
Which of the following is the logical corollary to the last sentence of the given passage?

a India has gone from being a net importer to a net exporter of food grains.

b India must stop the waste - one-third of the food India produces is wasted.

c The way India produces food must change through agroecology and sustainable production practices in
agriculture and allied sectors.

d A food system must provide enough nutritious food for all without compromising feeding future
generations.

https://www.aspiration.ai/LAW/sis/Solution.jsp?qsetId=Lw5e7noFooA=&qsetName=LST Mock 20 2021 (CLAT) 180/208


12/18/2020 Mock Analysis

Solution:
 Answer key/Solution
Correct Answer : c
Your Answer : c
The second half of the given passage talks about the intensi ed food production
systems with excessive use of chemicals and unsustainable farming practices and also refers to farmers in
India and their landholdings. Therefore, option (c) is the best corollary to the given passage. Option (a) is
beyond the scope of the passage. Options (b) and (d) are also out of scope.

FeedBack Bookmark

Directions for questions 106 to 130: Each set of questions in this section is based on the reasoning and
arguments set out in the preceding passage. Please answer each question on the basis of what is stated or
implied in the corresponding passage. Do not rely on any information or facts other than the ones supplied to
you. In some instances, more than one option may be the answer to the question; in such a case, please choose
the option that most accurately and comprehensively answers the question.

Passage – 4

The presumption that existence of acutely sharp divisions between liberals and conservatives, and competition
between ideologues and hardcore practitioners alone are endangering democracy in today's world would hardly
be correct. The shenanigans currently taking place in the world's oldest democracy, and efforts by the U.S.
President, Donald Trump, to negate the verdict of the recently held presidential elections, partake of a new set of
tactics, previously seen only in dictatorships. The extent to which Mr. Trump has been willing to go in his
attempt to negate the election, and the fact that a very sizeable segment of the U.S. population seems to be
backing him in this attempt, suggest that this is the 'new reality', which not only the U.S., but also the world, may
have to reckon with in times to come. The lack of shame-facedness on the part of the U.S. authorities even as
they engage in this, while lecturing the world about the virtues of democracy, represent a new pole in the
utilisation of fake news. The spectre that confronts democracy today is a grave one. In the case of the U.S., one
of the world's oldest democracies, what we are witnessing is a deep divide, one that is equally true of many
other democratic nations today, even if ngers are not being pointed at them. Bridging the divide between Mr.
Trump's entrenched supporters and the victorious camp of President-elect Joe Biden will, hence, be di cult. If
the size of a vote bank is an index of an individual's popularity in an election, it will be di cult to wish away the
70 million and odd votes that Mr. Trump has secured in this election - way higher than what he obtained in 2016.

Q.121 [30465398]
Which of the following conveys the main idea expressed in the passage?

a American is divided and so are other democracies of the world.

b Trump has garnered more support in 2020 presidential election than he did in 2016.

c Democracy has descended to a deplorable level.

d Fake news has gained the upper hand in democracies.

https://www.aspiration.ai/LAW/sis/Solution.jsp?qsetId=Lw5e7noFooA=&qsetName=LST Mock 20 2021 (CLAT) 181/208


12/18/2020 Mock Analysis

Solution:
 Answer key/Solution
Correct Answer : a
Option (a) is the answer. Refer to "In the case of the U.S., one of the world's oldest
democracies, what we are witnessing is a deep divide, one that is equally true of
many other democratic nations today…" None of the other options can convey the main idea.
FeedBack Bookmark

Directions for questions 106 to 130: Each set of questions in this section is based on the reasoning and
arguments set out in the preceding passage. Please answer each question on the basis of what is stated or
implied in the corresponding passage. Do not rely on any information or facts other than the ones supplied to
you. In some instances, more than one option may be the answer to the question; in such a case, please choose
the option that most accurately and comprehensively answers the question.

Passage – 4

The presumption that existence of acutely sharp divisions between liberals and conservatives, and competition
between ideologues and hardcore practitioners alone are endangering democracy in today's world would hardly
be correct. The shenanigans currently taking place in the world's oldest democracy, and efforts by the U.S.
President, Donald Trump, to negate the verdict of the recently held presidential elections, partake of a new set of
tactics, previously seen only in dictatorships. The extent to which Mr. Trump has been willing to go in his
attempt to negate the election, and the fact that a very sizeable segment of the U.S. population seems to be
backing him in this attempt, suggest that this is the 'new reality', which not only the U.S., but also the world, may
have to reckon with in times to come. The lack of shame-facedness on the part of the U.S. authorities even as
they engage in this, while lecturing the world about the virtues of democracy, represent a new pole in the
utilisation of fake news. The spectre that confronts democracy today is a grave one. In the case of the U.S., one
of the world's oldest democracies, what we are witnessing is a deep divide, one that is equally true of many
other democratic nations today, even if ngers are not being pointed at them. Bridging the divide between Mr.
Trump's entrenched supporters and the victorious camp of President-elect Joe Biden will, hence, be di cult. If
the size of a vote bank is an index of an individual's popularity in an election, it will be di cult to wish away the
70 million and odd votes that Mr. Trump has secured in this election - way higher than what he obtained in 2016.

Q.122 [30465398]
Which of the following would weaken the author's argument?

a Informational autocracy is the latest danger that threatens democracies.

b Issues of identity, or threats to identity, are becoming an important issue in elections across democracies

c There is a growing concern across the globe about increasing efforts to manipulate information in order to
perpetuate power.

d Authorities in power around the democratic world still uphold democracy with great reverence.

https://www.aspiration.ai/LAW/sis/Solution.jsp?qsetId=Lw5e7noFooA=&qsetName=LST Mock 20 2021 (CLAT) 182/208


12/18/2020 Mock Analysis

Solution:
 Answer key/Solution
Correct Answer : d
Given the present state of democracies in the US in particular and in the other
democratic countries of the world in general according to the author of the given
passage, options (a), (b) and (c) would strengthen the author's argument as they point out the shortcomings
observed in democracies around the world. On the contrary, option (d) weakens the author's argument.
FeedBack Bookmark

Directions for questions 106 to 130: Each set of questions in this section is based on the reasoning and
arguments set out in the preceding passage. Please answer each question on the basis of what is stated or
implied in the corresponding passage. Do not rely on any information or facts other than the ones supplied to
you. In some instances, more than one option may be the answer to the question; in such a case, please choose
the option that most accurately and comprehensively answers the question.

Passage – 4

The presumption that existence of acutely sharp divisions between liberals and conservatives, and competition
between ideologues and hardcore practitioners alone are endangering democracy in today's world would hardly
be correct. The shenanigans currently taking place in the world's oldest democracy, and efforts by the U.S.
President, Donald Trump, to negate the verdict of the recently held presidential elections, partake of a new set of
tactics, previously seen only in dictatorships. The extent to which Mr. Trump has been willing to go in his
attempt to negate the election, and the fact that a very sizeable segment of the U.S. population seems to be
backing him in this attempt, suggest that this is the 'new reality', which not only the U.S., but also the world, may
have to reckon with in times to come. The lack of shame-facedness on the part of the U.S. authorities even as
they engage in this, while lecturing the world about the virtues of democracy, represent a new pole in the
utilisation of fake news. The spectre that confronts democracy today is a grave one. In the case of the U.S., one
of the world's oldest democracies, what we are witnessing is a deep divide, one that is equally true of many
other democratic nations today, even if ngers are not being pointed at them. Bridging the divide between Mr.
Trump's entrenched supporters and the victorious camp of President-elect Joe Biden will, hence, be di cult. If
the size of a vote bank is an index of an individual's popularity in an election, it will be di cult to wish away the
70 million and odd votes that Mr. Trump has secured in this election - way higher than what he obtained in 2016.

Q.123 [30465398]
In the above passage, the author uses a/an__________ to strengthen his argument regarding the use of fake
news in democracy.

a Euphemism

b Hyperbole

c Metonymy

d Irony

https://www.aspiration.ai/LAW/sis/Solution.jsp?qsetId=Lw5e7noFooA=&qsetName=LST Mock 20 2021 (CLAT) 183/208


12/18/2020 Mock Analysis

Solution:
 Answer key/Solution
Correct Answer : d
Refer to "The lack of shame-facedness on the part of the U.S. authorities even as
they engage in this, while lecturing the world about the virtues of democracy,
represent a new pole in the utilisation of fake news." It is ironical that a country which is a democracy involves
undemocratic practices and at the same time lectures the world about the virtues of democracy. Hence, option
(d) is the answer.

FeedBack Bookmark

Directions for questions 106 to 130: Each set of questions in this section is based on the reasoning and
arguments set out in the preceding passage. Please answer each question on the basis of what is stated or
implied in the corresponding passage. Do not rely on any information or facts other than the ones supplied to
you. In some instances, more than one option may be the answer to the question; in such a case, please choose
the option that most accurately and comprehensively answers the question.

Passage – 4

The presumption that existence of acutely sharp divisions between liberals and conservatives, and competition
between ideologues and hardcore practitioners alone are endangering democracy in today's world would hardly
be correct. The shenanigans currently taking place in the world's oldest democracy, and efforts by the U.S.
President, Donald Trump, to negate the verdict of the recently held presidential elections, partake of a new set of
tactics, previously seen only in dictatorships. The extent to which Mr. Trump has been willing to go in his
attempt to negate the election, and the fact that a very sizeable segment of the U.S. population seems to be
backing him in this attempt, suggest that this is the 'new reality', which not only the U.S., but also the world, may
have to reckon with in times to come. The lack of shame-facedness on the part of the U.S. authorities even as
they engage in this, while lecturing the world about the virtues of democracy, represent a new pole in the
utilisation of fake news. The spectre that confronts democracy today is a grave one. In the case of the U.S., one
of the world's oldest democracies, what we are witnessing is a deep divide, one that is equally true of many
other democratic nations today, even if ngers are not being pointed at them. Bridging the divide between Mr.
Trump's entrenched supporters and the victorious camp of President-elect Joe Biden will, hence, be di cult. If
the size of a vote bank is an index of an individual's popularity in an election, it will be di cult to wish away the
70 million and odd votes that Mr. Trump has secured in this election - way higher than what he obtained in 2016.

Q.124 [30465398]
What can be inferred about Trump's attitude towards power as he refuses to accept the verdict of the US
presidential election 2020?

a He is a ghter who does not accept defeat at the hands of his opponent.

b He exhibits characteristics that are associated with a dictator.

c He wants to be in the o ce for the next term as well.

d He believes that he is still a more popular president than the president-elect Biden.

https://www.aspiration.ai/LAW/sis/Solution.jsp?qsetId=Lw5e7noFooA=&qsetName=LST Mock 20 2021 (CLAT) 184/208


12/18/2020 Mock Analysis

Solution:
 Answer key/Solution
Correct Answer : b
Only option (b) can be inferred from "efforts by the U.S. President, Donald Trump,
to negate the verdict of the recently held presidential elections, partake of a new
set of tactics, previously seen only in dictatorships." Other options cannot be inferred.
FeedBack Bookmark

Directions for questions 106 to 130: Each set of questions in this section is based on the reasoning and
arguments set out in the preceding passage. Please answer each question on the basis of what is stated or
implied in the corresponding passage. Do not rely on any information or facts other than the ones supplied to
you. In some instances, more than one option may be the answer to the question; in such a case, please choose
the option that most accurately and comprehensively answers the question.

Passage – 4

The presumption that existence of acutely sharp divisions between liberals and conservatives, and competition
between ideologues and hardcore practitioners alone are endangering democracy in today's world would hardly
be correct. The shenanigans currently taking place in the world's oldest democracy, and efforts by the U.S.
President, Donald Trump, to negate the verdict of the recently held presidential elections, partake of a new set of
tactics, previously seen only in dictatorships. The extent to which Mr. Trump has been willing to go in his
attempt to negate the election, and the fact that a very sizeable segment of the U.S. population seems to be
backing him in this attempt, suggest that this is the 'new reality', which not only the U.S., but also the world, may
have to reckon with in times to come. The lack of shame-facedness on the part of the U.S. authorities even as
they engage in this, while lecturing the world about the virtues of democracy, represent a new pole in the
utilisation of fake news. The spectre that confronts democracy today is a grave one. In the case of the U.S., one
of the world's oldest democracies, what we are witnessing is a deep divide, one that is equally true of many
other democratic nations today, even if ngers are not being pointed at them. Bridging the divide between Mr.
Trump's entrenched supporters and the victorious camp of President-elect Joe Biden will, hence, be di cult. If
the size of a vote bank is an index of an individual's popularity in an election, it will be di cult to wish away the
70 million and odd votes that Mr. Trump has secured in this election - way higher than what he obtained in 2016.

Q.125 [30465398]
What does the author indicate when he uses the phrase 'new reality' in the given passage?

a A new way of electing a presidential candidate through postal voting.

b Biden's alleged attempt of horse trading in the recently concluded US presidential election.

c Trump's attempt to negate the recently concluded US presidential election in which Biden has been elected.

d Losing an election in spite of being a popular presidential candidate.

https://www.aspiration.ai/LAW/sis/Solution.jsp?qsetId=Lw5e7noFooA=&qsetName=LST Mock 20 2021 (CLAT) 185/208


12/18/2020 Mock Analysis

Solution:
 Answer key/Solution
Correct Answer : c
Option (a) cannot be inferred. Nothing is either suggested about postal voting in
the given passage. Option (b) cannot be inferred either. Option (d) is incorrect.
Option (c) is correct. Refer to "The extent to which Mr. Trump has been willing to go in his attempt to negate
the election, and the fact that a very sizeable segment of the U.S. population seems to be backing him in this
attempt, suggest that this is the 'new reality'…"

FeedBack Bookmark

Directions for questions 106 to 130: Each set of questions in this section is based on the reasoning and
arguments set out in the preceding passage. Please answer each question on the basis of what is stated or
implied in the corresponding passage. Do not rely on any information or facts other than the ones supplied to
you. In some instances, more than one option may be the answer to the question; in such a case, please choose
the option that most accurately and comprehensively answers the question.

Passage – 5

The Regional Comprehensive Economic Partnership (RCEP) of Asia and the Paci c signed the mega trade deal
among the 10 Southeast Asian countries and ve FTA partners (Australia, China, Japan, New Zealand and South
Korea) on November 15. While the deal was moving towards its nal stages, India chose to pull out at the
eleventh hour, seven years after joining the negotiation. The decision to come out of the RCEP was
predominantly "the unsatisfactory address of India's outstanding issues and concerns". During negotiations,
though several issues were ironed out towards preparing the mutual ground for trade and market access, the
concern around protecting the interests of India's domestic producers couldn't be sidelined. Though the post-
RCEP scenario might offer India an opportunity for greater market access, imports from partner countries would
increase at a much higher rate than export in a reduced tariff situation. The door to the RCEP is open for India in
the future. The question is: Should it do so?

Q.126 [30465398]
Out of the following options, which one is an appropriate logical corollary to the passage given above?

a India's re-joining the RCEP might rile Pakistan.

b In the recent past, India has implemented several macroeconomic plans.

c If India joins, it should examine the possibilities of stronger bilateral trade with other countries.

d These reforms effectively strengthen India's base if India joins the RCEP.

https://www.aspiration.ai/LAW/sis/Solution.jsp?qsetId=Lw5e7noFooA=&qsetName=LST Mock 20 2021 (CLAT) 186/208


12/18/2020 Mock Analysis

Solution:
 Answer key/Solution
Correct Answer : c
Your Answer : c
If India wishes to re-join the RCEP, it should see that trade agreements with other
countries remain favourable. Therefore, option (c) is a possible logical corollary to the passage. Option (d) is
wrong because we are not sure what reforms is the option talking about. Option (b) is wrong because
thematically it is not connected with the core topic of the passage's discussion. Option (a) is wrong because
nothing has been mentioned in the passage about Pakistan with regard to India's joining the RCEP.
FeedBack Bookmark

Directions for questions 106 to 130: Each set of questions in this section is based on the reasoning and
arguments set out in the preceding passage. Please answer each question on the basis of what is stated or
implied in the corresponding passage. Do not rely on any information or facts other than the ones supplied to
you. In some instances, more than one option may be the answer to the question; in such a case, please choose
the option that most accurately and comprehensively answers the question.

Passage – 5

The Regional Comprehensive Economic Partnership (RCEP) of Asia and the Paci c signed the mega trade deal
among the 10 Southeast Asian countries and ve FTA partners (Australia, China, Japan, New Zealand and South
Korea) on November 15. While the deal was moving towards its nal stages, India chose to pull out at the
eleventh hour, seven years after joining the negotiation. The decision to come out of the RCEP was
predominantly "the unsatisfactory address of India's outstanding issues and concerns". During negotiations,
though several issues were ironed out towards preparing the mutual ground for trade and market access, the
concern around protecting the interests of India's domestic producers couldn't be sidelined. Though the post-
RCEP scenario might offer India an opportunity for greater market access, imports from partner countries would
increase at a much higher rate than export in a reduced tariff situation. The door to the RCEP is open for India in
the future. The question is: Should it do so?

Q.127 [30465398]
Out of the following options, which one captures the central idea of the passage?

a The role of RCEP in the international trade scenario

b The RCEP question

c The importance of RCEP in the Indian manufacturing scenario

d The role of RCEP in a post Covid-19 world

https://www.aspiration.ai/LAW/sis/Solution.jsp?qsetId=Lw5e7noFooA=&qsetName=LST Mock 20 2021 (CLAT) 187/208


12/18/2020 Mock Analysis

Solution:
 Answer key/Solution
Correct Answer : b
Your Answer : b
The passage talks about India leaving the RCEP at the eleventh hour and the
possibility of whether India will rejoin the RCEP. Hence, option (b) is the most appropriate answer that
captures the central idea of the passage. Option (a) is wrong because we are not sure from the passage how
the RCEP can in uence international trade practices. Option (c) is too speci c because in the light of the
passage, we are not sure how rejoining the RCEP can have an impact on the Indian manufacturing scenario.
Option (d) is out of scope.
FeedBack Bookmark

Directions for questions 106 to 130: Each set of questions in this section is based on the reasoning and
arguments set out in the preceding passage. Please answer each question on the basis of what is stated or
implied in the corresponding passage. Do not rely on any information or facts other than the ones supplied to
you. In some instances, more than one option may be the answer to the question; in such a case, please choose
the option that most accurately and comprehensively answers the question.

Passage – 5

The Regional Comprehensive Economic Partnership (RCEP) of Asia and the Paci c signed the mega trade deal
among the 10 Southeast Asian countries and ve FTA partners (Australia, China, Japan, New Zealand and South
Korea) on November 15. While the deal was moving towards its nal stages, India chose to pull out at the
eleventh hour, seven years after joining the negotiation. The decision to come out of the RCEP was
predominantly "the unsatisfactory address of India's outstanding issues and concerns". During negotiations,
though several issues were ironed out towards preparing the mutual ground for trade and market access, the
concern around protecting the interests of India's domestic producers couldn't be sidelined. Though the post-
RCEP scenario might offer India an opportunity for greater market access, imports from partner countries would
increase at a much higher rate than export in a reduced tariff situation. The door to the RCEP is open for India in
the future. The question is: Should it do so?

Q.128 [30465398]
In the light of the passage, it can be inferred that in a post-RCEP scenario:

a India's foreign exchange reserves will increase.

b India will be exporting more to countries like Brazil and Germany.

c There will be a favourable balance of trade.

d There will be an unfavourable balance of trade.

https://www.aspiration.ai/LAW/sis/Solution.jsp?qsetId=Lw5e7noFooA=&qsetName=LST Mock 20 2021 (CLAT) 188/208


12/18/2020 Mock Analysis

Solution:
 Answer key/Solution
Correct Answer : d
Your Answer : a
If the imports exceed exports, an unfavourable balance of trade, or a trade de cit,
exists. Hence, option (d) is correct. Refer to the sentence, "Though the post-RCEP scenario might offer India
an opportunity for greater market access, imports from partner countries would increase at a much higher
rate than export in a reduced tariff situation." Option (c) is exactly the opposite of what the passage says.
Options (a) and (b) are wrong because they are far-fetched and cannot be inferred.
FeedBack Bookmark

Directions for questions 106 to 130: Each set of questions in this section is based on the reasoning and
arguments set out in the preceding passage. Please answer each question on the basis of what is stated or
implied in the corresponding passage. Do not rely on any information or facts other than the ones supplied to
you. In some instances, more than one option may be the answer to the question; in such a case, please choose
the option that most accurately and comprehensively answers the question.

Passage – 5

The Regional Comprehensive Economic Partnership (RCEP) of Asia and the Paci c signed the mega trade deal
among the 10 Southeast Asian countries and ve FTA partners (Australia, China, Japan, New Zealand and South
Korea) on November 15. While the deal was moving towards its nal stages, India chose to pull out at the
eleventh hour, seven years after joining the negotiation. The decision to come out of the RCEP was
predominantly "the unsatisfactory address of India's outstanding issues and concerns". During negotiations,
though several issues were ironed out towards preparing the mutual ground for trade and market access, the
concern around protecting the interests of India's domestic producers couldn't be sidelined. Though the post-
RCEP scenario might offer India an opportunity for greater market access, imports from partner countries would
increase at a much higher rate than export in a reduced tariff situation. The door to the RCEP is open for India in
the future. The question is: Should it do so?

Q.129 [30465398]
The author of the passage is most likely to agree with which of the following:

a India should rejoin the RCEP only if its exports outweigh the imports.

b India should rejoin the RCEP only if the other member countries request it to join.

c India should rejoin the RCEP only if the USA joins the organisation.

d India should rejoin the RCEP only if it is allowed to export the maximum among the member nations.

https://www.aspiration.ai/LAW/sis/Solution.jsp?qsetId=Lw5e7noFooA=&qsetName=LST Mock 20 2021 (CLAT) 189/208


12/18/2020 Mock Analysis

Solution:
 Answer key/Solution
Correct Answer : a
Your Answer : a
If option (a) is true, it would mean a favourable trade balance for India. Hence,
India would stand to gain. Therefore, the author is most likely to agree with option (a). Option (c) is wrong
because there is no mention of the USA. Option (b) is wrong because other member nations may or may not
request India to rejoin the RCEP. Option (d) is wrong because the passage doesn't mention if India should be
allowed to export the maximum. The use of the superlative makes the option incorrect.
FeedBack Bookmark

Directions for questions 106 to 130: Each set of questions in this section is based on the reasoning and
arguments set out in the preceding passage. Please answer each question on the basis of what is stated or
implied in the corresponding passage. Do not rely on any information or facts other than the ones supplied to
you. In some instances, more than one option may be the answer to the question; in such a case, please choose
the option that most accurately and comprehensively answers the question.

Passage – 5

The Regional Comprehensive Economic Partnership (RCEP) of Asia and the Paci c signed the mega trade deal
among the 10 Southeast Asian countries and ve FTA partners (Australia, China, Japan, New Zealand and South
Korea) on November 15. While the deal was moving towards its nal stages, India chose to pull out at the
eleventh hour, seven years after joining the negotiation. The decision to come out of the RCEP was
predominantly "the unsatisfactory address of India's outstanding issues and concerns". During negotiations,
though several issues were ironed out towards preparing the mutual ground for trade and market access, the
concern around protecting the interests of India's domestic producers couldn't be sidelined. Though the post-
RCEP scenario might offer India an opportunity for greater market access, imports from partner countries would
increase at a much higher rate than export in a reduced tariff situation. The door to the RCEP is open for India in
the future. The question is: Should it do so?

Q.130 [30465398]
A possible reason for India's withdrawal from the RCEP is:

a India was peeved at higher excise duties on trade.

b Japan's unfavourable trade balance with India.

c India's consternation was not addressed.

d the partisan politics of West Asian countries.

https://www.aspiration.ai/LAW/sis/Solution.jsp?qsetId=Lw5e7noFooA=&qsetName=LST Mock 20 2021 (CLAT) 190/208


12/18/2020 Mock Analysis

Solution:
 Answer key/Solution
Correct Answer : c
Your Answer : c
Refer to the sentence, "The decision to come out of the RCEP was predominantly
"the unsatisfactory address of India's outstanding issues and concerns". 'Concern' and 'consternation' are
synonyms. Option (a) is wrong because nothing has been mentioned in the passage on higher excise duty.
Option (b) is wrong because Japan's trade balance is not mentioned in the passage. Option (d) is wrong
because no West Asian country is mentioned in the passage.
FeedBack Bookmark

Directions for questions 131 to 133: Following is an array of questions to test your reasoning ability in different
situations. Answer each of them according to the question asked in each of them respectively:

Q.131 [30465398]
Statement: "You are hereby terminated from the post of Manager with immediate effect and are hereby
discharged from all your duties. You are instructed to handover your duties to the new appointee" - A line in a
termination letter.

Assumptions:
I. The terminated individual has got a new job at some other organisation
II. The new appointee would be as e cient as his predecessor

a If only assumption I is implicit

b If only assumption II is implicit

c If neither I nor II is implicit

d If both I and II are implicit.

Solution:
 Answer key/Solution
Correct Answer : c
Your Answer : b
Neither the e ciency of the new appointee nor the future of the terminated
individual can be ascertained by the statement. Hence no assumption is implicit.
FeedBack Bookmark

Directions for questions 131 to 133: Following is an array of questions to test your reasoning ability in different
situations. Answer each of them according to the question asked in each of them respectively:

https://www.aspiration.ai/LAW/sis/Solution.jsp?qsetId=Lw5e7noFooA=&qsetName=LST Mock 20 2021 (CLAT) 191/208


12/18/2020 Mock Analysis

Q.132 [30465398]
Statement: It is desirable for an individual to get retired from work after the age of 60.

Assumptions:
I. The o ces do not allow people to work post 60 years of age.
II. The mental and physical faculties seem to dial down that that age

a If only assumption I is implicit

b If only assumption II is implicit

c If neither I nor II is implicit

d If both I and II are implicit.

Solution:
 Answer key/Solution
Correct Answer : b
Your Answer : b
I is not implicit as the eligibility to work in o ces has not been given in the
statement however II is implicit as it reasons out the statement.
FeedBack Bookmark

Directions for questions 131 to 133: Following is an array of questions to test your reasoning ability in different
situations. Answer each of them according to the question asked in each of them respectively:

Q.133 [30465398]
Statement 1: Some oranges are peaches.
Statement 2: All peaches are mangoes.

Conclusion I: All oranges are peaches.


Conclusion II: All oranges being mangoes is a possibility.

a If only Conclusion I follows

b If only Conclusion II follows

c If both Conclusions I and II follow

d If Neither Conclusion I nor II follows

https://www.aspiration.ai/LAW/sis/Solution.jsp?qsetId=Lw5e7noFooA=&qsetName=LST Mock 20 2021 (CLAT) 192/208


12/18/2020 Mock Analysis

Solution:
 Answer key/Solution
Correct Answer : b
Your Answer : b

FeedBack Bookmark

Q.134 [30465398]
Navi is sitting at a circular table along with ve other friends, facing the centre. Tavi faces Ravi and sits to the
immediate right of Savi, who neither faces Yuvi nor Kavi. Which of the following statements is de nitely true
regarding the seating arrangement?

a Yuvi is an immediate neighbor of Savi.

b Kavi is an immediate neighbor of Ravi.

c Navi is an immediate neighbor of Ravi.

d Tavi sits second to the right of Kavi.

Solution:
 Answer key/Solution
Correct Answer : c
Your Answer : d

FeedBack Bookmark

https://www.aspiration.ai/LAW/sis/Solution.jsp?qsetId=Lw5e7noFooA=&qsetName=LST Mock 20 2021 (CLAT) 193/208


12/18/2020 Mock Analysis

Q.135 [30465398]
Arun is the brother of Siya, who is the daughter of Viru’s father. Jaya is the sister of Niya, whose father is Kalam.
Jaya is the mother of Arun, who has exactly one brother and one sister. Dev is the husband of Niya’s only sister.
How is Kalam related to Viru?

a Son-in-law

b Brother

c Grandson

d Grandfather

Solution:
 Answer key/Solution
Correct Answer : d
Your Answer : d

FeedBack Bookmark

Sec 5

https://www.aspiration.ai/LAW/sis/Solution.jsp?qsetId=Lw5e7noFooA=&qsetName=LST Mock 20 2021 (CLAT) 194/208


12/18/2020 Mock Analysis

Directions for questions 136 to 140: Answer the questions on the basis of the information given below.

The two pie charts given below show the number of voters in ve wards of a city during the municipal elections
held in 2015 and in 2020. The total number of voters in 2020 was 4.5 lakh.

Q.136 [30465398]
If 48% of the total voters in the city during the municipal elections of 2015 were female, then nd the number of
male voters during those elections.

a 153600

b 160000

c 166400

d 154800

Solution:
 Answer key/Solution
Correct Answer : c
Your Answer : c
Required number of male voters = 0.52 × 320000 = 166400.
FeedBack Bookmark

https://www.aspiration.ai/LAW/sis/Solution.jsp?qsetId=Lw5e7noFooA=&qsetName=LST Mock 20 2021 (CLAT) 195/208


12/18/2020 Mock Analysis

Directions for questions 136 to 140: Answer the questions on the basis of the information given below.

The two pie charts given below show the number of voters in ve wards of a city during the municipal elections
held in 2015 and in 2020. The total number of voters in 2020 was 4.5 lakh.

Q.137 [30465398]
Which ward witnessed a maximum increase in the number of voters during the 2020 municipal elections when
compared to 2015?

a B

b D

c E

d C

https://www.aspiration.ai/LAW/sis/Solution.jsp?qsetId=Lw5e7noFooA=&qsetName=LST Mock 20 2021 (CLAT) 196/208


12/18/2020 Mock Analysis

Solution:
 Answer key/Solution
Correct Answer : b
Your Answer : b

FeedBack Bookmark

Directions for questions 136 to 140: Answer the questions on the basis of the information given below.

The two pie charts given below show the number of voters in ve wards of a city during the municipal elections
held in 2015 and in 2020. The total number of voters in 2020 was 4.5 lakh.

Q.138 [30465398]
During 2020 municipal elections in which ward was the percentage of total voters lower than the percentage of
voters during the 2015 elections?

a A

b B

c E

https://www.aspiration.ai/LAW/sis/Solution.jsp?qsetId=Lw5e7noFooA=&qsetName=LST Mock 20 2021 (CLAT) 197/208


12/18/2020 Mock Analysis

d Both A and E

Solution:
 Answer key/Solution
Correct Answer : d
Your Answer : d

FeedBack Bookmark

Directions for questions 136 to 140: Answer the questions on the basis of the information given below.

The two pie charts given below show the number of voters in ve wards of a city during the municipal elections
held in 2015 and in 2020. The total number of voters in 2020 was 4.5 lakh.

Q.139 [30465398]
If during the 2020 elections 75% of the voters from ward C had their Jan-Dhan accounts linked to their Aadhar
cards, then nd the number of voters from ward C who did not have their accounts linked to their Aadhar cards.

https://www.aspiration.ai/LAW/sis/Solution.jsp?qsetId=Lw5e7noFooA=&qsetName=LST Mock 20 2021 (CLAT) 198/208


12/18/2020 Mock Analysis

a 24750

b 28570

c 25280

d 48275

Solution:
 Answer key/Solution
Correct Answer : a
Your Answer : a

FeedBack Bookmark

Directions for questions 136 to 140: Answer the questions on the basis of the information given below.

The two pie charts given below show the number of voters in ve wards of a city during the municipal elections
held in 2015 and in 2020. The total number of voters in 2020 was 4.5 lakh.

Q.140 [30465398]
What was the approximate percentage increase in the number of voters in wards B and E from 2015 to 2020?

a 25%

b 20%

https://www.aspiration.ai/LAW/sis/Solution.jsp?qsetId=Lw5e7noFooA=&qsetName=LST Mock 20 2021 (CLAT) 199/208


12/18/2020 Mock Analysis

c 18%

d 32%

Solution:
 Answer key/Solution
Correct Answer : b
Your Answer : b

FeedBack Bookmark

Directions for questions 141 to 145: Answer the questions on the basis of the information given below.

A contractor is making plans to renovate an old house that has two bed rooms with attached bathrooms, a hall
with balcony, a kitchen and a study room. The dimensions of each part of the house is given in the table below.
The height of the ceiling from the oor is 12 feet throughout the house.

Q.141 [30465398]
If the contractor plans on laying new tiles in the entire house, then what is the total oor area that needs to be
tiled?

a 924 square feet

b 972 square feet

c 884 square feet

d 996 square feet

Solution:
 Answer key/Solution
Correct Answer : b

FeedBack Bookmark

https://www.aspiration.ai/LAW/sis/Solution.jsp?qsetId=Lw5e7noFooA=&qsetName=LST Mock 20 2021 (CLAT) 200/208


12/18/2020 Mock Analysis

Directions for questions 141 to 145: Answer the questions on the basis of the information given below.

A contractor is making plans to renovate an old house that has two bed rooms with attached bathrooms, a hall
with balcony, a kitchen and a study room. The dimensions of each part of the house is given in the table below.
The height of the ceiling from the oor is 12 feet throughout the house.

Q.142 [30465398]
The hall is to be painted using a special emulsion paint which costs Rs.1,050 per liter and one liter of paint can
cover 30 square feet. Find the cost of painting all the walls of the hall, if there are two doors and two windows
having a total area of 192 square feet.

a Rs.19,600

b Rs.12,600

c Rs.23,600

d Rs.25,200

Solution:
 Answer key/Solution
Correct Answer : d

FeedBack Bookmark

https://www.aspiration.ai/LAW/sis/Solution.jsp?qsetId=Lw5e7noFooA=&qsetName=LST Mock 20 2021 (CLAT) 201/208


12/18/2020 Mock Analysis

Directions for questions 141 to 145: Answer the questions on the basis of the information given below.

A contractor is making plans to renovate an old house that has two bed rooms with attached bathrooms, a hall
with balcony, a kitchen and a study room. The dimensions of each part of the house is given in the table below.
The height of the ceiling from the oor is 12 feet throughout the house.

Q.143 [30465398]
If "0.5 feet × 0.5 feet" tiles come in boxes of 16, which has an MRP of Rs.350 and the labour cost is Rs.1,100,
then nd the cost of tiling two bathrooms and the balcony.

a Rs.11,900

b Rs.12,000

c Rs.13,000

d Rs.10,800

Solution:
 Answer key/Solution
Correct Answer : c

FeedBack Bookmark

https://www.aspiration.ai/LAW/sis/Solution.jsp?qsetId=Lw5e7noFooA=&qsetName=LST Mock 20 2021 (CLAT) 202/208


12/18/2020 Mock Analysis

Directions for questions 141 to 145: Answer the questions on the basis of the information given below.

A contractor is making plans to renovate an old house that has two bed rooms with attached bathrooms, a hall
with balcony, a kitchen and a study room. The dimensions of each part of the house is given in the table below.
The height of the ceiling from the oor is 12 feet throughout the house.

Q.144 [30465398]
The total cost of tiling the oors of the kitchen and study room was Rs.8,000, which included labor charges of
Rs.850. If tiles come in boxes of 20 each, then nd the cost of a box of tiles.

a Rs.650

b Rs.250

c Rs.450

d Rs.550

Solution:
 Answer key/Solution
Correct Answer : a

FeedBack Bookmark

https://www.aspiration.ai/LAW/sis/Solution.jsp?qsetId=Lw5e7noFooA=&qsetName=LST Mock 20 2021 (CLAT) 203/208


12/18/2020 Mock Analysis

Directions for questions 141 to 145: Answer the questions on the basis of the information given below.

A contractor is making plans to renovate an old house that has two bed rooms with attached bathrooms, a hall
with balcony, a kitchen and a study room. The dimensions of each part of the house is given in the table below.
The height of the ceiling from the oor is 12 feet throughout the house.

Q.145 [30465398]
If each bedroom has two doors and two windows having a total area of 92 square feet, then nd the cost of
painting all the walls and ceiling of the bedrooms at the rate of Rs.25 per square feet.

a Rs.42,000

b Rs.35,000

c Rs.34,540

d Rs.45,500

Solution:
 Answer key/Solution
Correct Answer : b

FeedBack Bookmark

Directions for questions 146 to 150: Answer the questions on the basis of the information given below.

The distance between Kanpur and Lucknow is 87 km and there are two intermediate stations in between,
namely Unnao and Aishbagh. Unnao is at a distance of 27 km from Kanpur. An express train running between
Kanpur and Lucknow faces some technical problems at Unnao. It then runs at 75% of its original speed. Due to
this, it reaches 20 minutes late.

Q.146 [30465398]
Find the original time for the journey from Unnao to Lucknow.

a 45 minutes

https://www.aspiration.ai/LAW/sis/Solution.jsp?qsetId=Lw5e7noFooA=&qsetName=LST Mock 20 2021 (CLAT) 204/208


12/18/2020 Mock Analysis

b 80 minutes

c 50 minutes

d 60 minutes

Solution:
 Answer key/Solution
Correct Answer : d

FeedBack Bookmark

Directions for questions 146 to 150: Answer the questions on the basis of the information given below.

The distance between Kanpur and Lucknow is 87 km and there are two intermediate stations in between,
namely Unnao and Aishbagh. Unnao is at a distance of 27 km from Kanpur. An express train running between
Kanpur and Lucknow faces some technical problems at Unnao. It then runs at 75% of its original speed. Due to
this, it reaches 20 minutes late.

Q.147 [30465398]
What is speed of the train after it faces technical problems?

a 60 km/hr

b 80 km/hr

c 45 km/hr

d 75 km/hr

https://www.aspiration.ai/LAW/sis/Solution.jsp?qsetId=Lw5e7noFooA=&qsetName=LST Mock 20 2021 (CLAT) 205/208


12/18/2020 Mock Analysis

Solution:
 Answer key/Solution
Correct Answer : c

FeedBack Bookmark

Directions for questions 146 to 150: Answer the questions on the basis of the information given below.

The distance between Kanpur and Lucknow is 87 km and there are two intermediate stations in between,
namely Unnao and Aishbagh. Unnao is at a distance of 27 km from Kanpur. An express train running between
Kanpur and Lucknow faces some technical problems at Unnao. It then runs at 75% of its original speed. Due to
this, it reaches 20 minutes late.

Q.148 [30465398]
If the train takes 36 seconds to cross the platform at Aishbagh, which is of length 240 m, nd the length of the
train.

a 480 m

b 360 m

c 260 m

d 600 m

Solution:
 Answer key/Solution
Correct Answer : b

FeedBack Bookmark

https://www.aspiration.ai/LAW/sis/Solution.jsp?qsetId=Lw5e7noFooA=&qsetName=LST Mock 20 2021 (CLAT) 206/208


12/18/2020 Mock Analysis

Directions for questions 146 to 150: Answer the questions on the basis of the information given below.

The distance between Kanpur and Lucknow is 87 km and there are two intermediate stations in between,
namely Unnao and Aishbagh. Unnao is at a distance of 27 km from Kanpur. An express train running between
Kanpur and Lucknow faces some technical problems at Unnao. It then runs at 75% of its original speed. Due to
this, it reaches 20 minutes late.

Q.149 [30465398]
Additional information for questions 149 and 150: The express train starts at 7:00 AM from Kanpur and it stops
for 4 minutes each at the two intermediate stations on a normal day.

What is the time taken by the train to cover the distance from Kanpur to Lucknow?

a 1 hour 35 minutes

b 1 hour 27 minutes

c 1 hour 45 minutes

d 1 hour 25 minutes

Solution:
 Answer key/Solution
Correct Answer : a

FeedBack Bookmark

Directions for questions 146 to 150: Answer the questions on the basis of the information given below.

The distance between Kanpur and Lucknow is 87 km and there are two intermediate stations in between,
namely Unnao and Aishbagh. Unnao is at a distance of 27 km from Kanpur. An express train running between
Kanpur and Lucknow faces some technical problems at Unnao. It then runs at 75% of its original speed. Due to
this, it reaches 20 minutes late.

Q.150 [30465398]
Mr. Gupta travels by the express train from Kanpur to Lucknow to go to o ce, which starts at 9:00 AM. He takes
5 minutes to walk out and hire a taxi from the station to his o ce. If the taxi travels at 30 km/hr and drops him
exactly on time, then nd the distance between the station and his o ce.

a 12 km

b 10 km

https://www.aspiration.ai/LAW/sis/Solution.jsp?qsetId=Lw5e7noFooA=&qsetName=LST Mock 20 2021 (CLAT) 207/208


12/18/2020 Mock Analysis

c 11.5 km

d 10.5 km

Solution:
 Answer key/Solution
Correct Answer : b

FeedBack Bookmark

https://www.aspiration.ai/LAW/sis/Solution.jsp?qsetId=Lw5e7noFooA=&qsetName=LST Mock 20 2021 (CLAT) 208/208

You might also like